SlideShare a Scribd company logo
1 of 80
ĐẠI HOC THÁI NGUYÊN
TRƯ NG ĐẠI HOC KHOA HOC
Tải tài liệu tại sividoc.com
Viết đề tài giá sinh viên – ZALO:0973.287.149-TEAMLUANVAN.COM
TRAN TH± HƯ NG
M T SO DẠNG TOÁN CỰC TR± TRONG L P
HÀM MŨ VÀ HÀM HYPERBOLIC
LU N VĂN THẠC SĨ TOÁN HOC
THÁI NGUYÊN - 2018
ĐẠI HOC THÁI NGUYÊN
TRƯ NG ĐẠI HOC KHOA HOC
Tải tài liệu tại sividoc.com
Viết đề tài giá sinh viên – ZALO:0973.287.149-TEAMLUANVAN.COM
TRAN TH± HƯ NG
M T SO DẠNG TOÁN CỰC TR± TRONG L P
HÀM MŨ VÀ HÀM HYPERBOLIC
Chuyên ngành: Phương pháp toán sơ cap
Mã so: 84 60 113
LU N VĂN THẠC SĨ TOÁN HOC
NGƯŐI HƯŐNG DAN KHOA HOC
GS.TSKH. Nguyen Văn M u
THÁI NGUYÊN - 2018
i
Viết đề tài giá sinh viên – ZALO:0973.287.149-TEAMLUANVAN.COM
Mục lục
M ĐAU ii
1 M t so kien th c liên quan đen các hàm mũ và hyperbolic 1
1.1 Tính chat cơ bản của các hàm mũ và hyperbolic . . . . . . . . . . . 1
1.1.1 Tính chat cơ bản của hàm mũ . . . . . . . . . . . . . . . . 1
1.1.2 Tính chat cơ bản của hàm hyperbolic . . . . . . . . . . . . 2
1.2 Đȁng thức sinh bỏi hàm mũ và hàm hyperbolic . . . . . . . . . . . 5
1.3 M t so bat đȁng thức chứa đạo hàm và tích phân quan trong .................. 10
2 Bat đang th c và c c trị trong lỚp hàm mũ và hàm hyperbolic 27
2.1 Bat đȁng thức trong lóp hàm mũ và hàm hyperbolic............................... 27
2.2 Các dạng toán cục trị sinh bỏi hàm mũ và hyperbolic............................. 47
3 M t so dạng toán liên quan 59
3.1 Các phương trình đại so giải bang phương pháp hàm hyperbolic ........... 59
3.2 Khảo sát m t so lóp phương trình chứa hàm mũ và hàm hyperbolic . 67
KET LU N 74
TÀI LI U THAM KHÂO 75
ii
Viết đề tài giá sinh viên – ZALO:0973.287.149-TEAMLUANVAN.COM
M ĐAU
Chuyên đe ve các hàm siêu vi t (hàm mũ và logarit) đưoc đe c p ỏ lóp 12 b c
trung hoc pho thông. Vì v y các ứng dụng của hàm mũ và logarit không đưoc đe c p
trong các lóp 10 và 11. Đ c bi t, do giảm tải chương trình, lóp các hàm hyperbobic
cũng không đưoc đưa vào SGK. Các hàm này chỉ đưoc khảo sát trong chương trình
boi dưõng HSG ỏ các lóp chuyên Toán phục vụ các kỳ thi HSG quoc gia, Olympic
khu vục và quoc te.
Trong các kì thi hoc sinh giỏi toán các cap b c THPT và Olympic khu vục và
quoc te, các bài toán liên quan tói hàm mũ và hàm hyperbolic thưòng xuyên đưoc
đe c p. Những dạng toán này thưòng đưoc xem là thu c loại khó vì phan kien thức
sâu sac ve hàm mũ và hàm hyperbolic không nam trong chương trình chính thức của
giáo trình Đại so và Giải tích b c trung hoc pho thông.
Đe đáp ứng nhu cau boi dưõng giáo viên và boi dưõng hoc sinh giỏi ve chuyên đe
hàm mũ và hàm hyperbolic, tôi chon đe tài lu n văn “M t so dạng toán cục trị trong
lóp hàm mũ và hàm hyperbolic”.
Lu n văn nham tong hop m t so tính chat của hàm mũ và hàm hyperbolic và moi
quan h giữa chúng. Tiep theo, xét các bài toán cục trị, khảo sát m t so lóp phương
trình, bat phương trình cùng m t so dạng toán đại so có sử dụng tính chat hàm mũ,
hàm ngưoc của nó là hàm logarit và hàm hyperbolic.
Cau trúc lu n văn gom 3 chương:
Chương 1. M t so kien thức liên quan đen các hàm mũ và hyperbolic.
Chương 2. Bat đȁng thức và cục trị trong lóp hàm mũ và hàm hyperbolic.
Chương 3. M t so dạng toán liên quan.
Lu n văn sử dụng m t so dạng toán và bài t p từ các tài li u [1]-[9] và m t so
iii
Viết đề tài giá sinh viên – ZALO:0973.287.149-TEAMLUANVAN.COM
đe thi Olympic liên quan đen hàm hàm mũ và hàm hyperbolic trong những năm gan
đây.
Lu n văn đưoc hoàn thành vói sụ hưóng dȁn của GS.TSKH. Nguyen Văn M u.
Em xin bày tỏ lòng biet ơn sâu sac đoi vói sụ quan tâm, đ ng viên và sụ chỉ bảo
hưóng dȁn của thay.
Tác giả xin chân thành cảm ơn Ban giám hi u, Phòng Đào tạo, Khoa Toán và các
thay cô trưòng Đại hoc Khoa hoc - Đại hoc Thái Nguyên đã tạo đieu ki n thu n loi
cho tác giả hoàn thành bản lu n này.
Nhân dịp này, tác giả cũng xin chân thành cảm ơn Ban giám hi u và các đong
nghi p trưòng THPT Nguyen Bỉnh Khiêm, huy n Vĩnh Bảo, thành pho Hải Phòng
đã tạo đieu ki n cho tác giả hoàn thành tot nhi m vụ hoc t p và công tác của mình.
Thái Nguyên, ngày 20 tháng 5 năm 2018
Tác gia lu n văn
Tran Thị HưỜng
1
Viết đề tài giá sinh viên – ZALO:0973.287.149-TEAMLUANVAN.COM
Chương 1
M t so kien th c liên quan đen các hàm mũ
và hyperbolic
1.1 Tính chat cơ ban cua các hàm mũ và hyperbolic
1.1.1 Tính chat cơ ban cua hàm mũ
Xét hàm so mũ dạng f (x) = ax
vói 0 < a /= 1.
* T p xác định: Df = R.
* T p giá trị: If = R+
.
* Tính đơn đi u: Hàm so f(x) = ax
đong bien trên R khi a > 1 và nghịch bien
trên R khi 0 < a < 1.
Nh n xét 1.1. Đo thị hàm so mũ có ti m c n ngang là trục Ox ve phía −∞ khi a > 1
và ti m c n ngang là trục Ox ve phía +∞ khi 0 < a < 1.
Tiep theo, ta xét m t so đȁng thức trong lóp hàm mũ.
Tính chat 1.1 (Công thức tính đạo hàm).
(ex
)′ = ex
; (eu
)′ = u′eu
,
(ax
)′ = ax
ln a; (au
)′ = u′au
ln a.
Tính chat 1.2 (Đong nhat thức trong lóp hàm mũ). Cho 0 < a /= 1. Khi đó:
2
Viết đề tài giá sinh viên – ZALO:0973.287.149-TEAMLUANVAN.COM
a) af (x) = ag(x) ⇔ f(x) = g(x).
b) Giả sử b > 0. Khi đó af (x) = b ⇔ f (x) = loga b.
c) af (x) > ag(x) ⇔ (a − 1)( f (x) − g(x)) > 0.
d) Giả sử b > 0. Khi đó af (x) > b ⇔ (a−1)(f(x)−loga b) > 0.
1.1.2 Tính chat cơ ban cua hàm hyperbolic
Trong phan này, ta trình bày m t so tính chat của các hàm mũ đ c bi t, đó là các
hàm hyperbolic sinh bỏi e±x
.
Tính chat 1.3 (Hàm sin hyperbolic). Hàm sin hyperbolic
là hàm so lẻ trên R và
sinhx =
ex −e−x
2
sinhx ≥ 0, ∀x ≥ 0, sinhx < 0, ∀x < 0.
(sinhx)′ = coshx;(sinhu)′ = u′ coshu.
Ta có (sinhx)′ = coshx ≥ 1, ∀x ∈ R nên hàm so sinhx đong bien trên R.
Do (sinhx)′′ = sinhx nên hàm so sinhx loi trên (0;+∞) và lõm trên (−∞;0).
Tính chat 1.4 (Hàm cosin hyperbolic). Hàm cosin hyperbolic
là hàm so chȁn trên R.
coshx =
ex +e−x
2
Ta có (coshx)′ = sinhx;(coshu)′ = u′ sinhu và (coshx)′ = sinhx nên hàm so
coshx đong bien trên (0;+∞) và nghịch bien trên (−∞;0).
Do (coshx)′′ = coshx ≥ 1, ∀x ∈ R coshx loi trên R.
3
Viết đề tài giá sinh viên – ZALO:0973.287.149-TEAMLUANVAN.COM
cosh2
x
( ) = −
Tính chat 1.5 (Hàm tang hyperbolic). Hàm tang hyperbolic
tanhx sinhx ex −e−x
là hàm so lẻ trên R.
Ta có
=
coshx
=
ex + e−x
(tanhx)′ =
1
;(tanhu)′ =
u′
.
cosh2
x
Do
cosh2
u
(tanhx)′ =
1
> 0, ∀x ∈ R
nên hàm so tanh x đong bien trên R.
Tính chat 1.6 (Hàm cotang hyperbolic). Hàm cotang
cothx coshx ex +e−x
là hàm so lẻ trên R  {0}.
Ta có
=
sinhx
=
ex − e−x
(cothx)′ =
−1
;(cothu)′ =
−u′
sinh2
x sinh2
u
và cothx ′
1
sinh2
x < 0, ∀x ∈ R{0} nên hàm so cothx đong bien trên mői
khoảng (−∞;−1) và (1;+∞).
Tính chat 1.7 (Công thức khai trien tong và hi u).
cosh(x + y) = coshx.coshy + sinhx.sinhy, (1.1)
cosh(x − y) = coshx.coshy − sinhx.sinhy, (1.2)
sinh(x + y) = sinhxcoshy + coshx.sinhy, (1.3)
4
Viết đề tài giá sinh viên – ZALO:0973.287.149-TEAMLUANVAN.COM
−
ChÝng minh. Ta có
sinh(x − y) = sinhxcoshy − coshx.sinhy, (1.4)
tanh(x + y) =
tanhx + tanhy
, (1.5)
1 + tanhxtanhy
tanh(x y) =
tanhx − tanhy
. (1.6)
1 − tanhxtanhy
coshx coshy sinhx sinhy ex +e−x ey +e−y ex −e−x ey −e−y
. + . =
2 2
+
2 2
Từ đó, suy ra (1.1).
ex+y +e−x−y
=
2
= cosh(x+y).
Tiep theo, trong công thức (1.1) thay y bang −y, ta thu đưoc
cosh(x−y) = coshx.cosh(−y) +sinhx.sinh(−y)
= coshx.coshy − sinhx.sinhy.
Ta nh n đưoc (1.2).
Các công thức còn lại (1.3)-(1.6) đưoc chứng minh tương tụ.
Từ công thức c ng ta cũng de dàng chứng minh đưoc các công thức nhân sau đây.
Tính chat 1.8 (Công thức khai trien góc nhân hai và nhân ba).
sinh(2x) = 2 sinh x. cosh x,
cosh(2x) = cosh2
x +sinh2
x = 2cosh2
x − 1 = 1+2sinh2
x,
tanh(2x) =
2tanhx
,
1+tanh2
x
sinh(3x) = 4sinh3
x +3sinhx,
5
Viết đề tài giá sinh viên – ZALO:0973.287.149-TEAMLUANVAN.COM
−
2
−
cosh(3x) = 4cosh3
x −3coshx.
Tính chat 1.9 (Công thức bien đoi tích thành tong).
coshxcoshy =
1
[cosh(x+y)+cosh(x y)],
2
sinhx sinhy =
1
[cosh(x + y) − cosh(x − y)] ,
sinhxcoshy =
1
[sinh(x+y)+sinh(x y)].
2
Tính chat 1.10 (Công thức bien đoi tong thành tích).
coshx+coshy = 2cosh
x + y
cosh
x − y
,
2 2
coshx − coshy = 2sinh
x + y
sinh
x − y
,
2 2
sinhx + sinhy = 2sinh
x + y
cosh
x − y
,
2 2
sinhx − sinhy = 2cosh
x + y
sinh
x − y
,
2 2
tanhx + tanhy =
sinh(x + y)
,
coshx.coshy
tanhxtanhy =
sinh(x − y)
.
coshx.coshy
1.2 Đang th c sinh bƠi hàm mũ và hàm hyperbolic
Trong phan này ta xét m t so dạng toán áp dụng các tính chat của hàm mũ và các
hàm hyperbolic.
Bài toán 1.1. Tính giá trị các hàm hyperbolic tại điem ln 2 và ln 3.
L i giai. Theo định nghĩa, ta có
sinh ln 2 eln 2 − e−ln 2 3
( ) =
2
=
4
6
Viết đề tài giá sinh viên – ZALO:0973.287.149-TEAMLUANVAN.COM
3
3
√
√
3
⇔ a
⇔
3
−
⇔ ⇔
−
và
cosh(ln 2) = eln 2
+e−ln 2 =
5
;tanh(ln 2) =
3
;coth(ln 2
5
Tương tụ, ta có
2
sinh ln 3
4 5
eln 3 −e−ln 3 4
) =
3
.
( ) =
2
=
3
và
cosh(ln 3) = eln 3
+ e−ln 3 =
5
;tanh(ln 3) =
4
;coth(ln 3) =
5
.
2 3 5 4
Bài toán 1.2. Giải các phương trình và bat phương trình sau:
a. e2x +e−2x
5
=
2
.
b. e3x
−e−3x
≥
8
.
c. ax
− a−x
<
2
, 0 < a /= 1.
L i giai.
a. e2x
+e−2x =
5
2
e2x +e−2x 5
2
=
4
⇔ cosh2x = cosh(ln 2) ⇔ 2x = ±ln 2 do hàm coshx đong bien trên (0;+∞) và
nghịch bien trên (−∞;0).
V y phương trình có hai nghi m x = ±ln 2.
b. e3x
− e−3x ≥
8
⇔
e3x e−3x 4
2
≥
3
⇔ sinh3x ≥ sinh(ln 3) ⇔ 3x ≥ ln 3 do hàm sinhx đong bien trên R.
V y bat phương trình có nghi m x ≥ ln 3.
c. ax
− a−x <
3
2
exln a
− e−xln a <
3
2
exln a e−xln a 3
2
<
4
⇔ sinh(xln a) < sinh(ln 2) ⇔ xln a < ln 2.
ln 2
Neu a > 1 bat phương trình có nghi m x <
ln a
⇔ x < loga2.
Neu 0 < a < 1 bat phương trình có nghi m x >
ln 2
x > log 2.
ln a
Bài toán 1.3. Chứng minh bat đȁng thức
a. coshx ≥ 1, ∀x ∈ R.
7
Viết đề tài giá sinh viên – ZALO:0973.287.149-TEAMLUANVAN.COM
1
∀ /
x =
b. −1 < tanhx < 1, ∀x ∈ R.
c. cothx > 1, ∀x > 0 & cothx < −1, ∀x < 0.
d. sinh3
x+cosh3
x ≥ 1, ∀x ∈ R.
L i giai.
a. coshx ≥ 1. ∀x ∈ R.
Ta có coshx =
ex +e−x
2
≥
√
ex.e−x = 1. Dau đȁng thức xảy ra khi và chỉ khi x = 0.
Từ đó ta có đieu can chứng minh.
b. −1 < tanhx < 1. ∀x ∈ R.
Ta có tanhx ex −e−x e2x
−1
1
2
=
ex + e−x
=
e2x + 1
= −
e2x + 1
.
Do e2x
> 0 nên −1 < tanhx < 1, ∀x ∈ R.
c. cothx > 1, ∀x > 0 & cothx < −1, ∀x < 0.
Ta có cothx = , x = 0 và từ phan b. Từ đó ta có đieu can chứng minh.
tanhx
d. Ta có
3 3 ex −e−x 3
ex +e−x 3
e3x +3e−x
e3x +e−x +e−x +e−x √
4
=
4
≥ e3x.e−x.e−x.e−x
= 1.
Dau đȁng thức xảy ra khi và chỉ khi x = 0.
Từ đó ta có đieu can chứng minh.
Tiep theo, ta xét m t so ví dụ minh hoa sau.
Bài toán 1.4. Chứng minh rang
a.
sinhx + sinh3x + sinh5x
cosh x+cosh 3x + cosh 5x = tanh3x.
b.
tanhx + tanh2x − tanh3x = tanh xtanh 2x tanh3x.
L i giai.
4
2
2
sinh x + cosh + =
8
Viết đề tài giá sinh viên – ZALO:0973.287.149-TEAMLUANVAN.COM
2
−
2 2 2 2
2 2 2 2
a.
sinhx + sinh3x + sinh 5x
cosh x+cosh 3x + cosh 5x
2sinh 3xcos2x + sinh3x
=
2cosh3xcosh2x + cosh3x
= tanh3x.
b.
tanh x+tanh 2x − tanh 3x = tanh x+tanh 2x − tanh(x + 2x)
tanh x+tanh 2x
= tanh x + tanh2x −
1 + tanhxtanh 2x
= (tanhx+tanh2x) 1
1
1 + tanh x tanh 2x
= (tanhx+tanh2x)
tanh x tanh 2x
1 + tanh x tanh 2x
=
tanhx+tanh 2x
tanhx tanh2x = tanh x tanh2xtanh 3x.
1 + tanh x tanh 2x
Bài toán 1.5. Tính các tong sau:
Sn = sinhx + sinh2x + sinh3x + ·· · + sinhnx
Tn = coshx+2cosh2x+3cosh3x+···+ncoshnx
L i giai. Neu x = 0 thì Sn = 0
Xét x =
/ 0. Nhân cả hai ve Sn vói sinh
x
, ta đưoc
Sn.2sinh
x
2
x x x
= 2sinh
2
sinhx + 2sinh
2
sinh2x + 2sinh
2
sinh3x + ...
x
+ 2sinh
2
sinhnx
= cosh
3x
− cosh
x
+ cosh
5x
− cosh
3x
+ cosh
7x
−cosh
5x
+···+ cosh
2n+1
x −cosh
2n−1
x
9
Viết đề tài giá sinh viên – ZALO:0973.287.149-TEAMLUANVAN.COM
2
2
2
2
= 2
x
2
2 2 2 2 2
4sinh2 x
2 2 2
2
2
2
2n+ 1 x
Suy ra
= cosh
2
x − cosh
2
.
cosh 2n+1
x − cosh x
Sn = 2
x
2
.
2sinh 2
Neu x = 0 thì Tn = 1+2+3+···+n =
n(n+1)
.
Xét x /= 0, thì
Sn
′ = coshx+2cosh2x+3cosh3x+···+ncoshnx.
Suy ra
′ cosh 2n+1
x−cosh x
!′
2sinh 2
2n+1
sinh 2n+1
x − 1
sinh x
2sinh x
− cosh x
cosh 2n+1
x − cosh x
(2n + 1)sinh 2n+1
xsinh x
− sinh2 x
− cosh 2n+1
xcosh x
+ cosh2 x
= 2 2
2 2 2 2
4sinh2 x
1 + 2n+1
(cosh(n + 1)x − coshnx) − 1
(cosh(n + 1)x + coshnx)
= 2
2 x
2
4sinh 2
=
1+ncosh(n+1)x−(n+1)coshnx
4sinh2 x
=
n(cosh(n+1)x−coshnx)+1−coshnx
4sinh2 x
2nsinh 2n+1
xsinh x
− 2sinh2 nx
. 2 2 2
4sinh2 x
nsinh 2n+1
xsinh x
− sinh2 nx
= 2 2 2
.
2sinh2 x
Bài toán 1.6. Chứng minh bat đȁng thức cosh(5x − 7) ≥
√
25x2 − 70x + 50.
Tn = Sn =
10
Viết đề tài giá sinh viên – ZALO:0973.287.149-TEAMLUANVAN.COM
5
√
L i giai. Xét hàm so y = cosh2
(5x − 7) − (5x − 7)2
+ 5x − 1, ∀x ∈ R.
Ta có
y′ = 5sinh(2(5x −7))− 10(5x −7) + 5;y′′ = 50cosh(2(5x −7))− 50 ≥ 0.
Do đó
y ≥ y
7
+y′
7
x−
7
.
5 5 5
Ta có
y
7
= 7;y′
7
= 5
5 5
nên
cosh2
(5x−7)−(5x−7)2
+5x−1 ≥ 7+5 x−
7
.
Suy ra cosh2
(5x−7) ≥ 25x2
−70x+50.
Từ đó ta có đieu can chứng minh cosh(5x − 7) ≥ 25x2 − 70x + 50.
1.3 M t so bat đang th c ch a đạo hàm và tích phân quan
trong
Trong phan này, ta trình bày các bat đȁng thức Landau, Kolmogorov và Steklov
đoi vói đạo hàm và tích phân các hàm so đ c bi t. Và trình bày m t so áp dụng vào
chứng minh các bat đȁng thức và giải bài toán cục trị vói hàm mũ và hàm hyperbolic.
Định lí 1.1 (Bat đȁng thức Landau). Cho f : R → R là m t hàm của lóp C2
. Giả sử
f và f′′ bị ch n. Đ t
M0 = sup| f (x)|,M1 = sup| f ′(x)|, M2 = sup| f ′′(x)|.
x∈R x∈R x∈R
Chứng minh rang f′ bị ch n và M1 ≤ 2
√
M0M2.
ChÝng minh. Đau tiên, ta nh n thay rang neu M2 = 0 thì hàm f chỉ đáp ứng đưoc
11
Viết đề tài giá sinh viên – ZALO:0973.287.149-TEAMLUANVAN.COM
−
h
r
M
2
M
2
1
đieu ki n liên tục. Do v y, không mat tính tong quát ta có the giả sử M2 > 0. Lay
x ∈ R và m t giá trị tùy ý h > 0.
Theo công thức Taylor ton tại t ∈ (x;x + 2h) sao cho
f (x + 2h) = f (x) + 2h f ′(x) + 2h2
f ′′(x).
Do v y
f ′
(x) =
f (x +h)− f (x)
h f ′′
(x).
2h
Lay modun và áp dụng các giả thiet ban đau, ta thu đưoc
| f ′(x)| ≤
M0
+ M2h.
V y f′(x) bị ch n.
M t khác, chon h =
M0
, ta có
M2
|f′(x)| ≤ M0 :
r
M0
+M2
r
M0
⇔ M1 ≤ 2
√
M0M2.
Định lí 1.2 (Bat đȁng thức Kolmogorov). Cho f : R → R là m t hàm của lóp C3
.
Giả sử f và f′′′ bị ch n.
Đ t M0 = sup| f (x)|, M1 = sup| f ′(x)|,M3 = sup| f ′′′(x)|.
x∈R x∈R
′
x∈R
2 1
a. Chứng minh rang f bị ch n. Hơn nữa, M1 ≤
2
(9M0 M3)3
b. f′′ có bị ch n hay không ?
ChÝng minh. a. Co định x ∈ R và h /= 0. Áp dụng công thức khai trien Taylor cho
các giá trị ỏ giữa h và x+h, ta thu đưoc:
|f(x+h)− f(x)−hf ′
(x) −
h2
f
2
h3
′′
(x)| ≤ M3
6
12
Viết đề tài giá sinh viên – ZALO:0973.287.149-TEAMLUANVAN.COM
| |
và
V y nên
|f(x−h)− f(x)+hf ′
(x) −
h2
f
2
h3
′′
(x)| ≤ M3
6
.
2h|f′(x)| = |( f(x — h) − f (x) + hf′ (x) −
h2
f′′
2 (x))
−( f (x +h)− f (x)− h f
H thức này cho ta bat đȁng thức
′
(x) −
h2
f
2
′′
(x)) + f (x + h) − f (x − h)|.
2h|f′(x)| ≤ | f(x — h) − f (x) + hf′ (x) −
h2
f′′
2 (x)|
+|f (x+ h)− f (x)−h f
Do đó
′
(x) −
h2
f
2
′′
(x)|+ | f(x+h)|+ | f (x−h)| =
M3h3
3
+ 2M0.
V y f′(x) bị ch n.
|f′ (x)| ≤
M0
+
h
m3h2
6
=: ψ(x).
0 0
−1 1
M t khác, ψ đạt cục tieu tại h = (3M M3 )3 .
1 2 1 1 2 1
Từ ψ(h) =
2
(9M0 M3)3 ta có M1 ≤
2
(9M0 M3)3 .
b. Theo giả thiet và theo ý câu (a) các hàm f′ và f′′′ = ( f′′)′ là các hàm bị ch n.
Áp dụng bat đȁng thức Landau, ta có f′′ bị ch n.
Định lí 1.3 (Bat đȁng thức Landau-Kolmogorov). Cho f : R → R là m t hàm của
lóp Cn
. Giả sử f và f (n) bị ch n.
a. Chứng minh rang f (n−1) bị ch n.
b. Từ đó suy ra tat cả các đạo hàm f(k) vói moi 1 ≤ k ≤ n−1 đeu bị ch n.
c. Đ t Mk = sup f (k)(x) .
x∈R
Chứng minh rang Mk > 0 ứng vói moi 0 ≤ k ≤ n; k ∈ N.
d. Sử dụng các hàm uk = 2k−1
MkMk
−1
;0 ≤ k ≤ n,k ∈ N đe chứng minh rang
k(n−k) 1−k k
−
Mk ≤ 2 2 M0 Mn .
n
n
1
13
Viết đề tài giá sinh viên – ZALO:0973.287.149-TEAMLUANVAN.COM
. ( j)
. k . k
. . ≤ .
m
m
−
∑
j=1
( ) ∑(
k=1
) n−1 . ≤ ∑
k=1
n−1
ChÝng minh. Theo công thức khai trien Taylor, ta có
n−1
kj
f
x
.
2M Mnkn
Do v y
∑
j=1
( ). ≤ 0 +
n!
.
n−1
C
n−1
kj
f(j)
x
. n−1
C 2M Mnkn
.
∑
k=1
Đieu đó cho thay
n 1 ∑
k= j
( ) ∑
k=1
n−1
0 +
n!
.
n−1
1
f( j)
x
n−1
1 n−1−k
Ck
kj.
n−1
Ck
2M
Mnkn
m
M t khác ∑
k=1
(−1)m−k
Ck
k j
= 0 neu 1 ≤ j ≤ m − 1,
m
ho c ∑ (−1)m−k
Ck
k j
= m! neu j = m.
k=1
Ta thay rang tat cả các bat đȁng thức bên trái đeu không xảy ra ngoại trừ trưòng
hop j = n − 1.
Theo đó ∀x ∈ R, ta đeu có
(n−1) n−1
k
Mnkn
|f
V y nên f(n−1)(x) bị ch n.
(x)| ≤ ∑Cn 1(2M0 +
k=1 n!
).
b. ta de dàng có f(k) vói moi 1 ≤ k ≤ n − 1 đeu bị ch n bang phương pháp qui
nạp.
Th t v y theo giả thiet và theo ý a ta có f và f (n−1) bị ch n nên ta có f (n−2) bị
ch n.
Tương tụ, ta có f và f (n−2) bị ch n nên ta có f (n−3) bị ch n.
Cứ tiep tục quá trình trên, ta có tat cả các đạo hàm f (k) vói moi 1 ≤ k ≤ n − 1 đeu
bị ch n.
n!
.
j!
−
j!
.
.
. −
j! 0 + .
14
Viết đề tài giá sinh viên – ZALO:0973.287.149-TEAMLUANVAN.COM
0 n
a
|f(x)|
a
|f (x)|
( ) =
2 a
( )
x
( ) =
2
( )
c. Neu Mk = 0 thì f là m t đa thức có b c cao nhat là k − 1. Từ f là hàm bị ch n,
do đó f chỉ thỏa mãn đưoc đieu ki n liên tục của giả thiet. Đieu đó cho thay Mk > 0
vói moi 0 ≤ k ≤ n,k ∈ N.
d. Áp dụng bat đȁng thức Landau, ta có
Mk ≤
√
2Mk−1Mk+1,∀1 ≤ k ≤ n−1.
Đieu này. cho thay bat đȁng thức
u1 ≤ u2 ≤ ··· ≤ un,
trong đó
(u1u2 ...uk)n
≤ (u1u2 ...un)k
.
Áp dụng bat đȁng thức trong trưòng hop này, ta có đưoc
k
k(n−k) 1−
n
k
Mk ≤ 2 2 M Mn
.
V y ta có đieu phải chứng minh.
Định lí 1.4 (Bat đȁng thức Steklov). Giả sử hàm f (x) khả vi liên tục trên khoảng
(a;b) và sao cho f(a) = f(b) = 0. Khi đó, có bat đȁng thức
∫ b
2 (b−a)2 ∫ b
′ 2
ChÝng minh. Bieu dien hàm f đã cho ỏ dạng
f x
1h∫ x
f′ t dt −
∫ b
f ′ t dy
i 1
∫ b
sign x −t f ′ t dt
dx ≤ 4
dx. (1.7)
a
( ) .
15
Viết đề tài giá sinh viên – ZALO:0973.287.149-TEAMLUANVAN.COM
2 2 2
( )
2 a
( )
4 a a
( )
4 a
( )
a
|f(x)|
π2 a
|f (x)|
Áp dụng bat đȁng thức Buniakowski ta có
|f x |
h1
∫ b
sign x −t f′ t dt
i2
≤
1
∫ b
|sign x −t | dt
∫ b
|f′ t | dt
b− a
∫ b
| f ′
t | dt
lay tích phân theo x trên (a;b), ta đưoc
∫ b
2
(b−a)2 ∫ b
′ 2
a
|f (x)|
Định lý đưoc chứng minh.
dx ≤
4 a
|f (x)| dx.
Nh n xét 1.2. Nh n xét rang, có the làm tot bat đȁng thức (1.7) bỏi bat đȁng thức
∫ b
2 (b−a)2 ∫ b
′ 2
Tuy nhiên, chứng minh bat đȁng thức (1.8) không đơn giản vì nó đòi hỏi nhieu
kien thức của Toán cao cap (theo sụ hieu biet của chúng tôi), nên chúng tôi sẽ không
trình bày chứng minh ay trong lu n văn này.
Tiep theo, ta xét m t so dạng bat đȁng thức sinh bỏi đạo hàm liên quan đen các
bat đȁng thức co đien.
Ta chứng minh bo đe sau.
Bo đe 1.1. Cho f : R → R là m t hàm khả vi và
x
lim f(x) ≥ 0 và f′(x) > 0,∀x ∈ R.
Khi đó f (x) > 0 vói moi x ∈ R.
→−∞
ChÝng minh. Th t v y, giả sử ton tại m t so x0 ∈ R sao cho f(x0) < 0. Theo giả
thiet, ta có f′ > 0 nên f(x) < f(x0) vói x < x0.
Do đó lim
x→−∞
f (x) ≤ f (x0) < 0, mâu thuȁn.
Bài toán 1.7 (Kolmogorov). Cho f : R → R là m t hàm khả vi b c 3 trên R sao cho
f, f ′, f ′′, f ′′′ đeu là các so dương. Hơn nữa, ta giả sử f ′′′(x) ≤ f (x), ∀x ∈ R. Chứng
2
= ( )
( ) . =
dx ≤ dx. (1.8)
16
Viết đề tài giá sinh viên – ZALO:0973.287.149-TEAMLUANVAN.COM
−
2
minh rang f ′(x) < 2 f(x), ∀x ∈ R.
L i giai. Đau tiên ta chứng minh
lim
x→−∞
và
lim
x→−∞
f ′(x) = 0
f′′(x) = 0.
Th t v y, theo định lí giá trị trung bình ton tại ξn ∈ (−n2
;−n), sao cho
f (−n2
) − f (−n) = (n − n2
) f ′(ξn) < 0.
Theo giả thiet ban đau, các giói hạn của f và f ′ tại −∞ ton tại và chúng không
âm.
Ta giả sử phản chứng rang lim
x→∞ f ′(x) > 0.
Lay giói hạn trong đȁng thức trên tại −∞ ta có 0 = −∞ (mâu thuȁn).
Do đó lim
x→−∞
f′(x) = 0. ′′
Chứng minh tương tụ, ta cũng có lim f
x→−∞
(x) = 0.
′′′
Bây giò ta áp dụng bo đe, theo giả thiet, ta có f (x) ≤ f (x). Do v y
f ′′(x) f ′′′(x) ≤ f ′′(x) f (x) ≤ f ′′(x) f (x)+ f ′2
(x), ∀x ∈ R.
Sử dụng bat đȁng thức này, đong thòi áp dụng bo đe trên cho hàm
f (x) f ′(x)
1
( f ′′(x))2
,
2
ta thu đưoc
1
( f ′′(x))2
< f (x) f ′(x), ∀x ∈ R.
M t khác, do f > 0 và f ′′′ > 0, nên ta có
f ′2
(x) ≤ 2 f (x) f ′′(x),∀x ∈ R.
17
Viết đề tài giá sinh viên – ZALO:0973.287.149-TEAMLUANVAN.COM
2
b − a
Ket hop hai bat đȁng thức trên, ta thu đưoc
1 f′2
(x) 2
2
(
2 f(x)
) ≤
1
( f
2
′′
(x))2
< f(x)f ′(x).
Do v y f ′3
(x) < 8 f 3
(x),∀x ∈ R.
Đieu đó cho thay f′(x) < 2f(x).
Nh n xét 1.3. Ta có the chứng minh m t bat đȁng thức trên bang m t bat đȁng thức
mạnh hơn bang cách thay the hang so 2, bang m t hang so nhỏ hơn 2.
Th t v y, thêm
1
f ′(x) f′′(x) vào 2 ve của bat đȁng thức
2
1
( f′′(x))2
< f(x)f′(x), ∀x ∈ R
và áp dụng giả thiet f′′′(x) ≤ f(x) ta có
1
[ f ′(x) f ′′′(x) + ( f ′′′(x))2
] < f (x) f ′(x) +
1
f ′(x) f ′′′(x) ≤
3
f (x) f ′(x).
2 2 2
Áp dụng bo đe trên, ta có
1
f ′(x) f ′′(x) <
3
f 2
(x),∀x ∈ R.
2 4
Nhân hai ve vói f′(x) và áp dụng bo đe trên ta suy ra rang
1
f ′3
(x) <
1
f 3
(x),∀x ∈ R.
6 4
′ 3 1
Do đó f (x) < (
2
)2 f (x) < 2 f (x).
Bài toán 1.8. Cho f : [a; b] → R là m t hàm mà có đạo hàm (không nhat thiet phải
hữu hạn) tại bat kì tại điem nào trong [a;b].
Chứng minh rang ton tại x0 ∈ [a;b] sao cho
|
f (a) − f (b)
| ≤ | f ′
(x0)|.
18
Viết đề tài giá sinh viên – ZALO:0973.287.149-TEAMLUANVAN.COM
10
| − |
−
b − a
−
L i giai. Đoi vói bat kì so nguyên 0 ≤ n ≤ 10. Xét dãy cn = a +
(b − a)n
. ta có
f(a) f(b)
|
b − a
9
∑ f (cn+1 f (cn) .
n=0
Như v y, ton tại m t so nguyên k vói 0 ≤ k ≤ 0 sao cho
|
f (ck+1) − f (ck)
| ≥ |
f (a) − f (b)
|.
ck+1 −ck b − a
Xét k1 là so nhỏ nhat thỏa mãn đieu ki n trên và xác định a1 = ck1 và b1 = ck1+1.
Tiep theo, ta l p lu n tương tụ vói các khoảng so thục I1 := [a1; b1] ta có khoảng
so thục I2 := [a2;b2]. Cứ tiep tục l p lu n như trên, ta có m t chuői các so thục
In := [an;bn] sao cho bn −an = 10−n
(b−a) và
|
f (bn) − f (an)
| ≥ |
f (a) − f (b)
|.
bn −an b − a
Từ In là m t khoảng đóng |In| = (bn −an) → 0 và I1 ⊃ I2 ⊃ ··· ⊃ In ⊃ ... theo bo
đe Cantor ton tại duy nhat m t giá trị x0 sao cho x0 ∈
n
T
1
In.
Đ c bi t, đieu này cho thay x0 = lim an = lim bn.
≥
n→
f
∞ n→∞
Ta thay rang trong bat đȁng thức |
(bn) − f (an)
| ≥ |
f (a) − f (b)
| có the thay the
m t trong hai giá trị an ho c b
bn an
n bỏi giá trị x0.
b − a
Theo đó, do bat đȁng thức trên ket hop vói đong thức nhat cơ bản, ta có
f (bn) − f (an)
=
f (bn)− f (x0) bn − x0
+
f (x0) − f (an) x0 −an
.
bn − an bn − x0 bn − an x0 − an bn − an
Trong ket lu n, ta có the giả định bat đȁng thức trên cho vô hạn chỉ so nk,k ≥ 1
và cho an thay the bỏi x0.
Lay nk → ∞ và sử dụng định nghĩa của hàm f tại x0, ta có
|
f (a)− f (b)
| ≤ | f ′
(x0)|.
| ≤
19
Viết đề tài giá sinh viên – ZALO:0973.287.149-TEAMLUANVAN.COM
.
.
.
. ≤ ⇒ | | | ··· | ≤
n
1
2
| f ′
(0)| = lim .
f (x) − f (0).
= lim
. f (x).
.
.sinx|.
Bài toán 1.9. Cho hàm so
f(x) = a1 sinx+a2 sin2x+ ···+an sinnx.
Giả sử | f (x)| ≤ |sinx|. Chứng minh rang
|a1 + 2a2 + ··· + nan| ≤ 1.
L i giai. Vì f (0) = 0 và | f (x)| ≤ |sinx|, nên ta có
x→0 x
.
x→0
.
sinx
. .
x
.
f (x)
= lim 1, f′(0) = a +2a + +na 1.
x→0 sinx
Bài toán 1.10. Cho a là so thục dương. Chứng minh rang ex
> xa
⇔ a < e vói x > 0.
LỜi giai. Xét hàm f
ex
(x) =
xa
,x > 0.
Ta có f(x) → ∞ khi x → 0 và x → ∞. Do đó ton tại giá trị cục tieu của f tại x0. Theo
định lí Fermat ta có
f ′(x0) = ex
x−a 1 − a
= 0.
0 0
x0
Giá trị này nhỏ nhat tại x = a và f
ea
(a) =
aa
. Do v y
ex
> xa
⇔ a < e,x > 0.
Bài toán 1.11. Cho f là m t hàm có đạo hàm cap 3 liên tục trên đoạn [0;1], thỏa
mãn
f (0) = f ′(0) = f ′′(0) = f ′(1) = f ′′(1) = 0 và f (1) = 1.
Chứng minh rang ton tại c ∈ [0;1] sao cho f ′′′(c) ≥ 24.
L i giai. Hãy xem xét mỏ r ng chuői Taylor tại các điem x = 0 và x = 1 :
20
Viết đề tài giá sinh viên – ZALO:0973.287.149-TEAMLUANVAN.COM
−
−
f (x) = f (0) + f ′(0)x +
f ′′
(0)
x2
+
f ′′′
(ξx)
x3
,
2 6
f (x) = f (1) + f ′(1)(x − 1) +
f ′′
(1)
(x − 1)2
+
f ′′′
(ηx)
(x − 1)3
2 6
vói ≤ ξx ≤ x;x ≤ ηx ≤ 1.
Ket hop vói các giả thiet, ta có
f (x) =
f ′′′
(ξx)
x3
,
6
f (x) = 1 +
f ′′′
(ηx)
(x 1)3
.
6
Chon x =
1
ta thay ton tại ξ;η sao cho f ′′′(ξ)+ f ′′′(η) = 48.
2
Như v y, m t trong 2 so f ′′′(ξ), f ′′′(η) lón hơn ho c bang 24 .
V y ton tại c ∈ [0;1] đe f ′′′(c) ≥ 24.
Bài toán 1.12. Cho hàm so f khả vi trên [0;1] sao cho f (0) = 0, f (1) = 1. Chứng
minh ton tại 2 so phân bi t a,b thu c (0;1) sao cho f′(a). f′(b) = 1.
L i giai. Xét hàm so g(x) = f (x)+x−1 thì g khả vi trên [0;1]. Ta có g(0) = −1 < 0
và g(1) = 1 > 0 nên ton tại so c ∈ (0;1) sao cho g(c) = 0.
Do đó f (c) +c − 1 = 0 hay f (c) = 1− c.
Áp dụng định lý Lagrange cho f trên các đoạn [0;c] và [c;1] thì:
+ Ton tại a ∈ (0;c) sao cho
f(c)− f(0)
= f ′(a).
+ Ton tại b ∈ (c;1) sao cho
c 0
f(1)− f (c)
1 − c
= f ′(b), nên
f ′
(a). f ′
(b) =
f (c)
.
1 − f (c)
=
(1 − c)c
= 1.
c 1− c c(1− c)
V y ton tại 2 so phân bi t a,b thu c (0;1) sao cho f′(a). f′(b) = 1.
Bài toán 1.13. Cho hàm f : R → R là m t hàm trên C3
. Chứng minh rang ton tại
a ∈ R sao cho f (a) f ′(a) f ′′(a) f ′′′(a) ≥ 0.
21
Viết đề tài giá sinh viên – ZALO:0973.287.149-TEAMLUANVAN.COM
∀
L i giai. Neu m t trong các giá trị f (a), f ′(a), f ′′(a); f ′′′(a) bang 0 ta có luôn đieu
phải chứng minh.
Ta giả sử các giá trị f(x), f′(x), f′′(x), f′′′(x) đeu âm hay dương vói moi x.
Giả sử f′′(x) > 0, f(x) > 0, (neu f′′(x) < 0 ta có the thay f(x) thành −f (x), neu
f(x) < 0 ta có the thay the f(x) bang f(−x).
Từ f ′′(x) > 0 ta có f′(x) là hàm tăng, m t khác f ′′′(x) > 0 đieu đó cho thay f ′(x)
là m t hàm loi.
Khi đó ta có
f ′(x + a) > f ′(x) + a f ′′(x), ∀x ∈ R.
Cho x co định lay a đủ lón, ta sẽ có f ′(x) > 0,∀x ∈ R.
Vói l p lu n tương tụ ta sẽ có f ′(x) > 0 và f ′′(x) > 0 khi f (x) > 0, ∀x ∈ R.
Do v y ton tại a ∈ R sao cho f (a) f′(a) f ′′(a) f′′′(a) ≥ 0.
Bài toán 1.14. Cho hàm so f,g thỏa mãn đieu ki n
f′
(x)
g′(x)
= ef(x)−g(x), ∀x và
f(0) = g(2018) = 1. Tìm hang so c lón nhat sao cho f(2018) > c.
L i giai. Ta có
f′
(x)
= ef (x)−g(x), x hay f ′(x).e− f (x) = g′(x).e−g(x). Từ đó ta có
g′(s)
e− f (x) = e−g(x), ∀x ∈ R.
Xét hàm so
h(x) := e− f (x) − g′(x).e−g(x) = a
vói a là m t hang so. Khi đó ta có h(0) = h(2018), ket hop vói giả thiet ta đưoc
e−1
− e−g(0)
= e− f (2018)
− e−1
.
Do v y
e− f (2018)
= 2e−1
− e−g(0)
< 2e−1
= e−1+ln 2
.
V y ta chon hang so c lón nhat là c = −1 + ln 2 thỏa mãn đieu ki n f (2018) > c.
Bài toán 1.15. Chứng minh rang ab
+ ba
> 1,∀a > 0,b > 0
22
Viết đề tài giá sinh viên – ZALO:0973.287.149-TEAMLUANVAN.COM
( − ) = − < − .
L i giai.
- Bat đȁng thức trên là hien nhiên neu ít nhat các so a ho c b lón hơn 1.
- Ta đi xét các giá trị a,b ∈ (0;1).
Th t v y, đ t a = 1 − c,b = 1 − d;c,d ∈ (0;1). Ta xét hàm so sau f (x) = (a −
cx)d
,x ∈ [0;1]. Áp dụng định lí Lagrange ton tại x0 ∈ (0;1) sao cho:
1 c d
1
cd
1 cd
(1 − cx0)1−d
Từ đó
ab
+ ba
=
1 − c
+
1− d
>
1 − c
+
1− d
=
(1 − c)(1 − d)
+ 1 > 1.
(1− c)d (1 − d)c 1− cd 1−cd 1−cd
Bài toán 1.16. Cho f : R → R là m t hàm của lóp C3
sao cho f, f′, f′′, f′′′ đeu là
những so dương. Hơn nữa ta giả sử
f′′′(x) ≤ f(x), ∀x ∈ R.
Chứng minh rang f ′(x) < 2 f (x), ∀x ∈ R.
L i giai. Ta nh n thay, từ giả thiet ta có
lim
x→−∞
f ′(x) = lim
x→−∞
f′′(x) = 0.
Th t v y, theo định lí giá trị Lagrange, ton tại ξn ∈ (−n2
;−n) sao cho
f (−n2
) − f (−n) = (n − n2
) f ′(ξn) < 0.
Bang giả thiet giói hạn của f và f ′ tại −∞ ton tại và hơn nữa các giói hạn đó không
âm.
Ta giả sử phản chứng lim
x
f ′(x) > 0. Đe có giói hạn tại −∞ từ trên ta có đưoc
0 = −∞
→−∞
(mâu thuȁn). Do v y ta có lim
x→−∞
f′(x) = 0.
23
Viết đề tài giá sinh viên – ZALO:0973.287.149-TEAMLUANVAN.COM
µ λ
x>0
λ(1 − λ)π
2
1 −
a
≤ sup.
sin(ax)
−
sin(bx)
. ≤ 4 1 −
a
, ∀0 < a < b.
L p lu n tương tụ, ta cũng có đưoc
lim
x→−∞
f′′(x) = 0.
V y theo bo đe, ta có đưoc f ′(x) < 2 f (x),∀x.
Bài toán 1.17. Cho f : [0;1] → R là m t hàm liên tục trên [0;1] khả vi trên khoảng
(0; 1) và thỏa mãn 0 ≤ f ′(x) ≤ 2 f (x), ∀x ∈ (a; b). Chứng minh rang f đong nhat
bang 0.
L i giai. Xét g := [0;1] → R là m t hàm xác định bỏi g(x) = e−2x
f (x). Khi đó
g′(x) = e−2x
( f ′(x) − 2 f (x)) ≤ 0.
Do đó g là hàm giảm. M t khác, g(0) = 0,g ≥ 0, nên ta có g ≡ 0 hay f ≡ 0.
Bài toán 1.18. Chứng minh rang
π b x>0 ax bx b
L i giai.
Xét 0 < a < b. Đ t λ
a
=
b
, f (x) =
sin x
x
,x > 0 và ( ;x) =
f (x) − f (λx)
.
1− λ
Từ | f (x)| < 1,∀x > 0, ta có đưoc
2
|µ(λ;x)| <
1 − λ
≤ 4
vói đieu ki n 0 < λ < 1.
Đieu này chứng tỏ sup|µ(λ;x)| ≤ 4, ∀λ ∈ (0;1).
Đe chứng minh phan bat đȁng thức còn lại, ta nh n thay
µ(λ ;π) =
sin(λπ)
= µ(1 − λ, π).
24
Viết đề tài giá sinh viên – ZALO:0973.287.149-TEAMLUANVAN.COM
2
2
2
−
Từ đȁng thức đó, ta đi xem xét các trưòng hop ứng vói giá trị λ ∈ 0;
1
.
Vì f là hàm giảm trên
h
0;
π i
, ta có
1 sin
π
2
vói moi λ ∈ 0;
1i
.
µ(λ;π) ≥
λ π
2 ≥
π
2
Bài toán 1.19. Cho f là m t hàm thục khả vi cap n+1 trên [a;b], thỏa mãn f (i)(a) =
f (i)(b) = 0,∀i ∈ {1,2,3,...,n}. Chứng minh rang phương trình f (n+1)(x) = f (x) có
nghi m trên [a;b].
L i giai. Đau tiên ta giả sử n = 0 và xét hàm so g(x) = e−x
f (x). Theo định lí
Rolle, ton tại x0 ∈ [a;b] sao cho 0 = g′(x0) = e−x0 ( f ′(x0)− f (x0)). Đieu đó cho thay
f′(x0) = f (x0).
Tiep theo ta giả sử n ≥ 1 và xét hàm h(x) =
n
∑
k=0
f(k)(x). Khi đó h(a) = h(b) = 0
và h(x) = h′(x) = f (x)− f n+1
(x). Áp dụng cách chứng minh n = 0 đoi vói hàm h ta
suy ra đưoc rang f(n+1)(x) = f(x) có nghi m trên [a;b].
Bài toán 1.20. Cho f là m t hàm khả vi và f′ liên tục trên [a;b] sao cho ton tại giá
trị m ∈ [a;b] đe f ′(m) = 0. Chứng minh rang phương trình f ′(x) =
f (x) − f (a)
có
nghi m trên [a;b].
L i giai. Xét hàm g : [a;b] → R vói
g(x) = f
′
(x)
f (x) − f (a)
.
b − a
b − a
Không mat tính tong quát ta giả sử f(m) > f(a). Xét x1 ∈ [a;m] sao cho f(x1) =
max{ f (x);x ∈ [a;m]}.
Khi đó ta có f(x1)− f(a) > 0 ≥ (b−a) f′(x1).
Đieu đó cho thay g(x1) < 0.
25
Viết đề tài giá sinh viên – ZALO:0973.287.149-TEAMLUANVAN.COM
∈
2 1
2
Theo định lí Lagrange, ton tại x (a;x ) sao cho f ′(x ) =
f (x1)− f (a)
. Vì the
x1 − a
g(x2) =
f(x1 − f(a)
−
f(x2)− f(a)
.
x1 − a b − a
M t khác, do g là m t hàm liên tục, nên ton tại x ∈ [x2;x1] sao cho g(x) = 0. Do v y
phương trình f′(x) =
f(x)− f(a)
có nghi m trên [a;b].
b − a
Bài toán 1.21. Chứng minh rang không ton tại hàm dương liên tục, khả vi f trên
[0;∞) sao cho f′(x) ≥ f( f(x)) vói moi x ≥ 0.
L i giai. Giả sử f là m t hàm thỏa mãn đieu ki n bài toán.
Từ f′(x) ≥ f( f(x)) > 0 nên f là m t hàm tăng. Đieu đó cho thay
f ′(x) ≥ f ( f (x)) ≥ f ( f (0)) > 0.
Đieu này chứng tỏ f là m t hàm ch n dưói bỏi 0 và lim f(x) = ∞.
Vì the lim
x→∞ f(f (x)) = ∞ và lim
x→∞ f′(x) = ∞.
x→∞
Xét g(x) := f (x)−x−1. Do g′(x) dan tien ra ∞ khi x → ∞ nên g(x) → ∞ khi x → ∞.
Do đó ton tại x0 thỏa mãn f(x0) > x0 +1.
Tiep theo áp dụng định lí giá trị trung bình trên đoạn [x0; f(x0)], ta có đưoc m t điem
ξ ∈ (x0; f (x0)) thỏa mãn đieu ki n
f ( f (x0)) = f (x0) + f ′(ξ)( f (x0) − x0) > f ′(ξ)( f (x0) − x0),
≥ f ( f (ξ)( f (x0) − x0) > f ( f (x0))( f (x0) − x0) > f ( f (x0))
mâu thuȁn.
V y không ton tại hàm f thỏa mãn bài toán.
Bài toán 1.22. Cho hàm so f(x) khả vi trên đoạn [a;b] và thỏa mãn đieu ki n
| f (x)|2
+ | f ′(x)|2
> 0,∀x ∈ [a;b].
26
Viết đề tài giá sinh viên – ZALO:0973.287.149-TEAMLUANVAN.COM
Chứng minh rang các so các nghi m của phương trình f (x) = 0 trên đoạn [a; b] là
hữu hạn.
L i giai. Giả sử phương trình f(x) = 0 có vô so nghi m {xn} ⊂ [a;b] vói n = 1,2,...
Vì {xn} là dãy bị ch n nên ton tại dãy con {xnk } → α ∈ [a;b]. Do f (x) liên tục, nên
f(α) = 0. Từ giả thiet và vì f(α) = 0, nên f′(α) /= 0. Nhưng
f ′(α) = lim
f (x) − f (α)
/= 0.
x→α x−α
Đieu này chứng tỏ f(x) 0 trong m t lân c n nào đó của điem α. Đieu này lại mâu
thuȁn vói giả thiet α là điem tụ của dãy {xn}.
V y so các nghi m của phương trình f(x) = 0 trên đoạn [a;b] là hữu hạn.
27
Viết đề tài giá sinh viên – ZALO:0973.287.149-TEAMLUANVAN.COM
Chương 2
Bat đang th c và c c trị trong lỚp hàm mũ
và hàm hyperbolic
2.1 Bat đang th c trong lỚp hàm mũ và hàm hyperbolic
Trong phan này ta sử dụng ký hi u I đe chỉ m t trong các khoảng sau: [a,b], [a,b),
(a,b], (a,b) vói a < b.
Trưóc het, ta xét m t so bat đȁng thức cơ bản đoi vói hàm so.
Định lí 2.1. Giả sử f(x) là m t hàm loi trên khoảng I và λ1,λ2,λ3,...,λn là dãy các
so không âm thỏa mãn đieu ki n λ1 + λ2 + λ3 +···+ λn = 1.
Khi đó
f (λ1x1 + λ2x2 + · ·· + λnxn)
≤λ1 f (x1) + λ2 f (x2) + ··· + λn f (xn), ∀x1,x2,...,xn ∈ I.
H qua 2.1. Neu f (x) là m t hàm loi trên khoảng I thì
f
x1 + x2 + .... + xn
≤
f (x1) + f (x2) + .... + f (xn)
,
n n
∀x1,x2, .......,xn ∈ I.
28
Viết đề tài giá sinh viên – ZALO:0973.287.149-TEAMLUANVAN.COM
/
1
x
Định nghĩa 2.1. Vói các so dương a,b ta định nghĩa trung bình logarit của các so
a,b là bieu thức
Nh n xét rang
LM(a,b) =
b− a
, a = b, L(a,a) = a.
ln b − ln a
LM(a,b) =
1
=
1
.
1
∫ b dx
M
1
, (a,b)
Ký hi u: Ve sau, ta sử dụng các ký hi u đoi vói các đại lưong trung bình cho hai
so dương như sau
- Trung bình đieu hòa (HM(a,b)),
- Trung bình nhân (GM(a,b)),
- Trung bình logarit (LM(a,b)),
- Trung bình c ng (AM(a,b)).
Ta có
Định lí 2.2. Vói các so dương a < b ta luôn có
2ab
<
√
ab <
1
<
a+b
a+b 1
∫ b dx 2
hay
b− a a x
HM(a,b) < GM(a, b) < LM(a, b) < AM(a, b). (2.1)
ChÝng minh. Giả sử 0 < a < b xét hàm loi f (x) = ex
trên trên đoạn [ln a,ln b]. Ta
có bat đȁng thức
ln a+ln b
e 2 (ln b− ln a) <
ln b
ex
dx
ln a
ln a + ln b
2
(ln b − ln a).
x
a
b − a
∫
<
29
Viết đề tài giá sinh viên – ZALO:0973.287.149-TEAMLUANVAN.COM
⇔
a1/3 +b1/3
!3
) =
8
( )+
8 3
)+
8 3
Từ đây suy ra
√
ab <
b− a
<
a + b
⇔ GM(a,b) < LM(a,b) < AM(a,b).
Ta có
ln b−ln a 2
V y nên
2ab
<
√
ab HM(a, b) < GM(a, b).
a+b
HM(a,b) < GM(a,b) < LM(a,b) < AM(a,b), a,b > 0,a /= b.
Định lí 2.3. Vói 0 < a < b ta có các bat đȁng thức
LM(a,b) <
2
ChÝng minh. Ta có
, LM(a3
,b3
) < AM3
(a,b). (2.2)
∫ d
f x dx
1
f c d − c
3 2c+d
f d − c
3 c+2d
f d − c
trong đó c < η < d.
1
f(c)(d
8 —c) −
(c − d)5
f (4)
6480
Cho c = ln a,d = ln b, f (x) = ex
, ta thu đưoc
2ln a+ln b ln a+2ln b
b− a =
ln b
ex
dx
ln a
eln a
+ 3e 3 +3e
8
3 + eln b
.(ln b− ln a)
a1/3 +b1/3
!3
Từ đây suy ra bat đȁng thức thứ nhat trong (2.2).
Trong bat đȁng thức thứ nhat, thay a,b tương ứng bỏi a3
,b3
ta thu đưoc bat đȁng
∫
c
( )( ( ( )+
=
2
(ln b−ln a).
(η),
<
30
Viết đề tài giá sinh viên – ZALO:0973.287.149-TEAMLUANVAN.COM
2 2
⇔
thức thứ hai.
Bài toán 2.1. Ta có h thức
L i giai. Ta có
LM(x2
, y2
)
LM(x,y)
= AM(x,y). (2.3)
L x2
y2 x2
− y2
(x−y)(x+y) L x y x+y L x y A x y
( , ) =
ln x2 − ln y2
=
2(ln x − ln y)
= ( , )
2
= ( , ) ( , ).
Từ đây suy ra đieu phải chứng minh.
Bài toán 2.2. Giả sử b > a > 0. Khi đó ta luôn có bat đȁng thức
LM(a,b) <
AM(a,b) + GM(a,b)
. (2.4)
2
L i giai. Ta có
a1/3 +b1/3
!3
a1/2 +b1/2
!2
≤
√ √
b 2
⇔ 3(
√
3
a2b+
√
3
a2b) ≤ a+b+4
√
ab. (2.5)
Th t v y, áp dụng bat đȁng thức AM-GM cho ba so dương ta có
3
√
3
a2b = 3
q
3
3
√
3
ab2 = 3
q
3
C ng các ve tương ứng, ta đưoc
a.
√
ab.
√
ab ≤ a +
√
ab +
√
ab.
b.
√
ab.
√
ab ≤ b +
√
ab +
√
ab.
3(
√
3
a2b+
√
3
ab2) ≤ a+b+4
√
ab.
2
2
<
31
Viết đề tài giá sinh viên – ZALO:0973.287.149-TEAMLUANVAN.COM
2
2
=
2
.
ln b − ln a
< 2
M t khác
a1/2 +b1/2
!2 √
a+
√
b 2 a+b
+
√
ab AM + GM
Suy ra LM <
AM + GM
, đieu phải chứng minh.
2
Bài toán 2.3. Chứng minh bat đȁng thức
1 <
sinhx
< coshx, x > 0.
x
L i giai. Ta có
AM(ex
,e−x
) = cosh x, GM(ex
, e−x
) = 1, LM(ex
, e−x
) =
sinh x
.
x
Vói x =
/ 0, ex
/= e−x
. Áp dụng bat đȁng thức GM < LM < AM, ta có đieu phải chứng
minh.
Bài toán 2.4. Chứng minh bat đȁng thức
L i giai. Ta có
sinh x
x
<
cosh x + 1
2
, x > 0.
A(ex
,e−x
) = coshx, G(ex
,e−x
) = 1, L(ex
, e−x
) =
sinhx
.
x
Vói x > 0, ex
=
/
phải chứng minh.
e−x
ta áp dụng bat đȁng thức LM <
AM + GM
, ta thu đưoc đieu
2
Bài toán 2.5. Cho b > a > 0, chứng minh bat đȁng thức
b2
− a2
b+ a 2
2
2
2
= =
.
32
Viết đề tài giá sinh viên – ZALO:0973.287.149-TEAMLUANVAN.COM
−
b a b a
2 2
b − a
) =
b− a a
(− ) = −
b − a aa − )
ln −
—1 = −ln
−
aa e aa
L i giai. Theo đȁng thức (2.3) và bat đȁng thức (2.1), ta có
LM(a2
,b2
) = LM(a,b)AM(a,b) < AM2
(a,b).
Từ đó suy ra bat đȁng thức can phải chứng minh.
Tiep theo, ta xét m t loại trung bình nữa, đó là trung bình đong nhat (Identric) và
các bat đȁng thức gan vói hàm mũ liên quan.
Định nghĩa 2.2. Vói các so dương a,b trung bình đong nhat đưoc các đinh bỏi h
thức
IM(a,b) = a neu a = b,
1 bb (b
1
a)
e aa
neu a b.
Bài toán 2.6. Vói 0 < a < b, chứng minh bat đȁng thức
GM(a,b) < IM(a,b) < AM(a, b). (2.6)
L i giai. Áp dụng bat đȁng thức đoi vói hàm loi:
f
a+b
≤
1
∫ b
f x dx ≤
f (a) + f (b)
2
vói f(x) = −ln x, ta có
( )
b−a a 2
f
a+b
= −ln
a+ b
= −ln AM(a,b),
f (a) + f (b)
=
−ln a −ln b
= −
ln ab
= −ln
√
ab = −ln GM(a,b),
2
1
∫ b
f x dx
2
1
∫ b
2
ln x dx
1 h
ln
bb
b a
i
, hbb i 1 , 1hbb i 1
a
( —(
= − = −ln IM(a,b).
33
Viết đề tài giá sinh viên – ZALO:0973.287.149-TEAMLUANVAN.COM
x
a
= ( )
b − a a
)
Như v y, ta có
−ln AM(a,b) < −ln IM(a,b) < −ln GM(a,b)
⇔ AM(a,b) > IM(a,b) > GM(a,b).
Đe chứng minh bat đȁng thức (2.10) chúng ta can các bo đe sau đây:
Bài toán 2.7. Vói 0 < a < b, chứng minh bat đȁng thức
ln x ≤ x− 1, x > 0. (2.7)
L i giai. Xét hàm so f (x) = x −ln x − 1 trên mien x > 0. Ta có f ′(x) = 1−
1
= 0 ⇔
x = 1. De thay f ′(x) đoi dau từ âm sang dương khi đi qua x = 1. Do đó x = 1 là điem
cục tieu duy nhat. Nhưng, f (1) = 0. v y ta có f (x) ≥ 0, ∀x > 0. Dau đȁng thức xảyra
khi và chỉ khi x = 1.
V y bat đȁng thức (2.7) đưoc chứng minh.
Bài toán 2.8. Giả sử f (x) là hàm khả tong dương trên khoảng (a, b), a > 0. Chứng
minh bat đȁng thức
Hf < Gf < Af , (2.8)
trong đó
A
1
∫ b
f x dx G e
1 ∫ b
ln f (t)dt
H b − a (2.9)
=
b− a a
( ) , f = b−a a , f =
b
.
dx/ f (x)
a
L i giai. Trong bat đȁng thức (2.7), lay
x =
(b − a) f (t)
,
∫ b
f (τ)dτ
ln x ln f t −ln
1
∫ b
f τ dτ
f
∫
( .
34
Viết đề tài giá sinh viên – ZALO:0973.287.149-TEAMLUANVAN.COM
−
a
.
Theo bat đȁng thức (2.7), ta có
ln f(t) — ln
1
∫ b
f (τ)
dτ ≤
(b− a) f (t)
− 1.
b− a a ∫ b
f (τ)dτ
Lay tích phân theo t trên (a,b) bieu thức trên, ta có
∫ b 1
∫ b (b−a)
∫ b
f (t)dt
a
ln f (t)dt − (b − a)ln
b a
f (τ)dτ ≤ ∫ b
—(b − a) = 0.
Từ đây suy ra ve phải của (2.8).
Áp dụng bat đȁng thức này đoi vói
1
f
thay cho f vói chú ý rang
ta nh n đưoc ve trái của (2.8).
ln
1
= ln f(t)
f(t)
Bài toán 2.9. Vói 0 < a < b, chứng minh bat đȁng thức
L i giai. Vói f(x
1
) =
x
,
LM(a, b) < IM(a,b) < AM(a, b). (2.10)
ta có
A a b 1
∫ b dx ln b − ln a 1
f ( , ) =
b−a a x
=
1
∫ b
b− a
=
L(a, b)
,
Gf (a,b) = eb−a
a
(−ln x)x
= e−ln IM(a,b)
=
1
IM(a,b)
Suy ra LM(a, b) < IM(a,b).
Áp dụng bat đȁng thức Gf < Af vói f (x) = x ta có IM(a,b) < AM(a,b).
Như v y ta có
a f (τ)dτ
a
−
a
35
Viết đề tài giá sinh viên – ZALO:0973.287.149-TEAMLUANVAN.COM
2
2
H qua 2.2. Vói các so dương a,b có các bat đȁng thức kép
HM(a, b) ≤ GM(a, b) ≤ LM(a, b) ≤
AM(a, b) + GM(a, b)
≤ IM(a, b) ≤ AM(a, b).
(2.11)
ChÝng minh. Ta chỉ can chứng minh bat đȁng thức trung gian giữa LM và IM. Ta
có
LM(a,b) ≤
AM(a,b)+ GM(a,b)
≤ IM(a,b)
⇔ AM(a, b) + GM(a, b) ≤ 2IM(a, b) ≤ 2AM(a, b) ⇔ GM(a,b) ≤ AM(a, b).
Trong thục tien, có rat nhieu bài toán không liên quan trục tiep đen các hàm mũ
và hàm hyperbolic, nhưng chúng có những moi quan h m t thiet vói nhau, có the sử
dụng các hàm hyperbolic như là phương pháp giải hi u quả.
Bài toán 2.10. Cho a,b,c > 0 thỏa mãn đieu ki n
a+
√
a2 −1 b+
√
b2 −1 c+
√
c2 −1 = 10.
Tìm giá trị nhỏ nhat của
P = 4a3
+4b3
+4b3
− 3a−3b−3c.
L i giai. Theo giả thiet, ta có a,b,c ≥ 1 nên có the đ t a = coshx,b = coshy,c =
coshz, x,y,z ≥ 0.
Khi đó ta tìm đưoc
x = ln a+
√
a2 −1 ;y = ln b+
√
b2 −1 ;z = ln c+
√
c2 −1 .
Do đó ta có the viet giả thiet dưói dạng x + y + z = ln 10.
Sử dụng tính chat 1.9, ta thu đưoc
P = cosh3x + cosh3y + cosh3z.
Ta lại thay (cosh(3x))′′ = 9.cosh(3x) > 0, ∀x nên cosh(3x) là hàm loi trên R.
36
Viết đề tài giá sinh viên – ZALO:0973.287.149-TEAMLUANVAN.COM
3
/
∀
GM LM
Áp dụng bat đȁng thức Jensen, ta đưoc
P = cosh3x + cosh3y + cosh3z ≥ 3cosh
3x + 3y + 3z
hay
P eln 10 + e−ln 10 303
≥ 3cosh(ln 10) = 3.
Dau đȁng thức xảy ra khi
2
=
200
.
a = b = c =
√
3
10+
√
3
10−1
2
.
Bài toán 2.11. Chứng minh bat đȁng thức
1 <
sinh x
< cosh x, x = 0.
x
L i giai. Ta có
A(ex
,e−x
) = cosh x, GM(ex
, e−x
) = 1, LM(ex
,e−x
) =
sinhx
.
x
Vói x =
/ 0, ex
/= e−x
, áp dụng bat đȁng thức GM < LM < AM, ta có ngay đieu phải
chứng minh.
Bài toán 2.12. Chứng minh bat đȁng thức
L i giai. Ta có
cosh x +
x
> 2, x 0.
sinhx
AM(ex
,e−x
) = cosh x, GM(ex
, e−x
) = 1, LM(ex
, e−x
) =
sinh x
.
x
Vói x 0, ex
e−x
. Áp dụng bat đȁng thức
AM − GM
≥
LM − GM
,
37
Viết đề tài giá sinh viên – ZALO:0973.287.149-TEAMLUANVAN.COM
≥ ∀ ∈
2n
ta thu đưoc đieu phải chứng minh.
Bài toán 2.13. Chứng minh bat đȁng thức
L i giai. Ta có
sinh x
x
<
coshx+3coshx/3
4
, ∀x 0. (2.12)
LM(ex
,e−x
) =
sinhx
, M1
(ex
, e−x
) =
cosh x + 3 cosh x/3
.
x /3
4
Sử dụng bat đȁng thức LM ≤ M1/3 ta đưoc bat đȁng thức (2.12).
Bài toán 2.14. Cho a < b. Chứng minh rang ton tại c ∈ (a;b) sao cho
2 2
a− c
< sinhc <
b − c
.
L i giai. Đau tiên ta thay neu a < 0 < b thì sinhx đoi dau trên (a;b) vì sinh0 = 0.
Tức là ton tại x1 < 0,x2 > 0 sao cho sinhx1 ≥ 0,sinh x2 ≤ 0. Khi đó, chỉ can chon
c = 0, ta đưoc đieu can chứng minh.
Bài toán 2.15. Cho a < b. Chứng minh rang ton tại c ∈ (a;b) sao cho
2 2
a− c
< coshc <
b− c
.
L i giai. Vì coshx < 0 không đoi dau trên (a;b). Ta giả sử phản chứng rang vói moi
x ∈ (a;b), ta có
coshx
2
, x (a;b).
b − x
Xét dãy xn = b−
b− a
, ∀n ∈ N. Ket hop vói định lí giá trị trung bình Lagrange,
ta thay ton tại giá trị c1 ∈ (a;x1) sao cho
sinhx1 − sinha = sinh
a+ b
− sinha = coshc1
b− a
≥
2
.
b −a
≥ 1.
2 2 b−c1 2
38
Viết đề tài giá sinh viên – ZALO:0973.287.149-TEAMLUANVAN.COM
≤ . − .
t t
Cứ tiep tục quá trình trên, ta xây dụng dãy (cn)n≥1 sao cho cn+1 ∈ (xn;xn+1) và
sinhxn 1 − sinhxn = coshcn 1(xn 1 − xn) ≥
2 b− a
≥ 1.
+ + +
b − cn+1 2n+1
Do đó sinhxn − sinha ≥ n, ∀n ∈ N∗
. Đieu này cho thay sinhx là hàm không bị
ch n trên (a;b) (mâu thȁn vói tính chat hàm so sinhx là hàm liên tục khả vi . Do đó
giả sử là sai. V y ton tại giá trị c ∈ (a;b) sao cho coshc = sinh′ c <
2
.
Chứng minh tương tụ, ta thay ton tại giá trị c ∈ (a;b) sao cho
2
a − c
< coshc.
b − c
V y ta có đieu phải chứng minh.
Bài toán 2.16. Giả sử |coshx −coshy| ≤ M|x − y|, ∀x,y ∈ (0,1). Chứng minh rang
|coshx| ≤ M, ∀x,y ∈ (0,1).
L i giai. Xét hàm so f (x) = coshx. Co định x ∈ (0;1) và xét t /= 0, sao cho x +t ∈
(0;1). Theo giả thiet, ta có
| f′
(x)| ≤ .
f (x +t) f (x)
.+ .
f (x+t) f (x)
− f ′
(x).
M +
f(x+t)− f(x)
f′
(x) .
t
Chuyen qua giói hạn khi t → 0, ta có bat đȁng thức | f ′(x)| ≤ M hay |coshx| ≤
M, ∀x,y ∈ (0,1).
Bài toán 2.17. Cho hàm so f (x) = e−x2
. Chứng minh rang
L i giai. Ta có
max
x∈R
f′′
(x)| ≥
e1/2
4
.
f ′(x) = −2xe−x2
|
39
Viết đề tài giá sinh viên – ZALO:0973.287.149-TEAMLUANVAN.COM
| | ≤ ∀ ∈
R
x∈R x∈R
|f ′(x)| ≤ f
2+ 2
=
√
2e1/2
, ∀x ∈ R. Dau đȁng thức xảy ra khi x =
2+ 2
.
x∈R x∈R
và
f′′(x) = e−x2
(4x2
−1).
Nh n xét rang | f (x)| ≤ 1, ∀x ∈ R. Dau đȁng thức xảy ra khi x = 0. Tương tụ,
f ′(x) f
1
, x . Dau đȁng thức xảy ra khi x
2
Áp dụng bat đȁng thức Landau, ta có
1
=
2
.
max|f′(x)| ≤ 2
q
max|f(x)|M2,
trong đó M2 = max
x∈R
f′′
(x)|. Suy ra M2 ≥
e1/2
4
, đieu phải chứng minh.
Bài toán 2.18. Cho hàm so f(x) = e−x2+2x
. Chứng minh rang
max| f ′′(x)| ≥
e
.
L i giai. Ta có
x∈R 2
f ′(x) = 2(1− x)e−x2+2x
và
f ′′(x) = 2e−x2+2x
(2x2
−4x+1).
Nh n xét rang
√|f(x)| ≤ 1, ∀x ∈ R. Dau đȁng thức xảy ra khi x = 1. Tư√
ơng tụ,
2 2
Áp dụng bat đȁng thức Landau, ta có
max|f′(x)| ≤ 2
q
max|f(x)|M2,
trong đó M2 = max| f ′′(x)|. Suy ra M2 ≥
e
, đieu phải chứng minh.
x∈R 2
|
40
Viết đề tài giá sinh viên – ZALO:0973.287.149-TEAMLUANVAN.COM
≥
3C
2
2
cosh −
= 1 ⇔ A = B = C =
2.1.2. Bat đang th c trong tam giác trong lỚp hàm hyperbolic
Bài toán 2.19. Chứng minh rang trong tam giác ABC ta luôn có
sinh 3A+sinh 3B + sinh 3C ≥ 3 sinhπ.
L i giai. Ta có the giải bài toán 1.1 theo hai cách như sau:
Cách 1: Bien đoi trục tiep dụa vào tính chat coshx ≥ 1. ∀x ∈ R
sinh 3A+sinh 3B + sinh 3C+sinh π
= 2sinh
3A + 3B
cosh
3A − 3B
+ 2sinh
3C + π
cosh
3C − π
≥ 2sinh
2
3A + 3B
2
+ 2sinh
2 2 2
3C+π
2
= 4sinh
3A + 3B + 3C + π
cosh
3A+ 3B − 3C − π
Suy ra
≥ 4sinh
4 4
3A + 3B + 3C + π
4
= 4sinhπ.
sinh3A+sinh 3B + sinh 3C ≥ 3sinh π.
Dau đȁng thức xảy ra khi và chỉ khi
cosh
3A−3B
= 1
cosh
3A+3B− 3C −π
= 1
Cách 2: Xét hàm so y = sinhx. Do (sinhx)′′ = sinhx > 0, ∀x > 0 nên hàm so
sinhx loi trên (0;+∞) nên
sinh3A+sinh 3B + sinh3C 3sinh
3A + 3B + 3C
3
⇔ sinh3A+sinh3B + sinh3C ≥ 3sinh π.
π π
3
.
4
41
Viết đề tài giá sinh viên – ZALO:0973.287.149-TEAMLUANVAN.COM
≥
≥
≥
cosh2
x
≥
2 2 2 2 3
Dau đȁng thức xảy ra khi và chỉ khi A = B = C =
π
.
3
Ta có the tong quát thành bài toán sau: Chứng minh rang trong tam giác ABC ta
luôn có
sinhA+sinh B + sinhC 3sinh
kπ
,k > 0.
3
Bài toán 2.20. Chứng minh rang trong tam giác ABC ta luôn có
coshkA + coshkB + coshkC 3coshk
π
,k > 0.
3
L i giai. Xét hàm so y = coshx Do (coshx)′′ = coshx ≥ 1, ∀x ∈ R coshx loi trên
R nên
coshkA + cosh kA + cosh kA ≥ 3 cosh k
A + B +C
3
π
⇔ coshkA + coshkA + coshkA ≥ 3coshk
3
.
Dau đȁng thức xảy ra khi và chỉ khi A = B = C =
π
.
3
Bài toán 2.21. Chứng minh rang trong tam giác ABC ta luôn có
tanhkA + tanhkA + tanhkA 3tanhk
π
,k > 0.
3
L i giai. Xét hàm so y = tanhx. Do (tanhx)′′ =
2tanhx
≥ 0,∀x > 0 suy ra y = tanhx
loi trên (0;+∞) nên
tanhkA + tanhkA + tanhkA 3tanhk
A + B+C
, k > 0
3
π
⇔ tanhkA + tanhkA + tanhkA ≥ 3tanhk
3
.
Dau đȁng thức xảy ra khi và chỉ khi A = B = C =
π
.
3
Bài toán 2.22. Chứng minh rang trong tam giác ABC ta luôn có
sinh2 A
+ sinh2 B
+ sinh2C
≥
3
cosh
π
− 1 .
42
Viết đề tài giá sinh viên – ZALO:0973.287.149-TEAMLUANVAN.COM
≥
≥
2 2 2 2 3
cosh2 A
+cosh2 B
+cosh2C
≥
3
cosh
π
+1 .
L i giai. Ta có công thức:
sinh2 x
=
coshx−1
.
2 2
Suy ra
sinh2 A
+ sinh2 B
+ sinh2C
=
coshA − 1
+
coshB − 1
+
coshC − 1
2 2 2 2 2 2
cosh A + coshB + coshC 3
=
2
−
2
.
Áp dụng bat đȁng thức Jensen, ta đưoc
coshA + coshB + coshC 3cosh
π
.
3
Suy ra
sinh2 A
+ sinh2 B
+ sinh2C
≥
3
cosh
π
− 1 .
Bài toán 2.23. Chứng minh rang trong tam giác ABC ta luôn có
2 2
L i giai. Ta có công thức
2 2 3
cosh2 x
=
coshx + 1
2 2
nên
cosh2 A
+ cosh2 B
+ cosh2C
=
coshA + 1
+
cosh B + 1
+
coshC + 1
2 2 2 2 2 2
cosh A + coshB + coshC 3
=
2
+
2
.
Áp dụng bat đȁng thức Jensen, ta đưoc
coshA + coshB + coshC 3cosh
π
.
3
Suy ra
43
Viết đề tài giá sinh viên – ZALO:0973.287.149-TEAMLUANVAN.COM
2 2 2 2 3
1
2 2
cosh2 A
+cosh2 B
+cosh2C
≥
3
cosh
π
+1 .
Bài toán 2.24. Chứng minh rang trong tam giác ABC ta luôn có
sinhA+sinhB + sinhC ≥ sinh
π − A
+ sinh
π − B
+ sinh
π −C
.
2 2 2
L i giai. Ta có
sinhA+sinhB = 2sinh
A + B
cosh
A − B
≥ 2sinh
π −C
,
2 2 2
sinhB + sinhC = 2sinh
B +C
cosh
B −C
≥ 2sinh
π − A
,
2 2 2
sinhC+sinhA = 2sinh
C + A
cosh
C − A
≥ 2sinh
π − B
.
2 2 2
Do đó
sinhA+sinhB + sinhC ≥ sinh
π − A
+ sinh
π − B
+ sinh
π −C
.
2 2 2
Dau đȁng thức xảy ra khi và chỉ khi tam giác ABC đeu.
Bài toán 2.25. Chứng minh rang trong tam giác ABC ta luôn có
sinh
A
+ sinh
B
+ sinh
C
≥ tanh
A
+ tanh
B
+ tanh
C
.
2 2 2 2 2 2
L i giai. Xét hàm so y = sinh
x
− tanh
x
, ∀x ≥ 0.
Do y′
1
cosh
x 1 1
nên y′′
1
sinh
x sinh
x
2 0 x 0
=
2 2
−
2 cosh2 x ,
2
=
4 2
+
2 cosh3 x ≥
2
, ∀ ≥ .
Suy ra y(x)
x
≥ y(0)+y′(0).(x−0), ∀x ≥ 0.
Do đó sinh
2
x
—tanh
2
≥ 0, ∀x ≥ 0.
Thay x lan lưot bỏi A, B, C vào bat đȁng thức roi c ng theo ve, ta đưoc đieu phải
44
Viết đề tài giá sinh viên – ZALO:0973.287.149-TEAMLUANVAN.COM
≤
A B C
s .
Bài toán 2.26. Chứng minh rang trong tam giác ABC ta luôn có
3(sinhA + sinh B + sinhC) ≥ π
sinhA
+
sinhB
+
sinhC
.
A B C
L i giai. Xét hàm so y = x − tanhx. Ta có . . . ?
Do đó hàm so y = x − tanhx là hàm so đong bien trên R nên y ≥ y(0), ∀x ≥ 0.
Suy ra x− tanhx ≥ 0,∀x ≥ 0 nên x ≥ tanhx,∀x ≥ 0. Xét hàm so y=
sinhx
. Ta có
Do đó hàm so y=
x
sinhx
là hàm so đong bien (0;+∞)
x
Giả sử A ≥ B ≥ C ta có
sinhA
A
≥
sinhB
B
≥
sinhC
C
.
Theo bat đȁng thức Chebyshev, ta có
(A + B +C)
sinhA
+
sinhB
+
sinhC
≤ 3(sinhA + sinhB + sinhC).
A B C
Từ đó suy ra
3(sinhA + sinh B + sinhC) ≥ π
sinhA
+
sinhB
+
sinhC
.
Dau “=” xảy ra khi và chỉ khi tam giác ABC là tam giác đeu.
Bài toán 2.27. Chứng minh rang trong tam giác ABC ta luôn có
sinhAsinh BsinhC sinh3 π
.
3
L i giai. Xét hàm so y= ln (sinhx), ∀x > 0. Ta có
chứng minh
inh
A
+ sinh
B
+ sinh
C
≥ tanh
A
+ tanh
B
+ tanh
C
2 2 2 2 2 2
45
Viết đề tài giá sinh viên – ZALO:0973.287.149-TEAMLUANVAN.COM
3
3
≤
≥
3
3
≤
3 3
Do đó hàm so y= ln (sinhx) là hàm so lõm trên (0;+∞) nên ta có bat đȁng thức
ln (sinhA)+ln (sinhB)+ln (sinhC) ≤ 3ln sinh
A+B+C
.
Suy ra
ln (sinhAsinhBsinhC) ≤ ln sinh3 π
.
Vì hàm so y= ln x là hàm so đong bien trên (0;+∞) nên ta có đieu phải chứng minh
sinhAsinh BsinhC sinh3 π
.
3
Dau đȁng thức xảy ra khi và chỉ khi tam giác ABC đeu.
Bài toán 2.28. Chứng minh rang trong tam giác ABC ta luôn có
coshAcosh BcoshC cosh3 π
.
3
L i giai. Xét hàm so y= ln (coshx), ∀x > 0. Ta có
Do đó hàm so y= ln (coshx) là hàm so loi trên (0;+∞) nên ta có bat đȁng thức
ln (coshA)+ln (coshB)+ln (coshC) ≤ 3ln cosh
A+B+C
.
Suy ra
ln (coshAcoshBcoshC) ≥ ln cosh3 π
.
Vì hàm so y= ln x là hàm so đong bien trên (0;+∞) nên ta có đieu phải chứng minh
coshAcosh BcoshC cosh3 π
.
3
Dau đȁng thức xảy ra khi và chỉ khi tam giác ABC đeu.
Bài toán 2.29. Chứng minh rang trong tam giác ABC ta luôn có
sinhA +sinhB+ sinhC − tanhA− tanhB− tanhC ≥ 3 sinh
π
− tanh
π
.
46
Viết đề tài giá sinh viên – ZALO:0973.287.149-TEAMLUANVAN.COM
cosh2
x
sinhA +sinhB+ sinhC − tanhA− tanhB− tanhC ≥ 3 sinh
π
− tanh
π
.
L i giai. Xét hàm so y = sinhx − tanhx, ∀x > 0. Ta có y′ = coshx −
1
,y′′ =
sinhx +
sinhx
cosh3
x > 0, ∀x > 0. Do đó hàm so y = sinhx − tanhx là hàm so loi trên
(0;+∞) nên ta có bat đȁng thức
(sinhA − tanh A) + (sinh B − tanh B) + (sinhC − tanhC)
3
≥ sinh
A+B+C
− tanh
A+ B+C
.
3 3
Suy ra
3 3
Dau đȁng thức xảy ra khi và chỉ khi tam giác ABC đeu.
Bài toán 2.30. Chứng minh rang trong tam giác ABC ta luôn có
sinh2 A
+sinh
2
L i giai. Ta có công thức
2 B
+ sinh
2
2C
>
2
A2
+ B2
+C2
4
.
sinh2 x
=
coshx− 1
2 2
nên bat đȁng thức can chứng minh tương đương vói
coshA+ coshB + coshC − 3 ≥
A2
+ B2
+C2
2
.
Xét hàm so y = 2coshx−x2
, ∀x ≥ 0. Ta có y′ = 2sinhx−2x,y′′ = 2coshx−1 ≥
0, ∀x ≥ 0. Suy ra y(x) > y(0)+y′(0).(x−0), ∀x > 0 nên 2coshx−x2
> 2, ∀x > 0.
Lan lưot thay x bỏi A, B, C vào bat đȁng thức trên ta đưoc roi c ng từng ve các
bat đȁng thức đó, ta đưoc
2coshA − A2
+ 2coshB− B2
+ 2coshC −C2
> 6.
47
Viết đề tài giá sinh viên – ZALO:0973.287.149-TEAMLUANVAN.COM
Suy ra
coshA + coshB +coshC − 3 >
A2
+ B2
+C2
2
.
Bài toán 2.31. Chứng minh bat đȁng thức
ln
sinhx
>
1 xcoshx −sinhx
, x > 0 (2.13)
x 2 sinhx
L i giai. Đe chứng minh bat đȁng thức (2.13) có the sử dụng bat đȁng thức của các
đại lưong trung bình. Ta có
LM(ex
,e−x
) =
sinhx
, GM(ex
,e−x
) = 1, IM(ex
,e−x
) = excothx−1
. (2.14)
x
Lay logarit hai vé của (2.14), ta có bat đȁng thức (2.13).
2.2 Các dạng toán c c trị sinh bƠi hàm mũ và hyperbolic
2.2.1. Các dạng toán c c trị cơ ban
Trong mục này trình bày m t so kien thức cơ bản tìm cục trị hàm so.
Định nghĩa 2.3 (xem [1]). Giả sử hàm so f xác định trên t p hop D (D ⊂ R) và
x0 ∈ R.
a) x0 đưoc goi là m t điem cục đại của hàm so f neu ton tại m t khoảng (a; b)
chứa điem x0 sao cho (a; b) ⊂ D và f (x) < f (x0) vói moi x ∈ (a; b) {x0}. Khi đó
f(x0) đưoc goi là giá trị cục đại của hàm so f.
b) x0 đưoc goi là m t điem cục tieu của hàm so f neu ton tại m t khoảng (a; b)
chứa điem x0 sao cho(a;b) ⊂ D và f (x) > f (x0) vói moi x ∈ (a;b)  {x0}. Khi đó
f (x0) đưoc goi là giá trị cục tieu của hàm so f . Điem cục đại và điem cục tieu goi
chung là điem cục trị.
Giá trị cục đại và giá trị cục tieu đưoc goi chung là cục trị.
Định lí 2.4 (Đieu ki n can đe hàm so đạt cục trị, xem [1]). Giả sử hàm so f đạt cục
trị tại điem x0 . Khi đó, neu f có đạo hàm tại x0 thì f′(x0) = 0.
48
Viết đề tài giá sinh viên – ZALO:0973.287.149-TEAMLUANVAN.COM
3
≤
3
x
Định lí 2.5 (Đieu ki n đủ đe hàm so đạt cục trị, xem [1]). Giả sử hàm so f liên tục
trên khoảng (a;b)chứa điem x0 và có đạo hàm trên khoảng (a;x0) và (x0;b). Khi đó
a) Neu f′(x0) < 0 vói moi x ∈ (a;x0) và f ′(x0) > 0 vói moi x ∈ (x0;b) thì hàm so
đạt cục tieu tại điem x0.
b) Neu f′(x0) > 0 vói moi x ∈ (a;x0) và f′(x0) < 0 vói moi x ∈ (x0;b) thì hàm so
đạt cục đại tại điem x0.
Định lí 2.6 (xem [1]). Giả sử hàm so f có đạo hàm cap m t trên khoảng (a;b) chứa
điem x0, f′(x0) = 0 và f có đạo hàm cap hai khác 0 tại điem x0.
a) Neu f′′(x0) < 0 thì hàm so f đạt cục đại tại điem x0.
b) Neu f′′(x0) > 0 thì hàm so f đạt cục tieu tại điem x0.
Bài toán 2.32. Tìm giá trị nhỏ nhat của
M =
coshx+2
−
√
3
coshx, x /= 0.
L i giai. Ta có
AM(ex
,e−x
) = cosh x, GM(ex
,e−x
) = 1. (2.15)
Sử dụng bat đȁng thức
√
3
AG2
AM +2GM
3
và (2.15) de dàng suy ra bat đȁng thức
√
3
coshx ≤
coshx+2
, ∀x ∈ R. (2.16)
Từ đó suy ra M ≥ 0 và minM = 0 khi x = 0.
Bài toán 2.33. Tìm giá trị nhỏ nhat của
L i giai.
sinhx 2t
M =
tanhx t
, x /= 0.t > 0.
x
+
49
Viết đề tài giá sinh viên – ZALO:0973.287.149-TEAMLUANVAN.COM
Vói x /= 0, ta có
LM(ex
;e−x
) =
sinhx
, GM(ex
;e−x
) = 1, AM(ex
;e−x
) = coshx,
LM tanhx
= .
x
Từ đây, ta có chứng minh bat đȁng thức
AM x
(2.17)
sinhx 2t
tanhx t
> 2, x ∈ R.t > 0.
Dau đȁng thức không xảy ra xảy ra nên không có giá trị nhỏ nhat.
Bài toán 2.34. Tìm giá trị nhỏ nhat của bieu thức
M = excothx−1
,x ∈ R.
L i giai. Ta chứng minh bat đȁng thức
1 ≤ excothx−1
, x ∈ R.
Ta có
IM(ex
; e−x
) = excothx−1
,GM(ex
;e−x
) = 1, AM(ex
; e−x
) = coshx. (2.18)
V y nên ta có đieu phải chứng minh. Do đó minM = 1 khi x = 0.
Bài toán 2.35. Tìm giá trị nhỏ nhat của bieu thức
M = coshx − excothx−1
, x ∈ R.
L i giai. Ta chứng minh bat đȁng thức
excothx−1
≤ coshx, x ∈ R.
IM(ex
; e−x
) = excothx−1
,GM(ex
;e−x
) = 1, AM(ex
; e−x
) = cosh x. (2.19)
V y nên ta có đieu phải chứng minh. Do đó minM = 0 khi x = 0.
x
x
+
50
Viết đề tài giá sinh viên – ZALO:0973.287.149-TEAMLUANVAN.COM
Bài toán 2.36. Tìm giá trị nhỏ nhat của
M =
coshx + 3coshx/3
− e(xcothx−1)/2
<
sinhx
, x ∈ R.
4 x
L i giai. Ta chứng minh bat đȁng thức
e(xcothx−1)/2
<
sinhx
<
coshx + 3coshx/3
, x ∈ R.
x 4
Ta có
LM(ex
;e−x
) =
sinhx
, I(ex
;e−x
) = excothx−1
, (2.20)
x
GM(ex
;e−x
) = 1, M1/3 (ex
;e−x
) =
coshx+3coshx/3
4
(2.21)
Chú ý (2.19), áp dụng bat đȁng thức LM ≤ M1/3 đoi vói c p đoi so, ta có đieu
phải chứng minh.
V y nên minM = 0 khi x = 0.
Bài toán 2.37. Tìm giá trị lón nhat của
M = cosh2
x −2e2(xcothx−1), x ∈ R.
L i giai. Ta chứng minh bat đȁng thức
cosh2
x + 1 ≤ 2e2(xcothx−1), x ∈ R.
Ta có
AM(ex
,e−x
) = coshx, GM(ex
,e−x
) = 1, IM(ex
,e−x
) = excothx−1
.
Áp dụng bat đȁng thức
AM2
+ GM2
≤ 2IM2
51
Viết đề tài giá sinh viên – ZALO:0973.287.149-TEAMLUANVAN.COM
p
đoi vói c p đoi so, ta có ngay đieu phải chứng minh. Từ đó suy ra
M = cosh2
x − 2e2(xcothx−1) ≤ −1, x ∈ R.
Do đó minM = −1 khi x = 0.
Bài toán 2.38. Tìm giá trị lón nhat của
M = p(p + 1)sinh2x − 2sinh(p + 1)x + 2(p2
− 1)sinhx, p > −1, x ∈ R.
L i giai. Vói p = 0 ho c x = 0, de thay rang bat đȁng thức trỏ thành đȁng thức. Giả
sử p /= 0(p + 1 > 0) và x 0. Khi đó ta có 0 < e−x
< ex
và
AM(e−x
,ex
) = coshx, GM(e−x
,ex
) = 1, Lp
(e−x
,ex
) =
1 sinh(p + 1)x
.
Áp dụng bat đȁng thức
p
LMp
− GMp
(p + 1) sinh x
ta có
GMp−1
≥ p(AM − GM),
pcoshx ≤
1 sinh(p + 1)x
+ p − 1. (2.22)
Chú ý rang
(p + 1) sinh x
2sinhxcoshx = sinh2x, sinhaxsinhbx > 0, ∀x /= 0, ab > 0
từ (2.22) suy ra
p(p+1)sinh2x ≤ 2sinh(p+1)x +2(p2
−1)sinhx, ∀p > −1, ∀x ∈ R. (2.23)
V y maxM = 0 khi x = 0.
Bài toán 2.39. Xét phương trình x2
1
−ax−√
2
= 0 có các nghi m x1 và x2.
Tìm giá trị nhỏ nhat của bieu thức
52
Viết đề tài giá sinh viên – ZALO:0973.287.149-TEAMLUANVAN.COM
| |
+ x2 +x0 +
x0
+
x2
2 1 1 2
f (a) = (x1 −x2) + x1 −x2 +
x1
−
x2
.
1
L i giai. Vì x1.x2 = −√
2
< 0 nên có the coi x1 < 0,x2 > 0.
Đ t x0 = −x1 > 0. Khi đó
2 1 1 2
Sử dụng bat đȁng thức
1
x0
1
+
x2
4
≥
x0 + x2
vói x0 > 0,x2 > 0, ta đưoc
f (a) ≥ (x2 +x0)2
+
4 2
x0 + x2 +
(x0 + x2)
Suy ra
= 2(x2 + x0)2
+
16
(x0 + x2)2 + 8 ≥ 8
√
2 + 8.
x0 = x2
min f(a) = 8
√
2+8.
8 1 1 1
khi
0.
(x
0
+x2)2
=
(x0 + x2)2
⇔ x0 = x2 = √
4
2
⇔ x1 = −√
4
2
;x2 = √
4
2
⇔ a =
Bài toán 2.40. Cho hàm so f (x) = e−x2−x
. Goi M2 là giá trị lón nhat của | f ′′(x)|
trên trục thục, tức
Chứng minh rang M2 >
e1/2
4
.
M2 = max f′′(x) .
x∈R
L i giai. Ta có
và
f ′(x) = (−2x + 1)e−x2+x
f′′(x) = e−x2
(4x2
−1).
f (a) = (x2 +x0) .
53
Viết đề tài giá sinh viên – ZALO:0973.287.149-TEAMLUANVAN.COM
1
| | ≤ ∀ ∈
R
| |
| | ≤ ∀ ∈ √
x∈R x∈R
Tương tụ, ta có | f ′(x)| ≤ f
2 + 2
=
√
2e1/2
, ∀x ∈ R. Dau đȁng thức xảy ra
max|f′(x)| ≤ 2
q
max|f(x)|M2,
Nh n xét rang | f (x)| ≤ 1, ∀x ∈ R và dau đȁng thức xảy ra khi x = 0.
Tương tụ, ta có f′(x) f , x . Dau đȁng thức xảy ra khi x
2
Áp dụng bat đȁng thức Landau, ta có
max|f′(x)| ≤ 2
q
max|f(x)|M2,
1
=
2
.
trong đó M2 =
đưoc đpcm.
max
x∈R
f′′
(x)|. V y nên giá trị minM2 =
e1/2
4
không đạt đưoc, ta thu
Bài toán 2.41. Cho hàm so f(x) = e−x2+2x
. Goi M2 là giá trị lón nhat của |f′′(x)|
trên trục thục, tức
Chứng minh rang M2 >
e
.
2
M2 = max f′′(x) .
x∈R
L i giai. Ta có
và
f ′(x) = 2(1 − x)e−x2+2x
f ′′(x) = 2e−x2+2x
(2x2
−4x+1).
Nh n xét rang f (x) 1, x R và dau đȁng thức xảy ra khi x = 1.
khi x =
2+
√
2
2
2
.
Áp dụng bat đȁng thức Landau, ta có
x∈R
trong đó M2 = max| f ′′(x)|.
x∈R
x∈R
e
V y nên giá trị M2 =
2
không đạt đưoc, ta thu đưoc đpcm.
Bài toán 2.42. Xét tính đơn đi u của các hàm so sau:
|
54
Viết đề tài giá sinh viên – ZALO:0973.287.149-TEAMLUANVAN.COM
—∞
—1
—∞
+∞
ct
+∞
ln 2
x
a) f (x) = ln x − x; b) f (x) = e2x
− x.
L i giai.
a) T p xác định: D = (0;+∞).
Ta có f′(x) =
1
−1 =
1−x
; f′(x) = 0 ⇔ x = 1 ∈ D.
x x
Bảng bien thiên:
x 0 1 +∞
f ′(x) + 0 −
f (x)
−∞
−1
−∞
V y hàm so đong bien trên khoảng (0;1) và nghịch bien trên khoảng (1;+∞).
b) T p xác định: D = R. Ta có f ′(x) = 2e2x
− 1,
f ′(x) = 0 ⇔ e2x
=
1
⇔ x =
1
ln
1
= −ln
√
2.
2 2 2
Bảng bien thiên:
x −∞
−ln
√
2
+∞
f ′(x) − 0 +
f (x)
ct
V y hàm so nghịch bien trên khoảng (−∞;−ln
√
2) và đong bien trên khoảng
(−ln
√
2;+∞).
Bài toán 2.43. Tìm cục đại, cục tieu của hàm so f(x) =
x
.
ln x
L i giai. Đieu ki n: x > 0, x /= 1.
Ta có f ′(x) =
ln x − 1
≥ 0 ⇔ ln x ≥ 1, ⇔ x ≥ e.
Bảng bien thiên:
55
Viết đề tài giá sinh viên – ZALO:0973.287.149-TEAMLUANVAN.COM
+∞
e
+∞
—∞
loge
e
0
→
→
xln 10
2
=
e
x 1 e +∞
f ′(x) − 0 +
f (x)
e
V y hàm so chỉ có duy nhat m t cục tieu xct = e; yct = e.
Chú ý.
lim
x 1+
lim
x 1−
lim
x→0+
f (x) = +∞ vì tử so → 1, mȁu so → 0+,
f (x) = −∞ vì tử so → 1, mȁu so → 0−,f
(x) = 0 vì tử so → 0 , mȁu so → −∞.
Từ bảng bien thiên có the hỏi thêm:
1) Phương trình
x
lnxx = m Có hai nghi m khi nào ?; Đáp so : m > e
2) Phương trình
ln x
= m Vô nghi m khi nào ?; Đáp so : 0 ≤ m < e
Bài toán 2.44. Tìm cục đại, cục tieu của hàm so f(x) =
logx
trên R+
.
L i giai. Ta có f′ (x) =
x
1
− logx
x
loge− logx
x2
x
0 nên x ≤ e.
Bảng bien thiên:
x 0 e +∞
f ′(x) + 0 −
f (x)
−∞
loge
e
0
V y hàm so có duy nhat m t cục đại xcd = e; ycd =
loge
.
≥
56
Viết đề tài giá sinh viên – ZALO:0973.287.149-TEAMLUANVAN.COM
+∞
1
+∞
0
1
3e
0
x2 x x2
Nh n xét 2.1. Có the hỏi thêm các bài toán xác định max, min của hàm so và khảo
sát bat phương trình m >
logx
.
x
Bài toán 2.45. Tìm cục đại, cục tieu của hàm so f (x) = ex
.ln x, x ∈ R+
.
L i giai.
Ta có f ′(x) = ex
.
1
+ ln x.ex
= ex
.(
1
+ ln x) và ex
> 0, ∀x ∈ R+
.
1
x x
Xét h(x) =
x
+ ln x; x > 0, khi đó
h′(x) = −
1
+
1
=
x −1
= 0, suy ra x = 1.
Bảng bien thiên:
x 0 1 +∞
h′(x) − 0 +
h(x)
1
Từ bảng bien thiên suy ra h(x) ≥ 1. V y nên f (x) > 0 ∀x > 0, suy ra hàm so đã
cho không có cục trị.
Bài toán 2.46. Tìm cục trị của hàm so f(x) = x.e−3x
trên R.
L i giai. Ta có f′(x) = e−3x
+x.e−3x
.(−3) = e−3x
.(1−3x) = 0 nên x
Bảng bien thiên:
x −∞
1
e +∞
f ′(x) + 0 −
f (x)
0
1
3e
0
V y hàm so có duy nhat m t điem cục tieu xct =
1
; yct =
1
.
1
=
3
.
3 3e
57
Viết đề tài giá sinh viên – ZALO:0973.287.149-TEAMLUANVAN.COM
0
1
e2
0
1
0
( ) =
−
— ≤
— − ≤
Bài toán 2.48. Tìm cục trị của hàm so f(x) = |x|.e−|x−1| trên R.
Chú ý.
Đe tính lim
x→∓∞
f (x) = 0, ta sử dụng quy tac H’Lôpital.
Bài toán 2.47. Tìm cục trị của hàm so f x
ex
+e−x
2 trên R.
L i giai. Ta có f′ (x) =
ex e−x
2
= 0 nên ex
= e−x
, suy ra x = 0.
Cách 1: f′′
hàm so.
(x) =
ex +e−x
2
> 0, ∀x ∈ R nên x = 0 là hoành đ điem cục tieu của
Cách 2: L p bảng bien thiên.
KL: Hàm so có duy nhat m t điem cục tieu xct = 0; yct = 1.
Chú ý có the sử dụng quy tac 1, quy tac 2 đeu giải đưoc.
L i giai. Ta có f(x) =
Suy ra
x.ex−1
khi : x 0
x. ex−1
khi : 0 < x < 1
x. e1−x
khi : x ≥ 1
f
′
(x) =
1. ex−1
x.ex−1
khi : x 0
ex−1
+x.ex−1
khi : 0 < x < 1
e1−x
− x.e1−x
khi : x ≥ 1
ex−1
.(−x− 1) khi : x ≤ 0
=
Ta có bảng bien thiên:
ex−1
.(x+1) khi : 0 < x < 1
ex−1
.(1− x) khi : x ≥ 1
x −∞ −1 0 1 +∞
f ′(x) + 0 − ǁ + ǁ −
f (x)
0
1
e2
0
1
0
58
Viết đề tài giá sinh viên – ZALO:0973.287.149-TEAMLUANVAN.COM
e2
V y hàm so có hai điem cục đại: xcd = −1 nên ycd =
1
, xcd = 1 nên ycd = 1 và
hàm so có m t điem cục tieu: xct = 0 tức yct = 0.
Nh n xét 2.2. Từ bảng bien thiên ta có the đ t các câu hỏi liên quan mang n i dung
ứng dụng của các điem cục trị như sau.
1) Tìm a đe phương trình |x|.e−|x−1| = a có 4 nghi m phân bi t.
2) Tìm các đưòng ti m c n của hàm so tương ứng.
59
Viết đề tài giá sinh viên – ZALO:0973.287.149-TEAMLUANVAN.COM
⇔ ± − ∈
√
Chương 3
M t so dạng toán liên quan
3.1 Các phương trình đại so giai bang phương pháp hàm hy-
perbolic
Xét các phương trình cơ bản sau:
sinhx = a ⇔ x = ln (a+
√
a2 +1), a ∈ R,
coshx = a x = ln (a a2 1), a [1; +∞) ,
1 1+ a
tanhx = a ⇔ x = ln , a ∈ (−1;1),
2 1 − a
cothx = a ⇔ x =
1
ln
a+1
, a ∈ (−∞;−1) ∪(1;+∞).
2 a − 1
Nh n xét 3.1 (Phương trình b c 3).
a. Xét phương trình dạng
4x3
− 3x = q. (3.1)
Trưòng hop 1. Neu q < −1, thì đ t x = cost,t ∈ [0;π] , ta thu đưoc phương trình
cos3t = q ⇔ t = ±
1
arccos q +
k2π
.
3 3
Từ đó ta tìm đưoc 3 nghi m t1,t2,t3 ∈ [0;π] và x ∈ {cost1,cost2,cost3}.
Trưòng hop 2. Neu q > 1, sử dụng đạo hàm đe chứng minh phương trình có
60
Viết đề tài giá sinh viên – ZALO:0973.287.149-TEAMLUANVAN.COM
3
3
3
3
q √
q
−
√
nghi m duy nhat. Đ t q = cosh3t thì phương trình trỏ thành
4x3
− 3x = cosh3t ⇔ 4x3
−3x = 4cosh3
t − 3cosht.
Suy ra phương trình có nghi m x = cosht. Ta có
cosh3t = q ⇔ t =
1
ln (q±
√
q2 −1).
Suy ra
x = cosh
1
ln (q ±
√
q2 − 1)
x = cosh
1
ln (q +
√
q2 − 1)
⇔
x = cosh
1
ln (q−
√
q2 −1)
q
3
√
2 q
3
√ −1
⇔
x =
x =
q
3
q+ q
q−
√
q2
—1 +
—1 +
q
3 q+ q − 1
q−
√
q2 −1
⇔
x = 3
x =
q
3
q+ q2 1+ 3
q−
√
q2 −1+
q
3
q− q2 − 1
q+
√
q2 − 1
V y phương trình (3.1) có nghiêm duy nhat
x =
q
3
q+
√
q2 −1+
q
3
q−
√
q2 −1.
Trưòng hop 3. Neu q < −1 thì 4(−x)3
−3(−x) = −q.
Đ t y = (−x), ta thu đưoc phương trình 4y3
− 3y = −q. Đây chính là trưòng hop
đã xét ỏ trên.
b. Xét phương trình dạng
4x3
+ 3x = q. (3.2)
2
−1
61
Viết đề tài giá sinh viên – ZALO:0973.287.149-TEAMLUANVAN.COM
sinh3t = q ⇔ t =
1
ln (q±
√
q2 +1) ⇔ x = sinh
1
ln (q+
√
q2 + 1) .
a a a
Ta chứng minh phương trình (3.2) có nghi m duy nhat.
Th t v y, đ t x = sinht, ta đưoc phương trình
3 3
Từ đó, ta đưoc nghi m x =
q
3
q+
√
q2 +1+
q
3
q−
√
q2 +1.
c. Nh n xét rang phương trình x3
+ px = q luôn quy ve đưoc dạng (3.1) ho c (3.2)
bang cách đ t x = my;m2
= ±4p.
d. Xét phương trình
ax3
+ bx2
+ cx + d = 0, a 0.
Chia cả hai ve cho a, ta đưoc
x3
+
b
x2
+
c
x +
d
= 0, a /= 0.
Phương trình này luôn quy đưoc ve dạng (3.1) bang phép đ t y = x +
b
.
3a
Bài toán 3.1. Giải phương trình x3
− 3x = 10.
L i giai. Đ t x my ta đưoc m3
y3
3my 2015 chon
m3
4
m 2
= − =
3m
=
3
⇒ =
Thay vào ta đưoc 8y3
−6y = 10 ⇔ 4y3
−3y = 5
Áp dụng công thức nghi m của phương trình (3.1) ta đưoc
suy ra
y =
q
3
x = 2
q
3
5+
√
24+
q
3
5+
√
24+
q
3
5 −
√
24
5−
√
24 .
Bài toán 3.2. Giải phương trình x3
− 12x = −32.
62
Viết đề tài giá sinh viên – ZALO:0973.287.149-TEAMLUANVAN.COM
√
3
√
√
L i giai. Đ t x my ta đưoc m3
y3
12my 32 chon
m3
4
m 4
= − = − 12m
=
3
⇒ =
Thay vào ta đưoc 64y3
− 48y = −32 ⇔ 4y3
− 3y = −2 ⇔ 4(−y)3
− 3(−y) = 2.
Đ t z = −y, thì 4z3
− 3z = 2. Áp dụng công thức nghi m của phương trình (3.1), ta
đưoc
suy ra z =
q
3
2+
√
3+
q
3
2 −
√
3,
x = −4
q
3
2+
√
3+
q
3
2−
√
3 .
Bài toán 3.3. Giải phương trình x3
+ 5x = 1.
L i giai. Đ t x my ta đưoc m3
y3 5my 1 Chon m sao cho
m3
4
thì m
=
20
.
3
+ = .
5m
=
3
, =
Thay vào phương trình đã cho, ta thu đưoc
20
r
20
y3
+5
r
20
y = 1
3 3 3
3 3 3
⇔4y +3y =
5
√
20
.
Áp dụng công thức nghi m của phương trình (3.2) ta đưoc
Suy ra
y =
s
3
3
√
3+
√
527
5
√
20
+
3
√
3 −
√
527
.
5 20
x =
√
20
s
3
3
√
3+
√
527
5
√
20
+
3
√
3−
√
527
.
5 20
Bài toán 3.4. Giải phương trình x3
− 3x2
+ 4x+3 = 0.
r
s
3
s
3
63
Viết đề tài giá sinh viên – ZALO:0973.287.149-TEAMLUANVAN.COM
s
√
2
2
2 2
L i giai. Ta sử dụng các bien đoi như sau
x3
−3x2
+4x+3 = 0 ⇔ x3
−3x2
+3x−1+x+4 = 0 ⇔ (x−1)3
+(x−1) = −5.
Đăt y = x − 1 thì phương trình đã cho trỏ thành y3
+ y = −5.
3 3 m3
4 2
Đ t y = mz, ta đưoc m z + mz = −5. Chon m sao cho
m
=
3
hay m = √
3
.
Thay vào phương trình đã cho, ta thu đưoc
8
3
√
3
z
2
+ √
3
z = −5 ⇔ 4z +3z =
−15
√
3
.
2
Áp dụng công thức nghi m của phương trình (3.2), ta đưoc
hay
z =
s
3 −15
√
3 +
√
676
+
3
2
−15
√
3 −
√
676
2
2
x = √
3
s
3
3
√
3+
√
527
5
√
20
+ 3
√
3 −
√
527
5 20
+ 1.
Nh n xét 3.2. Ta nêu cách xây dụng các phương trình và các h quả liên quan:
sinhu = sinhv ⇔ 2cosh
u+v
sinh
u− v
= 0
2 2
⇔ sinh
u−v
⇔ u = v,
sinhu = −sinhv ⇔ 2sinh
u+v
cosh
u −v
= 0
2 2
⇔ sinh
u+v
⇔ u = −v,
coshu = coshv ⇔ 2sinh
u+v
sinh
u− v
= 0
3 3
s
3
64
Viết đề tài giá sinh viên – ZALO:0973.287.149-TEAMLUANVAN.COM
⇔
— ⇔
√
√
2
3 3
sinh3t = √
2
⇔ t =
3
ln √
2
+
⇔ sinh
u+v
⇔
u = −v
2
tanhu = tanhv
sinh(u − v)
cosh u cosh v
tanhu = tanhv
sinh(u + v)
cosh u cosh v
u = v
= 0 ⇔ sinh(u − v) ⇔ u = v,
= 0 ⇔ sinh(u + v) ⇔ u = −v,
a2
−b2
= 1 ⇒ ∃u : coshu = |a|;sinhu = b.
Bài toán 3.5. Giải phương trình 1+
√
1 +x2 = x 5− 2
√
1+ x2 .
L i giai. Đ t x = sinh2t phương trình trỏ thành
q
1 +
√
1 + sinh2
2t = x 5 − 2
√
1 + sinh2
2t
⇔
q
1 +
√
cosh2
2t = sinh2t 1 + 2
√
cosh2
2t
⇔
√
1 + cosh2t = sinh2t (1 + 2cosh2t)
⇔
√
2cosht = 2sinht cosht 1 + 2(1 + 2sin2
t)
⇔
√
2 = 2sinht 3 + 4sin2
t
1 1 1+
√
3
!
⇔ x = sinh
2
ln
3
1+
√
3
!!
1 1 √
3
!2
1+ √
3
!−2
⇔ x =
2
√
2
− √
2
.
65
Viết đề tài giá sinh viên – ZALO:0973.287.149-TEAMLUANVAN.COM
√
2
3 3
sinh3t = √
2
⇔ t =
3
ln √
2
+
Bài toán 3.6. Giải phương trình
√
1+
√
1+x2 = x 1+2
√
1+x2 .
L i giai. Đ t x = sinh2t phương trình trỏ thành
q
1 +
√
1 + sinh2
2t = x 1 + 2
√
1 + sinh2
2t
q
1+
√
cosh2
2t = sinh2t 1+2
√
cosh2
2t
⇔
√
1 + cosh2t = sinh2t (1 + 2cosh2t)
⇔
√
2cosht = 2sinht cosht 1 + 2(1 + 2sin2
t)
⇔
√
2 = 2sinht 3 + 4sin2
t
1 1 1+
√
3
!
⇔ x = sinh
2
ln
3
1+
√
3
!!
1 1 √
3
!2
1+ √
3
!−2
⇔ x =
2
√
2
− √
2
.
Bài toán 3.7. Giải phương trình 16x3
+ 7x = 3
√
1 + x2.
L i giai. Đ t x = sinht phương trình trỏ thành
16sinh3
t + 7sinht = 3
√
1 + sinh2
t
3
⇔ 16sinh t + 7sinht = 3cosht
⇔ 16sinh3
t + 12sinht = 5sinht + 3cosht
66
Viết đề tài giá sinh viên – ZALO:0973.287.149-TEAMLUANVAN.COM
√
− − −
√
− −
⇔ = ( + ) ⇔
3 5 3
⇔ 4sinh t + 3sinht =
4
sinht +
4
cosht. (∗)
Do
5
2
3
2
= 1 ⇒ ∃u :
coshu =
5
4
sinhu =
3
4
nên u = ln 2.
(∗) ⇔ sinh3t = coshusinht + sinhucosht
u
⇔ sinh3t = sinh(t + u) ⇔ t =
2
=
ln 2
2
.
Thay vào ta đưoc x = sinh
ln 2 1
=
2
√
2
.
Bài toán 3.8. Giải phương trình 32x4
− 32x2
− 5x + 4 = 3
√
x2 − 1.
L i giai. Đieu ki n
x ≥ 1
x ≤ −1
Neu x ≥ 1 thì đ t x = cosht phương trình trỏ thành
32cosh4
t 32cosh2
t 5cosht + 4 = 3 cosh2
t 1
⇔ 32cosh4
t − 32cosh2
t −5cosht + 4 = 3 sinh2
t
⇔ 4cosh4t = 5cosht +3|sinht|.
Nh n xét 3.3. Nh n xét rang neu phương trình có nghi m t thì nó cũng có nghi m
−t nên ta chỉ can xét t ≥ 0.
Phương trình trỏ thành
4cosh4t = 5cosht + 3sint ⇔ cosh4t =
5
cosht +
3
sint
4 4
cosh4t cosh t ln 2
4t = t ln 2
4t = t +ln 2
⇔
t =
−ln 2
5
t =
ln 2
3
⇔ t =
ln 2
3
.
2
4
4 −
67
Viết đề tài giá sinh viên – ZALO:0973.287.149-TEAMLUANVAN.COM
3
√
− −
√
Thay vào phương trình ban đau, ta đưoc x = cosh
ln 2
=
√
3
4+1
2
√
3
2
.
Neu x ≤ −1, đ t x = −cosht phương trình trỏ thành
32cosh4
t 32cosh2
t + 5cosht + 4 = 3 cosh2
t 1
⇔ 32cosh4
t − 32cosh2
t + 5cosht + 4 = 3 sinh2
t
⇔ 4cosh4t = −5cosht + 3|sinht|.
Nh n xét rang neu phương trình có t là nghi m thì −t cũng là nghi m, nên ta chỉ can
xét t ≥ 0.
Khi đó, phương trình trỏ thành
4cosh4t = −5cosht +3sint ⇔ cosh4t = −
5
cosht −
3
sint
4 4
⇔ cosh4t = −cosh(t − ln 2) ⇔ 0/.
3.2 Khao sát m t so lỚp phương trình ch a hàm mũ và hàm
hyperbolic
Bài toán 3.9. Giải phương trình
9x
(3x
+ 2x
) = 2x
(8x
+ 7x
) + 5x
(5x
− 2x
).
L i giai. Phương trình này chứa các hàm so mũ x, cơ so khác nhau. Ta sẽ dùng định
lí Lagrange sau khi đã đưa đưoc bieu thức trên ve dạng f(a) = f(b).
Ta có
9x
(3x
+ 2x
) = 2x
(8x
+ 7x
) + 5x
(5x
− 2x
) ⇔ 10x
− 16x
− 25x
= 14x
− 18x
− 27x
⇔ 10x
+ 12x
−16x
−25x
= 12x
+ 14x
−18x
−27x
. (∗)
Đen đây, ta xét hàm so f (t) = tx
+ (t + 2)x
− (t + 6)x
− (t + 15)x
, t > 0, trong đó
68
Viết đề tài giá sinh viên – ZALO:0973.287.149-TEAMLUANVAN.COM
−
0 0
x là nghi m của phương trình đã cho.
Do đó
(∗) ⇔ f(10) = f (12).
Hàm f (t) liên tục trên đoạn [10, 12] nên theo định lí Lagrange, ton tại c ∈ (10, 12)
sao cho f′(c) =
f(10)− f(12)
= 0. Do đó, neu x là nghi m của phương trình đã cho
10 12
thì nó phải thỏa mãn đieu ki n
x[cx−1
+(c+2)x−1
−(c+6)x−1
−(c+15)x−1
] = 0
⇔x = 0∨cx−1
+(c+2)x−1
= (c+6)x−1
+(c+15)x−1
.
Đȁng thức thứ hai cho ta x = 1 vì
• Neu x < 1 thì x−1 < 0 nên cx−1
+(c+2)x−1
> (c+6)x−1
+(c+15)x−1
.
• Neu x > 1 thì x−1 > 0 nên cx−1
+(c+2)x−1
< (c+6)x−1
+(c+15)x−1
.
Thử trục tiep, ta thay hai giá trị x = 0, x = 1 thỏa mãn bài ra nên phương trình đã cho
có hai nghi m là x = 0, x = 1.
Bài toán 3.10. Giải phương trình
ex
+(x3
−x)ln (x2
+1) = e
√
3 x
.
L i giai. Ta de thay phương trình này có ba nghi m là x = 0, x = ±1. Tiep theo, ta
sẽ chứng minh trong trưòng hop x /= 0, x /= ±1 thì phương trình đã cho vô nghi m
.
Th t v y, giả sử có so x0 /= 0, x0
Khi đó, rõ ràng là x0 /=
√
3 x0 và
±1 cũng là nghi m của phương trình đã cho.
ex0 +(x3
−x0)ln (x2
+1) = e
√
3 x0. (3.3)
Vì hàm so f(t) = et
liên tục nên ton tại m t giá trị c nam giữa hai so x0,
√
3 x0 sao
cho ex0 −e
√
3 x0
ec
=
x
√
x
.
0 − 3
0
69
Viết đề tài giá sinh viên – ZALO:0973.287.149-TEAMLUANVAN.COM
0 0
Nhưng từ (3.3), ta lại có
ex0 − e
√
3 x0 =
x0 −
√
3 x0
− x2
−x0
√
3 x0 +
q
3
x2
ln (x0
2
+1) < 0,
tức là ec
< 0, vô lí. Do đó, ta có đpcm. Phương trình đã cho chỉ có ba nghi m là
x = 0, x = ±1.
Bài toán 3.11. Cho a,b là các so thục vói a < b. Xét hàm f(x) = sinhx và g(x) =
coshx. Chứng minh rang các phương trình
sinha − sinhb = (a − b)coshx
và
cosha − coshb = (a − b)sinhx
đeu có ít nhat m t nghi m thu c (a;b).
L i giai. Áp dụng định lí giá trị trung bình Lagrange, ta thay neu f(x) khả vi trong
(a,b) và liên tục trong [a.b] thì ton tại x1 ∈ (a;b) sao cho
f(a)− f (b) = (a−b) f′(x1).
M t khác, vì (coshx)′ = sinhx và (sinhx)′ = coshx nên ta có the áp dụng định lí
giá trị trung bình Lagrange, thì ton tại x1,x2 ∈ (a;b) sao cho
sinha − sinhb = (a − b)coshx1
và
cosha − coshb = (a − b)sinhx2,
ta thu đưoc đieu can chứng minh.
70
Viết đề tài giá sinh viên – ZALO:0973.287.149-TEAMLUANVAN.COM
−
2
b − a
b a x0
2 b a x0
2
Bài toán 3.12. Cho 2 so thục 0 < a < b. Chứng minh rang phương trình
xcoshx sinhx =
asinhb − bsinha
b − a
có ít nhat m t nghi m thu c (a;b).
L i giai. Xét các hàm so g(x) =
sinhx
và h(x) =
1
trong (a,b). Ta có g′(x) =
xcoshx − sinhx
, h′
(x) =
x
x x
−1
.
x2
Theo định lý Cauchy thì ton tại x0 ∈ (a;b) sao cho
[h(b) − h(a)]g′(x0) = [g(b) − g(a)]h′(x0),
hay
1
−
1 x0 coshx0 −sinhx0
=
sinhb
−
sinha −1
.
Do đó
(a − b)(x0 coshx0 − sinhx0)
= −
a sinhb − bsinha
.
bax0
2 abx0
2
Suy ra x0 coshx0 − sinhx0 =
asinhb − bsinha
.
V y phương trình xcoshx − sinhx =
asinhb −bsinha
có ít nhat m t nghi m
thu c (a;b).
Bài toán 3.13. Cho a >
nghi m thục duy nhat.
b − a
0 chứng minh rang phương trình aex = 1 + x +
x
2
có m t
L i giai. Trưóc het, xét hàm so g(x) = e−x
f(x). Ta có
g′(x) = e−x
[ f ′(x) − f (x)] > 0.
Do v y g là hàm tăng nên g(x) > g(x0) = 0,∀
2
x > x0. V y f (x) > 0,∀x > x0.
Áp dụng vói hàm so f (x) = aex
−1−x −
x
. Khi đó lim f (x) = −∞ và lim f (x) =
2 x→+∞ x→+∞
+∞. Do v y f có ít nhat m t nghi m thục. Ta chứng minh f có nghi m duy nhat.
2
71
Viết đề tài giá sinh viên – ZALO:0973.287.149-TEAMLUANVAN.COM
1 và
axbx
— −
— −
Th t v y, ∀x ∈ R ta có
f ′(x) = aex
− 1− x > f (x).
Theo nh n xét trên thì phương trình có m t nghi m duy nhat.
Bài toán 3.14. Giải phương trình 4|x| + 2|x| = 4x + 2.
L i giai. Theo bat đȁng thức Bernoulli
tα
+ α −1 > αt ∀t > 1,α > 1
tα
+ α −1 < αt ∀t > 1,0 < α < 1
suy ra 2x
≥ x+1, 4x
≥ 3x +1 khi x ≥
2x
≤ x+1
4x
≤ 3x+1
khi x ∈ [0;1)
Khi x < 0 thì ve trái của phương trình nhỏ hơn ve phải của phương trình.
Đȁng thức xảy ra khi và chỉ khi x = 0 và x = 1, từ đó suy ra phương trình có
nghi m x = 0, x = 1.
Bài toán 3.15. Giải phương trình 41+x
+41−x
= 2x
+2−x
+31+x
+31−x
.
L i giai. Sử dụng bat đȁng thức a > b > 1 thì
ax
+ a−x
≥ bx
+ b−x
, ∀x ∈ R.
⇔ (ax
−bx
)+ a−x
− b−x
≥ 0
⇔ (ax
− bx
) 1 −
1
≥ 0.
Neu x > 0 thì (ax
bx
) > 0;1
1
(ab)
Neu x < 0 thì (ax
bx
) < 0;1
1
(ab)
x > 0.
x < 0.
Neu x = 0 thì ta có đȁng thức
72
Viết đề tài giá sinh viên – ZALO:0973.287.149-TEAMLUANVAN.COM
1
Áp dụng vào bài toán đã cho, ta có
4x
+ 4−x
≥ 2x
+ 2−x
.
4x
+ 4−x
≥ 3x
+ 3−x
.
Do v y 4[4x
+ 4−x
] ≥ 2x
+ 2−x
+ 3(3x
+ 3−x
). Dau = xảy ra khi và chỉ khi x = 0.
V y phương trình có nghi m duy nhat x = 0.
Bài toán 3.16 (Tuyen t p Olympic 30 tháng 4, lan XII - 2006). Giải phương trình
3x3+x+2 + x3 − 3x + 1 .32x−x3
= 34x+1.
L i giai. Ta có
3x3+x+2
+ x3
− 3x + 1 .32x−x3
= 34x+1
⇔ 3x3+x+2
.3x3−2x
+ x3
− 3x + 1 .32x−x3
.3x3−2x
= 34x+1
.3x3−2x
⇔ 32x3−x+2
+ x3
− 3x+ 1 = 3x3+2x+1
⇔ 32x3−x+2
+ 2x3
− x + 2 − x3
+2x + 1 = 3x3+2x+1
⇔ 32x3−x+2
+ 2x3
− x+ 2 = 3x3+2x+1
+ x3
+ 2x + 1 .
Đ t f (t) = 3t
+t,u = 2x3
− x + 2,v = x3
+ 2x + 1.
Phương trình trỏ thành
f (u) = f (v), f ′(t) = 3.ln 3+1 > 0, ∀t ∈ R nên f (t) đong bien trên R. V y nên
f (u) = f (v) ⇔ u = v.
V y nên 2x3
−x+2 = x3
+2x+1 ⇔ x3
−3x+1 = 0.
Đ t g(x) = x3
−3x+1. Ta có g(x) liên tục trên R và g(1) = −1 < 0,g(2) = 3 > 0.
Đ t x = 2cosα; α ∈ (0;π) thì phương trình x3
−3x+1 = 0 trỏ thành:
8cos3
α − 6cosα + 1 = 0 ⇔ 2.cos3α = −1
2π
⇔ cos3α = −
2
⇔ α = ±
9
+
k2π
3
.(k ∈ Z)
73
Viết đề tài giá sinh viên – ZALO:0973.287.149-TEAMLUANVAN.COM
2
V y phương trình đã cho có ba nghi m
x1 = 2cos
2π
;x2 = 2cos
4π
;x3 = 2cos
8π
.
9 9 9
Bài toán 3.17. Tìm nghi m nguyên dương a và b thỏa mãn phương trình ab
= ba
.
L i giai. Ta thay (a,b) = (n,n),n ∈ N∗
là nghi m. ln x
Xét trưòng hop a b. Giả sử a < b. Xét hàm so f(x) =
x
. Rõ ràng là ab
= ba
khi và chỉ khi f(a) = f(b).
Ta có f ′(x) =
1 − ln x
, do đó f x) tăng trên (0,e) và giảm trên (e,∞).
x
Như v y, đe f(a) = f(b) thì 0 < a < e,b > e. Ta có a ∈ {1,2},b ∈ {3,4, ......}.
Vói a = 1, bài toán không có lòi giải.
Vói a = 2, ta có b = 4. Rõ ràng là 24
= 42
.
Ket lu n: Phương trình đã cho có nghi m (a,b) = (n,n), n ∈ N∗
, (a,b) = (2,4)
và (a,b) = (4,2).
74
Viết đề tài giá sinh viên – ZALO:0973.287.149-TEAMLUANVAN.COM
Ket lu n
Lu n văn “M t so dạng toán cục trị trong lóp hàm mũ và hàm hyperbolic”
đã trình bày những van đe sau:
- Lu n văn trình bày chi tiet m t so đȁng thức và bat đȁng thức cơ bản liên
quan đen hàm mũ và hàm hyperbolic.
- Tiep theo trình bày chi tiet các dạng toán cục trị trong lóp hàm mũ và
hàm hyperbolic và nêu các phương pháp khảo sát chúng.
- Cuoi cùng, lu n văn trình bày các dạng toán liên quan đen phương trình,
bat phương trình và m t so bài toán chon loc từ các đe thi HSG quoc gia,
Olympic khu vục và quoc te.
75
Viết đề tài giá sinh viên – ZALO:0973.287.149-TEAMLUANVAN.COM
Tài li u tham khao
A Tieng Vi t
[1] Nguyen Văn M u 2006, Bat đȁng thŕc, định lí và áp dnng, NXB Giáo dục.
[2] Nguyen Văn M u (Chủ biên) 2010, So phŕc và áp dnng, NXB Giáo dục.
[3] Nguyen Văn M u, Phạm Thị Bạch Ngoc, 2003, M t so bài toán chon loc ve
lượng giác, NXB Giáo dục.
[4] Nguyen Văn M u, Lê Ngoc Lăng, Phạm the Long, Nguyen Minh Tuan (2006),
Các đe thi olympic Toán sinh viên toàn quoc, NXB Giáo dục.
[5] Tạ Duy Phưong, Hoàng Minh Quân (2017), Phương trình b c ba với các h
thŕc hình hoc và lượng giác trong tam giác, NXBGD Vi t Nam.
[6] Tạp chí TH&TT (2007), Các bài thi Olympic Toán trung hoc phő thông Vi t
Nam (1990-2006), NXB Giáo dục.
[7] Trương Đức Thịnh (2015), Đȁng thŕc và bat đȁng thŕc trong lớp hàm hyper-
bolic và áp dnng, Lu n văn Thạc sĩ, ĐH Thái Nguyên.
B Tieng Anh
[8] Paulo Ney de Sauza, Jorge- Nume Silva (1998), Berkeley Problems in Mathe-
matics, Springer.
[9] T-L.T. Radulescu, V.D. Radulescu, T.Andreescu (2009). Problems in Real
Analysis: Advanced Calculus on the real axis. Springer.

More Related Content

Similar to M T So Dạng Toán Cực Tr± Trong L P Hàm Mũ Và Hàm Hyperbolic.docx

Tich phan %28 nguyen duy khoi%29
Tich phan %28 nguyen duy khoi%29Tich phan %28 nguyen duy khoi%29
Tich phan %28 nguyen duy khoi%29
trongphuckhtn
 
Tich phan (nguyen duy khoi)
Tich phan (nguyen duy khoi)Tich phan (nguyen duy khoi)
Tich phan (nguyen duy khoi)
roggerbob
 
Phuong trinhluonggiackhongmaumuc[phongmath]
Phuong trinhluonggiackhongmaumuc[phongmath]Phuong trinhluonggiackhongmaumuc[phongmath]
Phuong trinhluonggiackhongmaumuc[phongmath]
phongmathbmt
 

Similar to M T So Dạng Toán Cực Tr± Trong L P Hàm Mũ Và Hàm Hyperbolic.docx (20)

Những điều cần biết luyện thi quốc gia kỹ thuật giải nhanh hệ phương trình tá...
Những điều cần biết luyện thi quốc gia kỹ thuật giải nhanh hệ phương trình tá...Những điều cần biết luyện thi quốc gia kỹ thuật giải nhanh hệ phương trình tá...
Những điều cần biết luyện thi quốc gia kỹ thuật giải nhanh hệ phương trình tá...
 
M T So Ứng Dụng Của Công Thức N I Suy Lagrange Và Hermite (2).docx
M T So Ứng Dụng Của Công Thức N I Suy Lagrange Và Hermite (2).docxM T So Ứng Dụng Của Công Thức N I Suy Lagrange Và Hermite (2).docx
M T So Ứng Dụng Của Công Thức N I Suy Lagrange Và Hermite (2).docx
 
Một Số Lớp Bất Đẳng Thức Lượng Giác Kiểu Klamkin Trong Tam Giác.doc
Một Số Lớp Bất Đẳng Thức Lượng Giác Kiểu Klamkin Trong Tam Giác.docMột Số Lớp Bất Đẳng Thức Lượng Giác Kiểu Klamkin Trong Tam Giác.doc
Một Số Lớp Bất Đẳng Thức Lượng Giác Kiểu Klamkin Trong Tam Giác.doc
 
Hàm Đơn Đi›U, Tựa Đơn Đi›U Và Một Số Ứng Dụng Của Phép Đơn Đi›U Hóa Hàm Số.docx
Hàm Đơn Đi›U, Tựa Đơn Đi›U Và Một Số Ứng Dụng Của Phép Đơn Đi›U Hóa Hàm Số.docxHàm Đơn Đi›U, Tựa Đơn Đi›U Và Một Số Ứng Dụng Của Phép Đơn Đi›U Hóa Hàm Số.docx
Hàm Đơn Đi›U, Tựa Đơn Đi›U Và Một Số Ứng Dụng Của Phép Đơn Đi›U Hóa Hàm Số.docx
 
Một số phương pháp tìm cực trị của các hàm phân thức Sinh bởi số tự nhiên.docx
Một số phương pháp tìm cực trị của các hàm phân thức Sinh bởi số tự nhiên.docxMột số phương pháp tìm cực trị của các hàm phân thức Sinh bởi số tự nhiên.docx
Một số phương pháp tìm cực trị của các hàm phân thức Sinh bởi số tự nhiên.docx
 
Một số phương pháp giải các đề thi olympic Về phương trình diophant.docx
Một số phương pháp giải các đề thi olympic Về phương trình diophant.docxMột số phương pháp giải các đề thi olympic Về phương trình diophant.docx
Một số phương pháp giải các đề thi olympic Về phương trình diophant.docx
 
Bat Đang Thức Và Bài Toán Cực Tr± Trong L P Các Đa Thức Và Phân Thức H So Ngu...
Bat Đang Thức Và Bài Toán Cực Tr± Trong L P Các Đa Thức Và Phân Thức H So Ngu...Bat Đang Thức Và Bài Toán Cực Tr± Trong L P Các Đa Thức Và Phân Thức H So Ngu...
Bat Đang Thức Và Bài Toán Cực Tr± Trong L P Các Đa Thức Và Phân Thức H So Ngu...
 
Tổng quát về tích phân
Tổng quát về tích phân Tổng quát về tích phân
Tổng quát về tích phân
 
Kĩ thuật tổng hợp Giải bất phương trình hỗn hợp.docx
Kĩ thuật tổng hợp Giải bất phương trình hỗn hợp.docxKĩ thuật tổng hợp Giải bất phương trình hỗn hợp.docx
Kĩ thuật tổng hợp Giải bất phương trình hỗn hợp.docx
 
Bat đang thức trong so hoc và m t so Dạng toán liên quan.docx
Bat đang thức trong so hoc và m t so Dạng toán liên quan.docxBat đang thức trong so hoc và m t so Dạng toán liên quan.docx
Bat đang thức trong so hoc và m t so Dạng toán liên quan.docx
 
Tich phan %28 nguyen duy khoi%29
Tich phan %28 nguyen duy khoi%29Tich phan %28 nguyen duy khoi%29
Tich phan %28 nguyen duy khoi%29
 
Tich phan (nguyen duy khoi)
Tich phan (nguyen duy khoi)Tich phan (nguyen duy khoi)
Tich phan (nguyen duy khoi)
 
Cẩm nang kết cấu xây dựng
Cẩm nang kết cấu xây dựngCẩm nang kết cấu xây dựng
Cẩm nang kết cấu xây dựng
 
Phương Trình Và Bất Phương Trình Hàm Trong Lớp Hàm Lượng Giác Ngược.doc
Phương Trình Và Bất Phương Trình Hàm Trong Lớp Hàm Lượng Giác Ngược.docPhương Trình Và Bất Phương Trình Hàm Trong Lớp Hàm Lượng Giác Ngược.doc
Phương Trình Và Bất Phương Trình Hàm Trong Lớp Hàm Lượng Giác Ngược.doc
 
Ứng Dụng Hình Học Giải Tích Vào Giải Phương Trình, Bất Phương Trình Và Hệ Phư...
Ứng Dụng Hình Học Giải Tích Vào Giải Phương Trình, Bất Phương Trình Và Hệ Phư...Ứng Dụng Hình Học Giải Tích Vào Giải Phương Trình, Bất Phương Trình Và Hệ Phư...
Ứng Dụng Hình Học Giải Tích Vào Giải Phương Trình, Bất Phương Trình Và Hệ Phư...
 
Phuong trinhluonggiackhongmaumuc[phongmath]
Phuong trinhluonggiackhongmaumuc[phongmath]Phuong trinhluonggiackhongmaumuc[phongmath]
Phuong trinhluonggiackhongmaumuc[phongmath]
 
Đa Thức Trong Các Bài Toán Thi Học Sinh Giỏi.docx
Đa Thức Trong Các Bài Toán Thi Học Sinh Giỏi.docxĐa Thức Trong Các Bài Toán Thi Học Sinh Giỏi.docx
Đa Thức Trong Các Bài Toán Thi Học Sinh Giỏi.docx
 
Toán Tử Sai Phân Và Ứng Dụng Vào Giải Toán Sơ Cấp.docx
Toán Tử Sai Phân Và Ứng Dụng Vào Giải Toán Sơ Cấp.docxToán Tử Sai Phân Và Ứng Dụng Vào Giải Toán Sơ Cấp.docx
Toán Tử Sai Phân Và Ứng Dụng Vào Giải Toán Sơ Cấp.docx
 
Luận Văn Các Nguyên Lý Biến Phân Thường Dùng Trong Cơ Học Công Trình.docx
Luận Văn Các Nguyên Lý Biến Phân Thường Dùng Trong Cơ Học Công Trình.docxLuận Văn Các Nguyên Lý Biến Phân Thường Dùng Trong Cơ Học Công Trình.docx
Luận Văn Các Nguyên Lý Biến Phân Thường Dùng Trong Cơ Học Công Trình.docx
 
Khảo sát nghi m của các phương trình sinh b i đạo hàm và nguyên hàm Của m t đ...
Khảo sát nghi m của các phương trình sinh b i đạo hàm và nguyên hàm Của m t đ...Khảo sát nghi m của các phương trình sinh b i đạo hàm và nguyên hàm Của m t đ...
Khảo sát nghi m của các phương trình sinh b i đạo hàm và nguyên hàm Của m t đ...
 

More from DV Viết Luận văn luanvanmaster.com ZALO 0973287149

More from DV Viết Luận văn luanvanmaster.com ZALO 0973287149 (20)

Ảnh Hưởng Của Marketing Quan Hệ Đến Lòng Trung Thành Của Khách Hàng.Tình Huốn...
Ảnh Hưởng Của Marketing Quan Hệ Đến Lòng Trung Thành Của Khách Hàng.Tình Huốn...Ảnh Hưởng Của Marketing Quan Hệ Đến Lòng Trung Thành Của Khách Hàng.Tình Huốn...
Ảnh Hưởng Của Marketing Quan Hệ Đến Lòng Trung Thành Của Khách Hàng.Tình Huốn...
 
Phát triển nguồn nhân lực tại Uỷ ban nhân dân huyện Trà Bồng, tỉnh Quảng Ngãi...
Phát triển nguồn nhân lực tại Uỷ ban nhân dân huyện Trà Bồng, tỉnh Quảng Ngãi...Phát triển nguồn nhân lực tại Uỷ ban nhân dân huyện Trà Bồng, tỉnh Quảng Ngãi...
Phát triển nguồn nhân lực tại Uỷ ban nhân dân huyện Trà Bồng, tỉnh Quảng Ngãi...
 
Báo cáo tốt Nghiệp tài chính hợp nhất tại tổng công ty Indochina gol...
Báo cáo tốt Nghiệp  tài chính hợp nhất tại tổng công ty Indochina gol...Báo cáo tốt Nghiệp  tài chính hợp nhất tại tổng công ty Indochina gol...
Báo cáo tốt Nghiệp tài chính hợp nhất tại tổng công ty Indochina gol...
 
Tạo động lực thúc đẩy nhân viên làm việc tại ngân hàng TMCP Ngoại Thương Việt...
Tạo động lực thúc đẩy nhân viên làm việc tại ngân hàng TMCP Ngoại Thương Việt...Tạo động lực thúc đẩy nhân viên làm việc tại ngân hàng TMCP Ngoại Thương Việt...
Tạo động lực thúc đẩy nhân viên làm việc tại ngân hàng TMCP Ngoại Thương Việt...
 
Phát triển công nghiệp trên địa bàn Thành phố Tam Kỳ, Tỉnh Quảng Na...
Phát triển công nghiệp trên địa bàn Thành phố Tam Kỳ, Tỉnh Quảng Na...Phát triển công nghiệp trên địa bàn Thành phố Tam Kỳ, Tỉnh Quảng Na...
Phát triển công nghiệp trên địa bàn Thành phố Tam Kỳ, Tỉnh Quảng Na...
 
Giải pháp phát triển cho vay xuất nhập khẩu tại ngân hàng NN&PTNN ch...
Giải pháp phát triển cho vay xuất nhập khẩu tại ngân hàng NN&PTNN ch...Giải pháp phát triển cho vay xuất nhập khẩu tại ngân hàng NN&PTNN ch...
Giải pháp phát triển cho vay xuất nhập khẩu tại ngân hàng NN&PTNN ch...
 
Hoàn thiện công tác lập báo cáo tài chính hợp nhất tại tổng công ...
Hoàn thiện công tác lập báo cáo tài chính hợp nhất tại tổng công ...Hoàn thiện công tác lập báo cáo tài chính hợp nhất tại tổng công ...
Hoàn thiện công tác lập báo cáo tài chính hợp nhất tại tổng công ...
 
Luận Văn Thạc Sĩ Quản trị thành tích nhân viên tại Cục Hải quan TP Đà Nẵng.doc
Luận Văn Thạc Sĩ  Quản trị thành tích nhân viên tại Cục Hải quan TP Đà Nẵng.docLuận Văn Thạc Sĩ  Quản trị thành tích nhân viên tại Cục Hải quan TP Đà Nẵng.doc
Luận Văn Thạc Sĩ Quản trị thành tích nhân viên tại Cục Hải quan TP Đà Nẵng.doc
 
Hoàn thiện công tác quản lý thuế thu nhập cá nhân tại cục thuế Tỉ...
Hoàn thiện công tác quản lý thuế thu nhập cá nhân tại cục thuế Tỉ...Hoàn thiện công tác quản lý thuế thu nhập cá nhân tại cục thuế Tỉ...
Hoàn thiện công tác quản lý thuế thu nhập cá nhân tại cục thuế Tỉ...
 
Đề Tài Phát triển bền vững nông nghiệp Huyện Ba Tơ, Tỉnh Quảng Ngãi....
Đề Tài Phát triển bền vững nông nghiệp Huyện Ba Tơ, Tỉnh Quảng Ngãi....Đề Tài Phát triển bền vững nông nghiệp Huyện Ba Tơ, Tỉnh Quảng Ngãi....
Đề Tài Phát triển bền vững nông nghiệp Huyện Ba Tơ, Tỉnh Quảng Ngãi....
 
Hoàn thiện công tác bảo trợ xã hội trên địa bàn huyện Phong Điền, tỉnh Thừa T...
Hoàn thiện công tác bảo trợ xã hội trên địa bàn huyện Phong Điền, tỉnh Thừa T...Hoàn thiện công tác bảo trợ xã hội trên địa bàn huyện Phong Điền, tỉnh Thừa T...
Hoàn thiện công tác bảo trợ xã hội trên địa bàn huyện Phong Điền, tỉnh Thừa T...
 
Đề Tài Luận VănPhát triển sản phẩm du lịch tại thành phố Đà Nẵng.doc
Đề Tài Luận VănPhát triển sản phẩm du lịch tại thành phố Đà Nẵng.docĐề Tài Luận VănPhát triển sản phẩm du lịch tại thành phố Đà Nẵng.doc
Đề Tài Luận VănPhát triển sản phẩm du lịch tại thành phố Đà Nẵng.doc
 
Đào tạo nghề cho lao động thuộc diện thu hồi đất trên địa bàn Thàn...
Đào tạo nghề cho lao động thuộc diện thu hồi đất trên địa bàn Thàn...Đào tạo nghề cho lao động thuộc diện thu hồi đất trên địa bàn Thàn...
Đào tạo nghề cho lao động thuộc diện thu hồi đất trên địa bàn Thàn...
 
Tóm Tắt Luận Văn Thạc Sĩ Quản Trị Kinh Doanh Xây dựng chính sách Marketing tạ...
Tóm Tắt Luận Văn Thạc Sĩ Quản Trị Kinh Doanh Xây dựng chính sách Marketing tạ...Tóm Tắt Luận Văn Thạc Sĩ Quản Trị Kinh Doanh Xây dựng chính sách Marketing tạ...
Tóm Tắt Luận Văn Thạc Sĩ Quản Trị Kinh Doanh Xây dựng chính sách Marketing tạ...
 
Đề Tài Nghiên cứu rủi ro cảm nhận đối với mua hàng thời trang trực tuyến.docx
Đề Tài Nghiên cứu rủi ro cảm nhận đối với mua hàng thời trang trực tuyến.docxĐề Tài Nghiên cứu rủi ro cảm nhận đối với mua hàng thời trang trực tuyến.docx
Đề Tài Nghiên cứu rủi ro cảm nhận đối với mua hàng thời trang trực tuyến.docx
 
Giải pháp nâng cao động lực thúc đẩy người lao động tại công ty khai...
Giải pháp nâng cao động lực thúc đẩy người lao động tại công ty khai...Giải pháp nâng cao động lực thúc đẩy người lao động tại công ty khai...
Giải pháp nâng cao động lực thúc đẩy người lao động tại công ty khai...
 
Giải pháp phát triển dịch vụ ngân hàng điện tử tại ngân hàng đầu ...
Giải pháp phát triển dịch vụ ngân hàng điện tử tại ngân hàng đầu ...Giải pháp phát triển dịch vụ ngân hàng điện tử tại ngân hàng đầu ...
Giải pháp phát triển dịch vụ ngân hàng điện tử tại ngân hàng đầu ...
 
Giải pháp phát triển dịch vụ ngân hàng điện tử tại ngân hàng đầu ...
Giải pháp phát triển dịch vụ ngân hàng điện tử tại ngân hàng đầu ...Giải pháp phát triển dịch vụ ngân hàng điện tử tại ngân hàng đầu ...
Giải pháp phát triển dịch vụ ngân hàng điện tử tại ngân hàng đầu ...
 
Quản trị quan hệ khách hàng tại Chi nhánh Viettel Đà Nẵng – Tập đoàn Viễn thô...
Quản trị quan hệ khách hàng tại Chi nhánh Viettel Đà Nẵng – Tập đoàn Viễn thô...Quản trị quan hệ khách hàng tại Chi nhánh Viettel Đà Nẵng – Tập đoàn Viễn thô...
Quản trị quan hệ khách hàng tại Chi nhánh Viettel Đà Nẵng – Tập đoàn Viễn thô...
 
Đề Tài Đánh giá thành tích đội ngũ giảng viên trường Đại Học Phạm ...
Đề Tài Đánh giá thành tích đội ngũ giảng viên trường Đại Học Phạm ...Đề Tài Đánh giá thành tích đội ngũ giảng viên trường Đại Học Phạm ...
Đề Tài Đánh giá thành tích đội ngũ giảng viên trường Đại Học Phạm ...
 

Recently uploaded

xemsomenh.com-Vòng Tràng Sinh - Cách An 12 Sao Và Ý Nghĩa Từng Sao.pdf
xemsomenh.com-Vòng Tràng Sinh - Cách An 12 Sao Và Ý Nghĩa Từng Sao.pdfxemsomenh.com-Vòng Tràng Sinh - Cách An 12 Sao Và Ý Nghĩa Từng Sao.pdf
xemsomenh.com-Vòng Tràng Sinh - Cách An 12 Sao Và Ý Nghĩa Từng Sao.pdf
Xem Số Mệnh
 
SLIDE - Tu van, huong dan cong tac tuyen sinh-2024 (đầy đủ chi tiết).pdf
SLIDE - Tu van, huong dan cong tac tuyen sinh-2024 (đầy đủ chi tiết).pdfSLIDE - Tu van, huong dan cong tac tuyen sinh-2024 (đầy đủ chi tiết).pdf
SLIDE - Tu van, huong dan cong tac tuyen sinh-2024 (đầy đủ chi tiết).pdf
hoangtuansinh1
 

Recently uploaded (20)

xemsomenh.com-Vòng Tràng Sinh - Cách An 12 Sao Và Ý Nghĩa Từng Sao.pdf
xemsomenh.com-Vòng Tràng Sinh - Cách An 12 Sao Và Ý Nghĩa Từng Sao.pdfxemsomenh.com-Vòng Tràng Sinh - Cách An 12 Sao Và Ý Nghĩa Từng Sao.pdf
xemsomenh.com-Vòng Tràng Sinh - Cách An 12 Sao Và Ý Nghĩa Từng Sao.pdf
 
SLIDE - Tu van, huong dan cong tac tuyen sinh-2024 (đầy đủ chi tiết).pdf
SLIDE - Tu van, huong dan cong tac tuyen sinh-2024 (đầy đủ chi tiết).pdfSLIDE - Tu van, huong dan cong tac tuyen sinh-2024 (đầy đủ chi tiết).pdf
SLIDE - Tu van, huong dan cong tac tuyen sinh-2024 (đầy đủ chi tiết).pdf
 
xemsomenh.com-Vòng Thái Tuế và Ý Nghĩa Các Sao Tại Cung Mệnh.pdf
xemsomenh.com-Vòng Thái Tuế và Ý Nghĩa Các Sao Tại Cung Mệnh.pdfxemsomenh.com-Vòng Thái Tuế và Ý Nghĩa Các Sao Tại Cung Mệnh.pdf
xemsomenh.com-Vòng Thái Tuế và Ý Nghĩa Các Sao Tại Cung Mệnh.pdf
 
powerpoint mẫu họp phụ huynh cuối kì 2 học sinh lớp 7 bgs
powerpoint mẫu họp phụ huynh cuối kì 2 học sinh lớp 7 bgspowerpoint mẫu họp phụ huynh cuối kì 2 học sinh lớp 7 bgs
powerpoint mẫu họp phụ huynh cuối kì 2 học sinh lớp 7 bgs
 
30 ĐỀ PHÁT TRIỂN THEO CẤU TRÚC ĐỀ MINH HỌA BGD NGÀY 22-3-2024 KỲ THI TỐT NGHI...
30 ĐỀ PHÁT TRIỂN THEO CẤU TRÚC ĐỀ MINH HỌA BGD NGÀY 22-3-2024 KỲ THI TỐT NGHI...30 ĐỀ PHÁT TRIỂN THEO CẤU TRÚC ĐỀ MINH HỌA BGD NGÀY 22-3-2024 KỲ THI TỐT NGHI...
30 ĐỀ PHÁT TRIỂN THEO CẤU TRÚC ĐỀ MINH HỌA BGD NGÀY 22-3-2024 KỲ THI TỐT NGHI...
 
Các điều kiện bảo hiểm trong bảo hiểm hàng hoá
Các điều kiện bảo hiểm trong bảo hiểm hàng hoáCác điều kiện bảo hiểm trong bảo hiểm hàng hoá
Các điều kiện bảo hiểm trong bảo hiểm hàng hoá
 
TÀI LIỆU BỒI DƯỠNG HỌC SINH GIỎI KỸ NĂNG VIẾT ĐOẠN VĂN NGHỊ LUẬN XÃ HỘI 200 C...
TÀI LIỆU BỒI DƯỠNG HỌC SINH GIỎI KỸ NĂNG VIẾT ĐOẠN VĂN NGHỊ LUẬN XÃ HỘI 200 C...TÀI LIỆU BỒI DƯỠNG HỌC SINH GIỎI KỸ NĂNG VIẾT ĐOẠN VĂN NGHỊ LUẬN XÃ HỘI 200 C...
TÀI LIỆU BỒI DƯỠNG HỌC SINH GIỎI KỸ NĂNG VIẾT ĐOẠN VĂN NGHỊ LUẬN XÃ HỘI 200 C...
 
GNHH và KBHQ - giao nhận hàng hoá và khai báo hải quan
GNHH và KBHQ - giao nhận hàng hoá và khai báo hải quanGNHH và KBHQ - giao nhận hàng hoá và khai báo hải quan
GNHH và KBHQ - giao nhận hàng hoá và khai báo hải quan
 
Đề cương môn giải phẫu......................
Đề cương môn giải phẫu......................Đề cương môn giải phẫu......................
Đề cương môn giải phẫu......................
 
sách sinh học đại cương - Textbook.pdf
sách sinh học đại cương   -   Textbook.pdfsách sinh học đại cương   -   Textbook.pdf
sách sinh học đại cương - Textbook.pdf
 
bài thi bảo vệ nền tảng tư tưởng của Đảng.docx
bài thi bảo vệ nền tảng tư tưởng của Đảng.docxbài thi bảo vệ nền tảng tư tưởng của Đảng.docx
bài thi bảo vệ nền tảng tư tưởng của Đảng.docx
 
GIÁO ÁN DẠY THÊM (KẾ HOẠCH BÀI DẠY BUỔI 2) - TIẾNG ANH 7 GLOBAL SUCCESS (2 CỘ...
GIÁO ÁN DẠY THÊM (KẾ HOẠCH BÀI DẠY BUỔI 2) - TIẾNG ANH 7 GLOBAL SUCCESS (2 CỘ...GIÁO ÁN DẠY THÊM (KẾ HOẠCH BÀI DẠY BUỔI 2) - TIẾNG ANH 7 GLOBAL SUCCESS (2 CỘ...
GIÁO ÁN DẠY THÊM (KẾ HOẠCH BÀI DẠY BUỔI 2) - TIẾNG ANH 7 GLOBAL SUCCESS (2 CỘ...
 
PHƯƠNG THỨC VẬN TẢI ĐƯỜNG SẮT TRONG VẬN TẢI
PHƯƠNG THỨC VẬN TẢI ĐƯỜNG SẮT TRONG VẬN TẢIPHƯƠNG THỨC VẬN TẢI ĐƯỜNG SẮT TRONG VẬN TẢI
PHƯƠNG THỨC VẬN TẢI ĐƯỜNG SẮT TRONG VẬN TẢI
 
TÀI LIỆU BỒI DƯỠNG HỌC SINH GIỎI LÝ LUẬN VĂN HỌC NĂM HỌC 2023-2024 - MÔN NGỮ ...
TÀI LIỆU BỒI DƯỠNG HỌC SINH GIỎI LÝ LUẬN VĂN HỌC NĂM HỌC 2023-2024 - MÔN NGỮ ...TÀI LIỆU BỒI DƯỠNG HỌC SINH GIỎI LÝ LUẬN VĂN HỌC NĂM HỌC 2023-2024 - MÔN NGỮ ...
TÀI LIỆU BỒI DƯỠNG HỌC SINH GIỎI LÝ LUẬN VĂN HỌC NĂM HỌC 2023-2024 - MÔN NGỮ ...
 
Access: Chuong III Thiet ke truy van Query.ppt
Access: Chuong III Thiet ke truy van Query.pptAccess: Chuong III Thiet ke truy van Query.ppt
Access: Chuong III Thiet ke truy van Query.ppt
 
1 - MÃ LỖI SỬA CHỮA BOARD MẠCH BẾP TỪ.pdf
1 - MÃ LỖI SỬA CHỮA BOARD MẠCH BẾP TỪ.pdf1 - MÃ LỖI SỬA CHỮA BOARD MẠCH BẾP TỪ.pdf
1 - MÃ LỖI SỬA CHỮA BOARD MẠCH BẾP TỪ.pdf
 
Trắc nghiệm CHƯƠNG 5 môn Chủ nghĩa xã hội
Trắc nghiệm CHƯƠNG 5 môn Chủ nghĩa xã hộiTrắc nghiệm CHƯƠNG 5 môn Chủ nghĩa xã hội
Trắc nghiệm CHƯƠNG 5 môn Chủ nghĩa xã hội
 
1.DOANNGOCPHUONGTHAO-APDUNGSTEMTHIETKEBTHHHGIUPHSHOCHIEUQUA (1).docx
1.DOANNGOCPHUONGTHAO-APDUNGSTEMTHIETKEBTHHHGIUPHSHOCHIEUQUA (1).docx1.DOANNGOCPHUONGTHAO-APDUNGSTEMTHIETKEBTHHHGIUPHSHOCHIEUQUA (1).docx
1.DOANNGOCPHUONGTHAO-APDUNGSTEMTHIETKEBTHHHGIUPHSHOCHIEUQUA (1).docx
 
SÁNG KIẾN ÁP DỤNG CLT (COMMUNICATIVE LANGUAGE TEACHING) VÀO QUÁ TRÌNH DẠY - H...
SÁNG KIẾN ÁP DỤNG CLT (COMMUNICATIVE LANGUAGE TEACHING) VÀO QUÁ TRÌNH DẠY - H...SÁNG KIẾN ÁP DỤNG CLT (COMMUNICATIVE LANGUAGE TEACHING) VÀO QUÁ TRÌNH DẠY - H...
SÁNG KIẾN ÁP DỤNG CLT (COMMUNICATIVE LANGUAGE TEACHING) VÀO QUÁ TRÌNH DẠY - H...
 
ĐỀ CHÍNH THỨC KỲ THI TUYỂN SINH VÀO LỚP 10 THPT CÁC TỈNH THÀNH NĂM HỌC 2020 –...
ĐỀ CHÍNH THỨC KỲ THI TUYỂN SINH VÀO LỚP 10 THPT CÁC TỈNH THÀNH NĂM HỌC 2020 –...ĐỀ CHÍNH THỨC KỲ THI TUYỂN SINH VÀO LỚP 10 THPT CÁC TỈNH THÀNH NĂM HỌC 2020 –...
ĐỀ CHÍNH THỨC KỲ THI TUYỂN SINH VÀO LỚP 10 THPT CÁC TỈNH THÀNH NĂM HỌC 2020 –...
 

M T So Dạng Toán Cực Tr± Trong L P Hàm Mũ Và Hàm Hyperbolic.docx

  • 1. ĐẠI HOC THÁI NGUYÊN TRƯ NG ĐẠI HOC KHOA HOC Tải tài liệu tại sividoc.com Viết đề tài giá sinh viên – ZALO:0973.287.149-TEAMLUANVAN.COM TRAN TH± HƯ NG M T SO DẠNG TOÁN CỰC TR± TRONG L P HÀM MŨ VÀ HÀM HYPERBOLIC LU N VĂN THẠC SĨ TOÁN HOC THÁI NGUYÊN - 2018
  • 2. ĐẠI HOC THÁI NGUYÊN TRƯ NG ĐẠI HOC KHOA HOC Tải tài liệu tại sividoc.com Viết đề tài giá sinh viên – ZALO:0973.287.149-TEAMLUANVAN.COM TRAN TH± HƯ NG M T SO DẠNG TOÁN CỰC TR± TRONG L P HÀM MŨ VÀ HÀM HYPERBOLIC Chuyên ngành: Phương pháp toán sơ cap Mã so: 84 60 113 LU N VĂN THẠC SĨ TOÁN HOC NGƯŐI HƯŐNG DAN KHOA HOC GS.TSKH. Nguyen Văn M u THÁI NGUYÊN - 2018
  • 3. i Viết đề tài giá sinh viên – ZALO:0973.287.149-TEAMLUANVAN.COM Mục lục M ĐAU ii 1 M t so kien th c liên quan đen các hàm mũ và hyperbolic 1 1.1 Tính chat cơ bản của các hàm mũ và hyperbolic . . . . . . . . . . . 1 1.1.1 Tính chat cơ bản của hàm mũ . . . . . . . . . . . . . . . . 1 1.1.2 Tính chat cơ bản của hàm hyperbolic . . . . . . . . . . . . 2 1.2 Đȁng thức sinh bỏi hàm mũ và hàm hyperbolic . . . . . . . . . . . 5 1.3 M t so bat đȁng thức chứa đạo hàm và tích phân quan trong .................. 10 2 Bat đang th c và c c trị trong lỚp hàm mũ và hàm hyperbolic 27 2.1 Bat đȁng thức trong lóp hàm mũ và hàm hyperbolic............................... 27 2.2 Các dạng toán cục trị sinh bỏi hàm mũ và hyperbolic............................. 47 3 M t so dạng toán liên quan 59 3.1 Các phương trình đại so giải bang phương pháp hàm hyperbolic ........... 59 3.2 Khảo sát m t so lóp phương trình chứa hàm mũ và hàm hyperbolic . 67 KET LU N 74 TÀI LI U THAM KHÂO 75
  • 4. ii Viết đề tài giá sinh viên – ZALO:0973.287.149-TEAMLUANVAN.COM M ĐAU Chuyên đe ve các hàm siêu vi t (hàm mũ và logarit) đưoc đe c p ỏ lóp 12 b c trung hoc pho thông. Vì v y các ứng dụng của hàm mũ và logarit không đưoc đe c p trong các lóp 10 và 11. Đ c bi t, do giảm tải chương trình, lóp các hàm hyperbobic cũng không đưoc đưa vào SGK. Các hàm này chỉ đưoc khảo sát trong chương trình boi dưõng HSG ỏ các lóp chuyên Toán phục vụ các kỳ thi HSG quoc gia, Olympic khu vục và quoc te. Trong các kì thi hoc sinh giỏi toán các cap b c THPT và Olympic khu vục và quoc te, các bài toán liên quan tói hàm mũ và hàm hyperbolic thưòng xuyên đưoc đe c p. Những dạng toán này thưòng đưoc xem là thu c loại khó vì phan kien thức sâu sac ve hàm mũ và hàm hyperbolic không nam trong chương trình chính thức của giáo trình Đại so và Giải tích b c trung hoc pho thông. Đe đáp ứng nhu cau boi dưõng giáo viên và boi dưõng hoc sinh giỏi ve chuyên đe hàm mũ và hàm hyperbolic, tôi chon đe tài lu n văn “M t so dạng toán cục trị trong lóp hàm mũ và hàm hyperbolic”. Lu n văn nham tong hop m t so tính chat của hàm mũ và hàm hyperbolic và moi quan h giữa chúng. Tiep theo, xét các bài toán cục trị, khảo sát m t so lóp phương trình, bat phương trình cùng m t so dạng toán đại so có sử dụng tính chat hàm mũ, hàm ngưoc của nó là hàm logarit và hàm hyperbolic. Cau trúc lu n văn gom 3 chương: Chương 1. M t so kien thức liên quan đen các hàm mũ và hyperbolic. Chương 2. Bat đȁng thức và cục trị trong lóp hàm mũ và hàm hyperbolic. Chương 3. M t so dạng toán liên quan. Lu n văn sử dụng m t so dạng toán và bài t p từ các tài li u [1]-[9] và m t so
  • 5. iii Viết đề tài giá sinh viên – ZALO:0973.287.149-TEAMLUANVAN.COM đe thi Olympic liên quan đen hàm hàm mũ và hàm hyperbolic trong những năm gan đây. Lu n văn đưoc hoàn thành vói sụ hưóng dȁn của GS.TSKH. Nguyen Văn M u. Em xin bày tỏ lòng biet ơn sâu sac đoi vói sụ quan tâm, đ ng viên và sụ chỉ bảo hưóng dȁn của thay. Tác giả xin chân thành cảm ơn Ban giám hi u, Phòng Đào tạo, Khoa Toán và các thay cô trưòng Đại hoc Khoa hoc - Đại hoc Thái Nguyên đã tạo đieu ki n thu n loi cho tác giả hoàn thành bản lu n này. Nhân dịp này, tác giả cũng xin chân thành cảm ơn Ban giám hi u và các đong nghi p trưòng THPT Nguyen Bỉnh Khiêm, huy n Vĩnh Bảo, thành pho Hải Phòng đã tạo đieu ki n cho tác giả hoàn thành tot nhi m vụ hoc t p và công tác của mình. Thái Nguyên, ngày 20 tháng 5 năm 2018 Tác gia lu n văn Tran Thị HưỜng
  • 6. 1 Viết đề tài giá sinh viên – ZALO:0973.287.149-TEAMLUANVAN.COM Chương 1 M t so kien th c liên quan đen các hàm mũ và hyperbolic 1.1 Tính chat cơ ban cua các hàm mũ và hyperbolic 1.1.1 Tính chat cơ ban cua hàm mũ Xét hàm so mũ dạng f (x) = ax vói 0 < a /= 1. * T p xác định: Df = R. * T p giá trị: If = R+ . * Tính đơn đi u: Hàm so f(x) = ax đong bien trên R khi a > 1 và nghịch bien trên R khi 0 < a < 1. Nh n xét 1.1. Đo thị hàm so mũ có ti m c n ngang là trục Ox ve phía −∞ khi a > 1 và ti m c n ngang là trục Ox ve phía +∞ khi 0 < a < 1. Tiep theo, ta xét m t so đȁng thức trong lóp hàm mũ. Tính chat 1.1 (Công thức tính đạo hàm). (ex )′ = ex ; (eu )′ = u′eu , (ax )′ = ax ln a; (au )′ = u′au ln a. Tính chat 1.2 (Đong nhat thức trong lóp hàm mũ). Cho 0 < a /= 1. Khi đó:
  • 7. 2 Viết đề tài giá sinh viên – ZALO:0973.287.149-TEAMLUANVAN.COM a) af (x) = ag(x) ⇔ f(x) = g(x). b) Giả sử b > 0. Khi đó af (x) = b ⇔ f (x) = loga b. c) af (x) > ag(x) ⇔ (a − 1)( f (x) − g(x)) > 0. d) Giả sử b > 0. Khi đó af (x) > b ⇔ (a−1)(f(x)−loga b) > 0. 1.1.2 Tính chat cơ ban cua hàm hyperbolic Trong phan này, ta trình bày m t so tính chat của các hàm mũ đ c bi t, đó là các hàm hyperbolic sinh bỏi e±x . Tính chat 1.3 (Hàm sin hyperbolic). Hàm sin hyperbolic là hàm so lẻ trên R và sinhx = ex −e−x 2 sinhx ≥ 0, ∀x ≥ 0, sinhx < 0, ∀x < 0. (sinhx)′ = coshx;(sinhu)′ = u′ coshu. Ta có (sinhx)′ = coshx ≥ 1, ∀x ∈ R nên hàm so sinhx đong bien trên R. Do (sinhx)′′ = sinhx nên hàm so sinhx loi trên (0;+∞) và lõm trên (−∞;0). Tính chat 1.4 (Hàm cosin hyperbolic). Hàm cosin hyperbolic là hàm so chȁn trên R. coshx = ex +e−x 2 Ta có (coshx)′ = sinhx;(coshu)′ = u′ sinhu và (coshx)′ = sinhx nên hàm so coshx đong bien trên (0;+∞) và nghịch bien trên (−∞;0). Do (coshx)′′ = coshx ≥ 1, ∀x ∈ R coshx loi trên R.
  • 8. 3 Viết đề tài giá sinh viên – ZALO:0973.287.149-TEAMLUANVAN.COM cosh2 x ( ) = − Tính chat 1.5 (Hàm tang hyperbolic). Hàm tang hyperbolic tanhx sinhx ex −e−x là hàm so lẻ trên R. Ta có = coshx = ex + e−x (tanhx)′ = 1 ;(tanhu)′ = u′ . cosh2 x Do cosh2 u (tanhx)′ = 1 > 0, ∀x ∈ R nên hàm so tanh x đong bien trên R. Tính chat 1.6 (Hàm cotang hyperbolic). Hàm cotang cothx coshx ex +e−x là hàm so lẻ trên R {0}. Ta có = sinhx = ex − e−x (cothx)′ = −1 ;(cothu)′ = −u′ sinh2 x sinh2 u và cothx ′ 1 sinh2 x < 0, ∀x ∈ R{0} nên hàm so cothx đong bien trên mői khoảng (−∞;−1) và (1;+∞). Tính chat 1.7 (Công thức khai trien tong và hi u). cosh(x + y) = coshx.coshy + sinhx.sinhy, (1.1) cosh(x − y) = coshx.coshy − sinhx.sinhy, (1.2) sinh(x + y) = sinhxcoshy + coshx.sinhy, (1.3)
  • 9. 4 Viết đề tài giá sinh viên – ZALO:0973.287.149-TEAMLUANVAN.COM − ChÝng minh. Ta có sinh(x − y) = sinhxcoshy − coshx.sinhy, (1.4) tanh(x + y) = tanhx + tanhy , (1.5) 1 + tanhxtanhy tanh(x y) = tanhx − tanhy . (1.6) 1 − tanhxtanhy coshx coshy sinhx sinhy ex +e−x ey +e−y ex −e−x ey −e−y . + . = 2 2 + 2 2 Từ đó, suy ra (1.1). ex+y +e−x−y = 2 = cosh(x+y). Tiep theo, trong công thức (1.1) thay y bang −y, ta thu đưoc cosh(x−y) = coshx.cosh(−y) +sinhx.sinh(−y) = coshx.coshy − sinhx.sinhy. Ta nh n đưoc (1.2). Các công thức còn lại (1.3)-(1.6) đưoc chứng minh tương tụ. Từ công thức c ng ta cũng de dàng chứng minh đưoc các công thức nhân sau đây. Tính chat 1.8 (Công thức khai trien góc nhân hai và nhân ba). sinh(2x) = 2 sinh x. cosh x, cosh(2x) = cosh2 x +sinh2 x = 2cosh2 x − 1 = 1+2sinh2 x, tanh(2x) = 2tanhx , 1+tanh2 x sinh(3x) = 4sinh3 x +3sinhx,
  • 10. 5 Viết đề tài giá sinh viên – ZALO:0973.287.149-TEAMLUANVAN.COM − 2 − cosh(3x) = 4cosh3 x −3coshx. Tính chat 1.9 (Công thức bien đoi tích thành tong). coshxcoshy = 1 [cosh(x+y)+cosh(x y)], 2 sinhx sinhy = 1 [cosh(x + y) − cosh(x − y)] , sinhxcoshy = 1 [sinh(x+y)+sinh(x y)]. 2 Tính chat 1.10 (Công thức bien đoi tong thành tích). coshx+coshy = 2cosh x + y cosh x − y , 2 2 coshx − coshy = 2sinh x + y sinh x − y , 2 2 sinhx + sinhy = 2sinh x + y cosh x − y , 2 2 sinhx − sinhy = 2cosh x + y sinh x − y , 2 2 tanhx + tanhy = sinh(x + y) , coshx.coshy tanhxtanhy = sinh(x − y) . coshx.coshy 1.2 Đang th c sinh bƠi hàm mũ và hàm hyperbolic Trong phan này ta xét m t so dạng toán áp dụng các tính chat của hàm mũ và các hàm hyperbolic. Bài toán 1.1. Tính giá trị các hàm hyperbolic tại điem ln 2 và ln 3. L i giai. Theo định nghĩa, ta có sinh ln 2 eln 2 − e−ln 2 3 ( ) = 2 = 4
  • 11. 6 Viết đề tài giá sinh viên – ZALO:0973.287.149-TEAMLUANVAN.COM 3 3 √ √ 3 ⇔ a ⇔ 3 − ⇔ ⇔ − và cosh(ln 2) = eln 2 +e−ln 2 = 5 ;tanh(ln 2) = 3 ;coth(ln 2 5 Tương tụ, ta có 2 sinh ln 3 4 5 eln 3 −e−ln 3 4 ) = 3 . ( ) = 2 = 3 và cosh(ln 3) = eln 3 + e−ln 3 = 5 ;tanh(ln 3) = 4 ;coth(ln 3) = 5 . 2 3 5 4 Bài toán 1.2. Giải các phương trình và bat phương trình sau: a. e2x +e−2x 5 = 2 . b. e3x −e−3x ≥ 8 . c. ax − a−x < 2 , 0 < a /= 1. L i giai. a. e2x +e−2x = 5 2 e2x +e−2x 5 2 = 4 ⇔ cosh2x = cosh(ln 2) ⇔ 2x = ±ln 2 do hàm coshx đong bien trên (0;+∞) và nghịch bien trên (−∞;0). V y phương trình có hai nghi m x = ±ln 2. b. e3x − e−3x ≥ 8 ⇔ e3x e−3x 4 2 ≥ 3 ⇔ sinh3x ≥ sinh(ln 3) ⇔ 3x ≥ ln 3 do hàm sinhx đong bien trên R. V y bat phương trình có nghi m x ≥ ln 3. c. ax − a−x < 3 2 exln a − e−xln a < 3 2 exln a e−xln a 3 2 < 4 ⇔ sinh(xln a) < sinh(ln 2) ⇔ xln a < ln 2. ln 2 Neu a > 1 bat phương trình có nghi m x < ln a ⇔ x < loga2. Neu 0 < a < 1 bat phương trình có nghi m x > ln 2 x > log 2. ln a Bài toán 1.3. Chứng minh bat đȁng thức a. coshx ≥ 1, ∀x ∈ R.
  • 12. 7 Viết đề tài giá sinh viên – ZALO:0973.287.149-TEAMLUANVAN.COM 1 ∀ / x = b. −1 < tanhx < 1, ∀x ∈ R. c. cothx > 1, ∀x > 0 & cothx < −1, ∀x < 0. d. sinh3 x+cosh3 x ≥ 1, ∀x ∈ R. L i giai. a. coshx ≥ 1. ∀x ∈ R. Ta có coshx = ex +e−x 2 ≥ √ ex.e−x = 1. Dau đȁng thức xảy ra khi và chỉ khi x = 0. Từ đó ta có đieu can chứng minh. b. −1 < tanhx < 1. ∀x ∈ R. Ta có tanhx ex −e−x e2x −1 1 2 = ex + e−x = e2x + 1 = − e2x + 1 . Do e2x > 0 nên −1 < tanhx < 1, ∀x ∈ R. c. cothx > 1, ∀x > 0 & cothx < −1, ∀x < 0. Ta có cothx = , x = 0 và từ phan b. Từ đó ta có đieu can chứng minh. tanhx d. Ta có 3 3 ex −e−x 3 ex +e−x 3 e3x +3e−x e3x +e−x +e−x +e−x √ 4 = 4 ≥ e3x.e−x.e−x.e−x = 1. Dau đȁng thức xảy ra khi và chỉ khi x = 0. Từ đó ta có đieu can chứng minh. Tiep theo, ta xét m t so ví dụ minh hoa sau. Bài toán 1.4. Chứng minh rang a. sinhx + sinh3x + sinh5x cosh x+cosh 3x + cosh 5x = tanh3x. b. tanhx + tanh2x − tanh3x = tanh xtanh 2x tanh3x. L i giai. 4 2 2 sinh x + cosh + =
  • 13. 8 Viết đề tài giá sinh viên – ZALO:0973.287.149-TEAMLUANVAN.COM 2 − 2 2 2 2 2 2 2 2 a. sinhx + sinh3x + sinh 5x cosh x+cosh 3x + cosh 5x 2sinh 3xcos2x + sinh3x = 2cosh3xcosh2x + cosh3x = tanh3x. b. tanh x+tanh 2x − tanh 3x = tanh x+tanh 2x − tanh(x + 2x) tanh x+tanh 2x = tanh x + tanh2x − 1 + tanhxtanh 2x = (tanhx+tanh2x) 1 1 1 + tanh x tanh 2x = (tanhx+tanh2x) tanh x tanh 2x 1 + tanh x tanh 2x = tanhx+tanh 2x tanhx tanh2x = tanh x tanh2xtanh 3x. 1 + tanh x tanh 2x Bài toán 1.5. Tính các tong sau: Sn = sinhx + sinh2x + sinh3x + ·· · + sinhnx Tn = coshx+2cosh2x+3cosh3x+···+ncoshnx L i giai. Neu x = 0 thì Sn = 0 Xét x = / 0. Nhân cả hai ve Sn vói sinh x , ta đưoc Sn.2sinh x 2 x x x = 2sinh 2 sinhx + 2sinh 2 sinh2x + 2sinh 2 sinh3x + ... x + 2sinh 2 sinhnx = cosh 3x − cosh x + cosh 5x − cosh 3x + cosh 7x −cosh 5x +···+ cosh 2n+1 x −cosh 2n−1 x
  • 14. 9 Viết đề tài giá sinh viên – ZALO:0973.287.149-TEAMLUANVAN.COM 2 2 2 2 = 2 x 2 2 2 2 2 2 4sinh2 x 2 2 2 2 2 2 2n+ 1 x Suy ra = cosh 2 x − cosh 2 . cosh 2n+1 x − cosh x Sn = 2 x 2 . 2sinh 2 Neu x = 0 thì Tn = 1+2+3+···+n = n(n+1) . Xét x /= 0, thì Sn ′ = coshx+2cosh2x+3cosh3x+···+ncoshnx. Suy ra ′ cosh 2n+1 x−cosh x !′ 2sinh 2 2n+1 sinh 2n+1 x − 1 sinh x 2sinh x − cosh x cosh 2n+1 x − cosh x (2n + 1)sinh 2n+1 xsinh x − sinh2 x − cosh 2n+1 xcosh x + cosh2 x = 2 2 2 2 2 2 4sinh2 x 1 + 2n+1 (cosh(n + 1)x − coshnx) − 1 (cosh(n + 1)x + coshnx) = 2 2 x 2 4sinh 2 = 1+ncosh(n+1)x−(n+1)coshnx 4sinh2 x = n(cosh(n+1)x−coshnx)+1−coshnx 4sinh2 x 2nsinh 2n+1 xsinh x − 2sinh2 nx . 2 2 2 4sinh2 x nsinh 2n+1 xsinh x − sinh2 nx = 2 2 2 . 2sinh2 x Bài toán 1.6. Chứng minh bat đȁng thức cosh(5x − 7) ≥ √ 25x2 − 70x + 50. Tn = Sn =
  • 15. 10 Viết đề tài giá sinh viên – ZALO:0973.287.149-TEAMLUANVAN.COM 5 √ L i giai. Xét hàm so y = cosh2 (5x − 7) − (5x − 7)2 + 5x − 1, ∀x ∈ R. Ta có y′ = 5sinh(2(5x −7))− 10(5x −7) + 5;y′′ = 50cosh(2(5x −7))− 50 ≥ 0. Do đó y ≥ y 7 +y′ 7 x− 7 . 5 5 5 Ta có y 7 = 7;y′ 7 = 5 5 5 nên cosh2 (5x−7)−(5x−7)2 +5x−1 ≥ 7+5 x− 7 . Suy ra cosh2 (5x−7) ≥ 25x2 −70x+50. Từ đó ta có đieu can chứng minh cosh(5x − 7) ≥ 25x2 − 70x + 50. 1.3 M t so bat đang th c ch a đạo hàm và tích phân quan trong Trong phan này, ta trình bày các bat đȁng thức Landau, Kolmogorov và Steklov đoi vói đạo hàm và tích phân các hàm so đ c bi t. Và trình bày m t so áp dụng vào chứng minh các bat đȁng thức và giải bài toán cục trị vói hàm mũ và hàm hyperbolic. Định lí 1.1 (Bat đȁng thức Landau). Cho f : R → R là m t hàm của lóp C2 . Giả sử f và f′′ bị ch n. Đ t M0 = sup| f (x)|,M1 = sup| f ′(x)|, M2 = sup| f ′′(x)|. x∈R x∈R x∈R Chứng minh rang f′ bị ch n và M1 ≤ 2 √ M0M2. ChÝng minh. Đau tiên, ta nh n thay rang neu M2 = 0 thì hàm f chỉ đáp ứng đưoc
  • 16. 11 Viết đề tài giá sinh viên – ZALO:0973.287.149-TEAMLUANVAN.COM − h r M 2 M 2 1 đieu ki n liên tục. Do v y, không mat tính tong quát ta có the giả sử M2 > 0. Lay x ∈ R và m t giá trị tùy ý h > 0. Theo công thức Taylor ton tại t ∈ (x;x + 2h) sao cho f (x + 2h) = f (x) + 2h f ′(x) + 2h2 f ′′(x). Do v y f ′ (x) = f (x +h)− f (x) h f ′′ (x). 2h Lay modun và áp dụng các giả thiet ban đau, ta thu đưoc | f ′(x)| ≤ M0 + M2h. V y f′(x) bị ch n. M t khác, chon h = M0 , ta có M2 |f′(x)| ≤ M0 : r M0 +M2 r M0 ⇔ M1 ≤ 2 √ M0M2. Định lí 1.2 (Bat đȁng thức Kolmogorov). Cho f : R → R là m t hàm của lóp C3 . Giả sử f và f′′′ bị ch n. Đ t M0 = sup| f (x)|, M1 = sup| f ′(x)|,M3 = sup| f ′′′(x)|. x∈R x∈R ′ x∈R 2 1 a. Chứng minh rang f bị ch n. Hơn nữa, M1 ≤ 2 (9M0 M3)3 b. f′′ có bị ch n hay không ? ChÝng minh. a. Co định x ∈ R và h /= 0. Áp dụng công thức khai trien Taylor cho các giá trị ỏ giữa h và x+h, ta thu đưoc: |f(x+h)− f(x)−hf ′ (x) − h2 f 2 h3 ′′ (x)| ≤ M3 6
  • 17. 12 Viết đề tài giá sinh viên – ZALO:0973.287.149-TEAMLUANVAN.COM | | và V y nên |f(x−h)− f(x)+hf ′ (x) − h2 f 2 h3 ′′ (x)| ≤ M3 6 . 2h|f′(x)| = |( f(x — h) − f (x) + hf′ (x) − h2 f′′ 2 (x)) −( f (x +h)− f (x)− h f H thức này cho ta bat đȁng thức ′ (x) − h2 f 2 ′′ (x)) + f (x + h) − f (x − h)|. 2h|f′(x)| ≤ | f(x — h) − f (x) + hf′ (x) − h2 f′′ 2 (x)| +|f (x+ h)− f (x)−h f Do đó ′ (x) − h2 f 2 ′′ (x)|+ | f(x+h)|+ | f (x−h)| = M3h3 3 + 2M0. V y f′(x) bị ch n. |f′ (x)| ≤ M0 + h m3h2 6 =: ψ(x). 0 0 −1 1 M t khác, ψ đạt cục tieu tại h = (3M M3 )3 . 1 2 1 1 2 1 Từ ψ(h) = 2 (9M0 M3)3 ta có M1 ≤ 2 (9M0 M3)3 . b. Theo giả thiet và theo ý câu (a) các hàm f′ và f′′′ = ( f′′)′ là các hàm bị ch n. Áp dụng bat đȁng thức Landau, ta có f′′ bị ch n. Định lí 1.3 (Bat đȁng thức Landau-Kolmogorov). Cho f : R → R là m t hàm của lóp Cn . Giả sử f và f (n) bị ch n. a. Chứng minh rang f (n−1) bị ch n. b. Từ đó suy ra tat cả các đạo hàm f(k) vói moi 1 ≤ k ≤ n−1 đeu bị ch n. c. Đ t Mk = sup f (k)(x) . x∈R Chứng minh rang Mk > 0 ứng vói moi 0 ≤ k ≤ n; k ∈ N. d. Sử dụng các hàm uk = 2k−1 MkMk −1 ;0 ≤ k ≤ n,k ∈ N đe chứng minh rang k(n−k) 1−k k − Mk ≤ 2 2 M0 Mn . n n 1
  • 18. 13 Viết đề tài giá sinh viên – ZALO:0973.287.149-TEAMLUANVAN.COM . ( j) . k . k . . ≤ . m m − ∑ j=1 ( ) ∑( k=1 ) n−1 . ≤ ∑ k=1 n−1 ChÝng minh. Theo công thức khai trien Taylor, ta có n−1 kj f x . 2M Mnkn Do v y ∑ j=1 ( ). ≤ 0 + n! . n−1 C n−1 kj f(j) x . n−1 C 2M Mnkn . ∑ k=1 Đieu đó cho thay n 1 ∑ k= j ( ) ∑ k=1 n−1 0 + n! . n−1 1 f( j) x n−1 1 n−1−k Ck kj. n−1 Ck 2M Mnkn m M t khác ∑ k=1 (−1)m−k Ck k j = 0 neu 1 ≤ j ≤ m − 1, m ho c ∑ (−1)m−k Ck k j = m! neu j = m. k=1 Ta thay rang tat cả các bat đȁng thức bên trái đeu không xảy ra ngoại trừ trưòng hop j = n − 1. Theo đó ∀x ∈ R, ta đeu có (n−1) n−1 k Mnkn |f V y nên f(n−1)(x) bị ch n. (x)| ≤ ∑Cn 1(2M0 + k=1 n! ). b. ta de dàng có f(k) vói moi 1 ≤ k ≤ n − 1 đeu bị ch n bang phương pháp qui nạp. Th t v y theo giả thiet và theo ý a ta có f và f (n−1) bị ch n nên ta có f (n−2) bị ch n. Tương tụ, ta có f và f (n−2) bị ch n nên ta có f (n−3) bị ch n. Cứ tiep tục quá trình trên, ta có tat cả các đạo hàm f (k) vói moi 1 ≤ k ≤ n − 1 đeu bị ch n. n! . j! − j! . . . − j! 0 + .
  • 19. 14 Viết đề tài giá sinh viên – ZALO:0973.287.149-TEAMLUANVAN.COM 0 n a |f(x)| a |f (x)| ( ) = 2 a ( ) x ( ) = 2 ( ) c. Neu Mk = 0 thì f là m t đa thức có b c cao nhat là k − 1. Từ f là hàm bị ch n, do đó f chỉ thỏa mãn đưoc đieu ki n liên tục của giả thiet. Đieu đó cho thay Mk > 0 vói moi 0 ≤ k ≤ n,k ∈ N. d. Áp dụng bat đȁng thức Landau, ta có Mk ≤ √ 2Mk−1Mk+1,∀1 ≤ k ≤ n−1. Đieu này. cho thay bat đȁng thức u1 ≤ u2 ≤ ··· ≤ un, trong đó (u1u2 ...uk)n ≤ (u1u2 ...un)k . Áp dụng bat đȁng thức trong trưòng hop này, ta có đưoc k k(n−k) 1− n k Mk ≤ 2 2 M Mn . V y ta có đieu phải chứng minh. Định lí 1.4 (Bat đȁng thức Steklov). Giả sử hàm f (x) khả vi liên tục trên khoảng (a;b) và sao cho f(a) = f(b) = 0. Khi đó, có bat đȁng thức ∫ b 2 (b−a)2 ∫ b ′ 2 ChÝng minh. Bieu dien hàm f đã cho ỏ dạng f x 1h∫ x f′ t dt − ∫ b f ′ t dy i 1 ∫ b sign x −t f ′ t dt dx ≤ 4 dx. (1.7) a ( ) .
  • 20. 15 Viết đề tài giá sinh viên – ZALO:0973.287.149-TEAMLUANVAN.COM 2 2 2 ( ) 2 a ( ) 4 a a ( ) 4 a ( ) a |f(x)| π2 a |f (x)| Áp dụng bat đȁng thức Buniakowski ta có |f x | h1 ∫ b sign x −t f′ t dt i2 ≤ 1 ∫ b |sign x −t | dt ∫ b |f′ t | dt b− a ∫ b | f ′ t | dt lay tích phân theo x trên (a;b), ta đưoc ∫ b 2 (b−a)2 ∫ b ′ 2 a |f (x)| Định lý đưoc chứng minh. dx ≤ 4 a |f (x)| dx. Nh n xét 1.2. Nh n xét rang, có the làm tot bat đȁng thức (1.7) bỏi bat đȁng thức ∫ b 2 (b−a)2 ∫ b ′ 2 Tuy nhiên, chứng minh bat đȁng thức (1.8) không đơn giản vì nó đòi hỏi nhieu kien thức của Toán cao cap (theo sụ hieu biet của chúng tôi), nên chúng tôi sẽ không trình bày chứng minh ay trong lu n văn này. Tiep theo, ta xét m t so dạng bat đȁng thức sinh bỏi đạo hàm liên quan đen các bat đȁng thức co đien. Ta chứng minh bo đe sau. Bo đe 1.1. Cho f : R → R là m t hàm khả vi và x lim f(x) ≥ 0 và f′(x) > 0,∀x ∈ R. Khi đó f (x) > 0 vói moi x ∈ R. →−∞ ChÝng minh. Th t v y, giả sử ton tại m t so x0 ∈ R sao cho f(x0) < 0. Theo giả thiet, ta có f′ > 0 nên f(x) < f(x0) vói x < x0. Do đó lim x→−∞ f (x) ≤ f (x0) < 0, mâu thuȁn. Bài toán 1.7 (Kolmogorov). Cho f : R → R là m t hàm khả vi b c 3 trên R sao cho f, f ′, f ′′, f ′′′ đeu là các so dương. Hơn nữa, ta giả sử f ′′′(x) ≤ f (x), ∀x ∈ R. Chứng 2 = ( ) ( ) . = dx ≤ dx. (1.8)
  • 21. 16 Viết đề tài giá sinh viên – ZALO:0973.287.149-TEAMLUANVAN.COM − 2 minh rang f ′(x) < 2 f(x), ∀x ∈ R. L i giai. Đau tiên ta chứng minh lim x→−∞ và lim x→−∞ f ′(x) = 0 f′′(x) = 0. Th t v y, theo định lí giá trị trung bình ton tại ξn ∈ (−n2 ;−n), sao cho f (−n2 ) − f (−n) = (n − n2 ) f ′(ξn) < 0. Theo giả thiet ban đau, các giói hạn của f và f ′ tại −∞ ton tại và chúng không âm. Ta giả sử phản chứng rang lim x→∞ f ′(x) > 0. Lay giói hạn trong đȁng thức trên tại −∞ ta có 0 = −∞ (mâu thuȁn). Do đó lim x→−∞ f′(x) = 0. ′′ Chứng minh tương tụ, ta cũng có lim f x→−∞ (x) = 0. ′′′ Bây giò ta áp dụng bo đe, theo giả thiet, ta có f (x) ≤ f (x). Do v y f ′′(x) f ′′′(x) ≤ f ′′(x) f (x) ≤ f ′′(x) f (x)+ f ′2 (x), ∀x ∈ R. Sử dụng bat đȁng thức này, đong thòi áp dụng bo đe trên cho hàm f (x) f ′(x) 1 ( f ′′(x))2 , 2 ta thu đưoc 1 ( f ′′(x))2 < f (x) f ′(x), ∀x ∈ R. M t khác, do f > 0 và f ′′′ > 0, nên ta có f ′2 (x) ≤ 2 f (x) f ′′(x),∀x ∈ R.
  • 22. 17 Viết đề tài giá sinh viên – ZALO:0973.287.149-TEAMLUANVAN.COM 2 b − a Ket hop hai bat đȁng thức trên, ta thu đưoc 1 f′2 (x) 2 2 ( 2 f(x) ) ≤ 1 ( f 2 ′′ (x))2 < f(x)f ′(x). Do v y f ′3 (x) < 8 f 3 (x),∀x ∈ R. Đieu đó cho thay f′(x) < 2f(x). Nh n xét 1.3. Ta có the chứng minh m t bat đȁng thức trên bang m t bat đȁng thức mạnh hơn bang cách thay the hang so 2, bang m t hang so nhỏ hơn 2. Th t v y, thêm 1 f ′(x) f′′(x) vào 2 ve của bat đȁng thức 2 1 ( f′′(x))2 < f(x)f′(x), ∀x ∈ R và áp dụng giả thiet f′′′(x) ≤ f(x) ta có 1 [ f ′(x) f ′′′(x) + ( f ′′′(x))2 ] < f (x) f ′(x) + 1 f ′(x) f ′′′(x) ≤ 3 f (x) f ′(x). 2 2 2 Áp dụng bo đe trên, ta có 1 f ′(x) f ′′(x) < 3 f 2 (x),∀x ∈ R. 2 4 Nhân hai ve vói f′(x) và áp dụng bo đe trên ta suy ra rang 1 f ′3 (x) < 1 f 3 (x),∀x ∈ R. 6 4 ′ 3 1 Do đó f (x) < ( 2 )2 f (x) < 2 f (x). Bài toán 1.8. Cho f : [a; b] → R là m t hàm mà có đạo hàm (không nhat thiet phải hữu hạn) tại bat kì tại điem nào trong [a;b]. Chứng minh rang ton tại x0 ∈ [a;b] sao cho | f (a) − f (b) | ≤ | f ′ (x0)|.
  • 23. 18 Viết đề tài giá sinh viên – ZALO:0973.287.149-TEAMLUANVAN.COM 10 | − | − b − a − L i giai. Đoi vói bat kì so nguyên 0 ≤ n ≤ 10. Xét dãy cn = a + (b − a)n . ta có f(a) f(b) | b − a 9 ∑ f (cn+1 f (cn) . n=0 Như v y, ton tại m t so nguyên k vói 0 ≤ k ≤ 0 sao cho | f (ck+1) − f (ck) | ≥ | f (a) − f (b) |. ck+1 −ck b − a Xét k1 là so nhỏ nhat thỏa mãn đieu ki n trên và xác định a1 = ck1 và b1 = ck1+1. Tiep theo, ta l p lu n tương tụ vói các khoảng so thục I1 := [a1; b1] ta có khoảng so thục I2 := [a2;b2]. Cứ tiep tục l p lu n như trên, ta có m t chuői các so thục In := [an;bn] sao cho bn −an = 10−n (b−a) và | f (bn) − f (an) | ≥ | f (a) − f (b) |. bn −an b − a Từ In là m t khoảng đóng |In| = (bn −an) → 0 và I1 ⊃ I2 ⊃ ··· ⊃ In ⊃ ... theo bo đe Cantor ton tại duy nhat m t giá trị x0 sao cho x0 ∈ n T 1 In. Đ c bi t, đieu này cho thay x0 = lim an = lim bn. ≥ n→ f ∞ n→∞ Ta thay rang trong bat đȁng thức | (bn) − f (an) | ≥ | f (a) − f (b) | có the thay the m t trong hai giá trị an ho c b bn an n bỏi giá trị x0. b − a Theo đó, do bat đȁng thức trên ket hop vói đong thức nhat cơ bản, ta có f (bn) − f (an) = f (bn)− f (x0) bn − x0 + f (x0) − f (an) x0 −an . bn − an bn − x0 bn − an x0 − an bn − an Trong ket lu n, ta có the giả định bat đȁng thức trên cho vô hạn chỉ so nk,k ≥ 1 và cho an thay the bỏi x0. Lay nk → ∞ và sử dụng định nghĩa của hàm f tại x0, ta có | f (a)− f (b) | ≤ | f ′ (x0)|. | ≤
  • 24. 19 Viết đề tài giá sinh viên – ZALO:0973.287.149-TEAMLUANVAN.COM . . . . ≤ ⇒ | | | ··· | ≤ n 1 2 | f ′ (0)| = lim . f (x) − f (0). = lim . f (x). . .sinx|. Bài toán 1.9. Cho hàm so f(x) = a1 sinx+a2 sin2x+ ···+an sinnx. Giả sử | f (x)| ≤ |sinx|. Chứng minh rang |a1 + 2a2 + ··· + nan| ≤ 1. L i giai. Vì f (0) = 0 và | f (x)| ≤ |sinx|, nên ta có x→0 x . x→0 . sinx . . x . f (x) = lim 1, f′(0) = a +2a + +na 1. x→0 sinx Bài toán 1.10. Cho a là so thục dương. Chứng minh rang ex > xa ⇔ a < e vói x > 0. LỜi giai. Xét hàm f ex (x) = xa ,x > 0. Ta có f(x) → ∞ khi x → 0 và x → ∞. Do đó ton tại giá trị cục tieu của f tại x0. Theo định lí Fermat ta có f ′(x0) = ex x−a 1 − a = 0. 0 0 x0 Giá trị này nhỏ nhat tại x = a và f ea (a) = aa . Do v y ex > xa ⇔ a < e,x > 0. Bài toán 1.11. Cho f là m t hàm có đạo hàm cap 3 liên tục trên đoạn [0;1], thỏa mãn f (0) = f ′(0) = f ′′(0) = f ′(1) = f ′′(1) = 0 và f (1) = 1. Chứng minh rang ton tại c ∈ [0;1] sao cho f ′′′(c) ≥ 24. L i giai. Hãy xem xét mỏ r ng chuői Taylor tại các điem x = 0 và x = 1 :
  • 25. 20 Viết đề tài giá sinh viên – ZALO:0973.287.149-TEAMLUANVAN.COM − − f (x) = f (0) + f ′(0)x + f ′′ (0) x2 + f ′′′ (ξx) x3 , 2 6 f (x) = f (1) + f ′(1)(x − 1) + f ′′ (1) (x − 1)2 + f ′′′ (ηx) (x − 1)3 2 6 vói ≤ ξx ≤ x;x ≤ ηx ≤ 1. Ket hop vói các giả thiet, ta có f (x) = f ′′′ (ξx) x3 , 6 f (x) = 1 + f ′′′ (ηx) (x 1)3 . 6 Chon x = 1 ta thay ton tại ξ;η sao cho f ′′′(ξ)+ f ′′′(η) = 48. 2 Như v y, m t trong 2 so f ′′′(ξ), f ′′′(η) lón hơn ho c bang 24 . V y ton tại c ∈ [0;1] đe f ′′′(c) ≥ 24. Bài toán 1.12. Cho hàm so f khả vi trên [0;1] sao cho f (0) = 0, f (1) = 1. Chứng minh ton tại 2 so phân bi t a,b thu c (0;1) sao cho f′(a). f′(b) = 1. L i giai. Xét hàm so g(x) = f (x)+x−1 thì g khả vi trên [0;1]. Ta có g(0) = −1 < 0 và g(1) = 1 > 0 nên ton tại so c ∈ (0;1) sao cho g(c) = 0. Do đó f (c) +c − 1 = 0 hay f (c) = 1− c. Áp dụng định lý Lagrange cho f trên các đoạn [0;c] và [c;1] thì: + Ton tại a ∈ (0;c) sao cho f(c)− f(0) = f ′(a). + Ton tại b ∈ (c;1) sao cho c 0 f(1)− f (c) 1 − c = f ′(b), nên f ′ (a). f ′ (b) = f (c) . 1 − f (c) = (1 − c)c = 1. c 1− c c(1− c) V y ton tại 2 so phân bi t a,b thu c (0;1) sao cho f′(a). f′(b) = 1. Bài toán 1.13. Cho hàm f : R → R là m t hàm trên C3 . Chứng minh rang ton tại a ∈ R sao cho f (a) f ′(a) f ′′(a) f ′′′(a) ≥ 0.
  • 26. 21 Viết đề tài giá sinh viên – ZALO:0973.287.149-TEAMLUANVAN.COM ∀ L i giai. Neu m t trong các giá trị f (a), f ′(a), f ′′(a); f ′′′(a) bang 0 ta có luôn đieu phải chứng minh. Ta giả sử các giá trị f(x), f′(x), f′′(x), f′′′(x) đeu âm hay dương vói moi x. Giả sử f′′(x) > 0, f(x) > 0, (neu f′′(x) < 0 ta có the thay f(x) thành −f (x), neu f(x) < 0 ta có the thay the f(x) bang f(−x). Từ f ′′(x) > 0 ta có f′(x) là hàm tăng, m t khác f ′′′(x) > 0 đieu đó cho thay f ′(x) là m t hàm loi. Khi đó ta có f ′(x + a) > f ′(x) + a f ′′(x), ∀x ∈ R. Cho x co định lay a đủ lón, ta sẽ có f ′(x) > 0,∀x ∈ R. Vói l p lu n tương tụ ta sẽ có f ′(x) > 0 và f ′′(x) > 0 khi f (x) > 0, ∀x ∈ R. Do v y ton tại a ∈ R sao cho f (a) f′(a) f ′′(a) f′′′(a) ≥ 0. Bài toán 1.14. Cho hàm so f,g thỏa mãn đieu ki n f′ (x) g′(x) = ef(x)−g(x), ∀x và f(0) = g(2018) = 1. Tìm hang so c lón nhat sao cho f(2018) > c. L i giai. Ta có f′ (x) = ef (x)−g(x), x hay f ′(x).e− f (x) = g′(x).e−g(x). Từ đó ta có g′(s) e− f (x) = e−g(x), ∀x ∈ R. Xét hàm so h(x) := e− f (x) − g′(x).e−g(x) = a vói a là m t hang so. Khi đó ta có h(0) = h(2018), ket hop vói giả thiet ta đưoc e−1 − e−g(0) = e− f (2018) − e−1 . Do v y e− f (2018) = 2e−1 − e−g(0) < 2e−1 = e−1+ln 2 . V y ta chon hang so c lón nhat là c = −1 + ln 2 thỏa mãn đieu ki n f (2018) > c. Bài toán 1.15. Chứng minh rang ab + ba > 1,∀a > 0,b > 0
  • 27. 22 Viết đề tài giá sinh viên – ZALO:0973.287.149-TEAMLUANVAN.COM ( − ) = − < − . L i giai. - Bat đȁng thức trên là hien nhiên neu ít nhat các so a ho c b lón hơn 1. - Ta đi xét các giá trị a,b ∈ (0;1). Th t v y, đ t a = 1 − c,b = 1 − d;c,d ∈ (0;1). Ta xét hàm so sau f (x) = (a − cx)d ,x ∈ [0;1]. Áp dụng định lí Lagrange ton tại x0 ∈ (0;1) sao cho: 1 c d 1 cd 1 cd (1 − cx0)1−d Từ đó ab + ba = 1 − c + 1− d > 1 − c + 1− d = (1 − c)(1 − d) + 1 > 1. (1− c)d (1 − d)c 1− cd 1−cd 1−cd Bài toán 1.16. Cho f : R → R là m t hàm của lóp C3 sao cho f, f′, f′′, f′′′ đeu là những so dương. Hơn nữa ta giả sử f′′′(x) ≤ f(x), ∀x ∈ R. Chứng minh rang f ′(x) < 2 f (x), ∀x ∈ R. L i giai. Ta nh n thay, từ giả thiet ta có lim x→−∞ f ′(x) = lim x→−∞ f′′(x) = 0. Th t v y, theo định lí giá trị Lagrange, ton tại ξn ∈ (−n2 ;−n) sao cho f (−n2 ) − f (−n) = (n − n2 ) f ′(ξn) < 0. Bang giả thiet giói hạn của f và f ′ tại −∞ ton tại và hơn nữa các giói hạn đó không âm. Ta giả sử phản chứng lim x f ′(x) > 0. Đe có giói hạn tại −∞ từ trên ta có đưoc 0 = −∞ →−∞ (mâu thuȁn). Do v y ta có lim x→−∞ f′(x) = 0.
  • 28. 23 Viết đề tài giá sinh viên – ZALO:0973.287.149-TEAMLUANVAN.COM µ λ x>0 λ(1 − λ)π 2 1 − a ≤ sup. sin(ax) − sin(bx) . ≤ 4 1 − a , ∀0 < a < b. L p lu n tương tụ, ta cũng có đưoc lim x→−∞ f′′(x) = 0. V y theo bo đe, ta có đưoc f ′(x) < 2 f (x),∀x. Bài toán 1.17. Cho f : [0;1] → R là m t hàm liên tục trên [0;1] khả vi trên khoảng (0; 1) và thỏa mãn 0 ≤ f ′(x) ≤ 2 f (x), ∀x ∈ (a; b). Chứng minh rang f đong nhat bang 0. L i giai. Xét g := [0;1] → R là m t hàm xác định bỏi g(x) = e−2x f (x). Khi đó g′(x) = e−2x ( f ′(x) − 2 f (x)) ≤ 0. Do đó g là hàm giảm. M t khác, g(0) = 0,g ≥ 0, nên ta có g ≡ 0 hay f ≡ 0. Bài toán 1.18. Chứng minh rang π b x>0 ax bx b L i giai. Xét 0 < a < b. Đ t λ a = b , f (x) = sin x x ,x > 0 và ( ;x) = f (x) − f (λx) . 1− λ Từ | f (x)| < 1,∀x > 0, ta có đưoc 2 |µ(λ;x)| < 1 − λ ≤ 4 vói đieu ki n 0 < λ < 1. Đieu này chứng tỏ sup|µ(λ;x)| ≤ 4, ∀λ ∈ (0;1). Đe chứng minh phan bat đȁng thức còn lại, ta nh n thay µ(λ ;π) = sin(λπ) = µ(1 − λ, π).
  • 29. 24 Viết đề tài giá sinh viên – ZALO:0973.287.149-TEAMLUANVAN.COM 2 2 2 − Từ đȁng thức đó, ta đi xem xét các trưòng hop ứng vói giá trị λ ∈ 0; 1 . Vì f là hàm giảm trên h 0; π i , ta có 1 sin π 2 vói moi λ ∈ 0; 1i . µ(λ;π) ≥ λ π 2 ≥ π 2 Bài toán 1.19. Cho f là m t hàm thục khả vi cap n+1 trên [a;b], thỏa mãn f (i)(a) = f (i)(b) = 0,∀i ∈ {1,2,3,...,n}. Chứng minh rang phương trình f (n+1)(x) = f (x) có nghi m trên [a;b]. L i giai. Đau tiên ta giả sử n = 0 và xét hàm so g(x) = e−x f (x). Theo định lí Rolle, ton tại x0 ∈ [a;b] sao cho 0 = g′(x0) = e−x0 ( f ′(x0)− f (x0)). Đieu đó cho thay f′(x0) = f (x0). Tiep theo ta giả sử n ≥ 1 và xét hàm h(x) = n ∑ k=0 f(k)(x). Khi đó h(a) = h(b) = 0 và h(x) = h′(x) = f (x)− f n+1 (x). Áp dụng cách chứng minh n = 0 đoi vói hàm h ta suy ra đưoc rang f(n+1)(x) = f(x) có nghi m trên [a;b]. Bài toán 1.20. Cho f là m t hàm khả vi và f′ liên tục trên [a;b] sao cho ton tại giá trị m ∈ [a;b] đe f ′(m) = 0. Chứng minh rang phương trình f ′(x) = f (x) − f (a) có nghi m trên [a;b]. L i giai. Xét hàm g : [a;b] → R vói g(x) = f ′ (x) f (x) − f (a) . b − a b − a Không mat tính tong quát ta giả sử f(m) > f(a). Xét x1 ∈ [a;m] sao cho f(x1) = max{ f (x);x ∈ [a;m]}. Khi đó ta có f(x1)− f(a) > 0 ≥ (b−a) f′(x1). Đieu đó cho thay g(x1) < 0.
  • 30. 25 Viết đề tài giá sinh viên – ZALO:0973.287.149-TEAMLUANVAN.COM ∈ 2 1 2 Theo định lí Lagrange, ton tại x (a;x ) sao cho f ′(x ) = f (x1)− f (a) . Vì the x1 − a g(x2) = f(x1 − f(a) − f(x2)− f(a) . x1 − a b − a M t khác, do g là m t hàm liên tục, nên ton tại x ∈ [x2;x1] sao cho g(x) = 0. Do v y phương trình f′(x) = f(x)− f(a) có nghi m trên [a;b]. b − a Bài toán 1.21. Chứng minh rang không ton tại hàm dương liên tục, khả vi f trên [0;∞) sao cho f′(x) ≥ f( f(x)) vói moi x ≥ 0. L i giai. Giả sử f là m t hàm thỏa mãn đieu ki n bài toán. Từ f′(x) ≥ f( f(x)) > 0 nên f là m t hàm tăng. Đieu đó cho thay f ′(x) ≥ f ( f (x)) ≥ f ( f (0)) > 0. Đieu này chứng tỏ f là m t hàm ch n dưói bỏi 0 và lim f(x) = ∞. Vì the lim x→∞ f(f (x)) = ∞ và lim x→∞ f′(x) = ∞. x→∞ Xét g(x) := f (x)−x−1. Do g′(x) dan tien ra ∞ khi x → ∞ nên g(x) → ∞ khi x → ∞. Do đó ton tại x0 thỏa mãn f(x0) > x0 +1. Tiep theo áp dụng định lí giá trị trung bình trên đoạn [x0; f(x0)], ta có đưoc m t điem ξ ∈ (x0; f (x0)) thỏa mãn đieu ki n f ( f (x0)) = f (x0) + f ′(ξ)( f (x0) − x0) > f ′(ξ)( f (x0) − x0), ≥ f ( f (ξ)( f (x0) − x0) > f ( f (x0))( f (x0) − x0) > f ( f (x0)) mâu thuȁn. V y không ton tại hàm f thỏa mãn bài toán. Bài toán 1.22. Cho hàm so f(x) khả vi trên đoạn [a;b] và thỏa mãn đieu ki n | f (x)|2 + | f ′(x)|2 > 0,∀x ∈ [a;b].
  • 31. 26 Viết đề tài giá sinh viên – ZALO:0973.287.149-TEAMLUANVAN.COM Chứng minh rang các so các nghi m của phương trình f (x) = 0 trên đoạn [a; b] là hữu hạn. L i giai. Giả sử phương trình f(x) = 0 có vô so nghi m {xn} ⊂ [a;b] vói n = 1,2,... Vì {xn} là dãy bị ch n nên ton tại dãy con {xnk } → α ∈ [a;b]. Do f (x) liên tục, nên f(α) = 0. Từ giả thiet và vì f(α) = 0, nên f′(α) /= 0. Nhưng f ′(α) = lim f (x) − f (α) /= 0. x→α x−α Đieu này chứng tỏ f(x) 0 trong m t lân c n nào đó của điem α. Đieu này lại mâu thuȁn vói giả thiet α là điem tụ của dãy {xn}. V y so các nghi m của phương trình f(x) = 0 trên đoạn [a;b] là hữu hạn.
  • 32. 27 Viết đề tài giá sinh viên – ZALO:0973.287.149-TEAMLUANVAN.COM Chương 2 Bat đang th c và c c trị trong lỚp hàm mũ và hàm hyperbolic 2.1 Bat đang th c trong lỚp hàm mũ và hàm hyperbolic Trong phan này ta sử dụng ký hi u I đe chỉ m t trong các khoảng sau: [a,b], [a,b), (a,b], (a,b) vói a < b. Trưóc het, ta xét m t so bat đȁng thức cơ bản đoi vói hàm so. Định lí 2.1. Giả sử f(x) là m t hàm loi trên khoảng I và λ1,λ2,λ3,...,λn là dãy các so không âm thỏa mãn đieu ki n λ1 + λ2 + λ3 +···+ λn = 1. Khi đó f (λ1x1 + λ2x2 + · ·· + λnxn) ≤λ1 f (x1) + λ2 f (x2) + ··· + λn f (xn), ∀x1,x2,...,xn ∈ I. H qua 2.1. Neu f (x) là m t hàm loi trên khoảng I thì f x1 + x2 + .... + xn ≤ f (x1) + f (x2) + .... + f (xn) , n n ∀x1,x2, .......,xn ∈ I.
  • 33. 28 Viết đề tài giá sinh viên – ZALO:0973.287.149-TEAMLUANVAN.COM / 1 x Định nghĩa 2.1. Vói các so dương a,b ta định nghĩa trung bình logarit của các so a,b là bieu thức Nh n xét rang LM(a,b) = b− a , a = b, L(a,a) = a. ln b − ln a LM(a,b) = 1 = 1 . 1 ∫ b dx M 1 , (a,b) Ký hi u: Ve sau, ta sử dụng các ký hi u đoi vói các đại lưong trung bình cho hai so dương như sau - Trung bình đieu hòa (HM(a,b)), - Trung bình nhân (GM(a,b)), - Trung bình logarit (LM(a,b)), - Trung bình c ng (AM(a,b)). Ta có Định lí 2.2. Vói các so dương a < b ta luôn có 2ab < √ ab < 1 < a+b a+b 1 ∫ b dx 2 hay b− a a x HM(a,b) < GM(a, b) < LM(a, b) < AM(a, b). (2.1) ChÝng minh. Giả sử 0 < a < b xét hàm loi f (x) = ex trên trên đoạn [ln a,ln b]. Ta có bat đȁng thức ln a+ln b e 2 (ln b− ln a) < ln b ex dx ln a ln a + ln b 2 (ln b − ln a). x a b − a ∫ <
  • 34. 29 Viết đề tài giá sinh viên – ZALO:0973.287.149-TEAMLUANVAN.COM ⇔ a1/3 +b1/3 !3 ) = 8 ( )+ 8 3 )+ 8 3 Từ đây suy ra √ ab < b− a < a + b ⇔ GM(a,b) < LM(a,b) < AM(a,b). Ta có ln b−ln a 2 V y nên 2ab < √ ab HM(a, b) < GM(a, b). a+b HM(a,b) < GM(a,b) < LM(a,b) < AM(a,b), a,b > 0,a /= b. Định lí 2.3. Vói 0 < a < b ta có các bat đȁng thức LM(a,b) < 2 ChÝng minh. Ta có , LM(a3 ,b3 ) < AM3 (a,b). (2.2) ∫ d f x dx 1 f c d − c 3 2c+d f d − c 3 c+2d f d − c trong đó c < η < d. 1 f(c)(d 8 —c) − (c − d)5 f (4) 6480 Cho c = ln a,d = ln b, f (x) = ex , ta thu đưoc 2ln a+ln b ln a+2ln b b− a = ln b ex dx ln a eln a + 3e 3 +3e 8 3 + eln b .(ln b− ln a) a1/3 +b1/3 !3 Từ đây suy ra bat đȁng thức thứ nhat trong (2.2). Trong bat đȁng thức thứ nhat, thay a,b tương ứng bỏi a3 ,b3 ta thu đưoc bat đȁng ∫ c ( )( ( ( )+ = 2 (ln b−ln a). (η), <
  • 35. 30 Viết đề tài giá sinh viên – ZALO:0973.287.149-TEAMLUANVAN.COM 2 2 ⇔ thức thứ hai. Bài toán 2.1. Ta có h thức L i giai. Ta có LM(x2 , y2 ) LM(x,y) = AM(x,y). (2.3) L x2 y2 x2 − y2 (x−y)(x+y) L x y x+y L x y A x y ( , ) = ln x2 − ln y2 = 2(ln x − ln y) = ( , ) 2 = ( , ) ( , ). Từ đây suy ra đieu phải chứng minh. Bài toán 2.2. Giả sử b > a > 0. Khi đó ta luôn có bat đȁng thức LM(a,b) < AM(a,b) + GM(a,b) . (2.4) 2 L i giai. Ta có a1/3 +b1/3 !3 a1/2 +b1/2 !2 ≤ √ √ b 2 ⇔ 3( √ 3 a2b+ √ 3 a2b) ≤ a+b+4 √ ab. (2.5) Th t v y, áp dụng bat đȁng thức AM-GM cho ba so dương ta có 3 √ 3 a2b = 3 q 3 3 √ 3 ab2 = 3 q 3 C ng các ve tương ứng, ta đưoc a. √ ab. √ ab ≤ a + √ ab + √ ab. b. √ ab. √ ab ≤ b + √ ab + √ ab. 3( √ 3 a2b+ √ 3 ab2) ≤ a+b+4 √ ab. 2 2 <
  • 36. 31 Viết đề tài giá sinh viên – ZALO:0973.287.149-TEAMLUANVAN.COM 2 2 = 2 . ln b − ln a < 2 M t khác a1/2 +b1/2 !2 √ a+ √ b 2 a+b + √ ab AM + GM Suy ra LM < AM + GM , đieu phải chứng minh. 2 Bài toán 2.3. Chứng minh bat đȁng thức 1 < sinhx < coshx, x > 0. x L i giai. Ta có AM(ex ,e−x ) = cosh x, GM(ex , e−x ) = 1, LM(ex , e−x ) = sinh x . x Vói x = / 0, ex /= e−x . Áp dụng bat đȁng thức GM < LM < AM, ta có đieu phải chứng minh. Bài toán 2.4. Chứng minh bat đȁng thức L i giai. Ta có sinh x x < cosh x + 1 2 , x > 0. A(ex ,e−x ) = coshx, G(ex ,e−x ) = 1, L(ex , e−x ) = sinhx . x Vói x > 0, ex = / phải chứng minh. e−x ta áp dụng bat đȁng thức LM < AM + GM , ta thu đưoc đieu 2 Bài toán 2.5. Cho b > a > 0, chứng minh bat đȁng thức b2 − a2 b+ a 2 2 2 2 = = .
  • 37. 32 Viết đề tài giá sinh viên – ZALO:0973.287.149-TEAMLUANVAN.COM − b a b a 2 2 b − a ) = b− a a (− ) = − b − a aa − ) ln − —1 = −ln − aa e aa L i giai. Theo đȁng thức (2.3) và bat đȁng thức (2.1), ta có LM(a2 ,b2 ) = LM(a,b)AM(a,b) < AM2 (a,b). Từ đó suy ra bat đȁng thức can phải chứng minh. Tiep theo, ta xét m t loại trung bình nữa, đó là trung bình đong nhat (Identric) và các bat đȁng thức gan vói hàm mũ liên quan. Định nghĩa 2.2. Vói các so dương a,b trung bình đong nhat đưoc các đinh bỏi h thức IM(a,b) = a neu a = b, 1 bb (b 1 a) e aa neu a b. Bài toán 2.6. Vói 0 < a < b, chứng minh bat đȁng thức GM(a,b) < IM(a,b) < AM(a, b). (2.6) L i giai. Áp dụng bat đȁng thức đoi vói hàm loi: f a+b ≤ 1 ∫ b f x dx ≤ f (a) + f (b) 2 vói f(x) = −ln x, ta có ( ) b−a a 2 f a+b = −ln a+ b = −ln AM(a,b), f (a) + f (b) = −ln a −ln b = − ln ab = −ln √ ab = −ln GM(a,b), 2 1 ∫ b f x dx 2 1 ∫ b 2 ln x dx 1 h ln bb b a i , hbb i 1 , 1hbb i 1 a ( —( = − = −ln IM(a,b).
  • 38. 33 Viết đề tài giá sinh viên – ZALO:0973.287.149-TEAMLUANVAN.COM x a = ( ) b − a a ) Như v y, ta có −ln AM(a,b) < −ln IM(a,b) < −ln GM(a,b) ⇔ AM(a,b) > IM(a,b) > GM(a,b). Đe chứng minh bat đȁng thức (2.10) chúng ta can các bo đe sau đây: Bài toán 2.7. Vói 0 < a < b, chứng minh bat đȁng thức ln x ≤ x− 1, x > 0. (2.7) L i giai. Xét hàm so f (x) = x −ln x − 1 trên mien x > 0. Ta có f ′(x) = 1− 1 = 0 ⇔ x = 1. De thay f ′(x) đoi dau từ âm sang dương khi đi qua x = 1. Do đó x = 1 là điem cục tieu duy nhat. Nhưng, f (1) = 0. v y ta có f (x) ≥ 0, ∀x > 0. Dau đȁng thức xảyra khi và chỉ khi x = 1. V y bat đȁng thức (2.7) đưoc chứng minh. Bài toán 2.8. Giả sử f (x) là hàm khả tong dương trên khoảng (a, b), a > 0. Chứng minh bat đȁng thức Hf < Gf < Af , (2.8) trong đó A 1 ∫ b f x dx G e 1 ∫ b ln f (t)dt H b − a (2.9) = b− a a ( ) , f = b−a a , f = b . dx/ f (x) a L i giai. Trong bat đȁng thức (2.7), lay x = (b − a) f (t) , ∫ b f (τ)dτ ln x ln f t −ln 1 ∫ b f τ dτ f ∫ ( .
  • 39. 34 Viết đề tài giá sinh viên – ZALO:0973.287.149-TEAMLUANVAN.COM − a . Theo bat đȁng thức (2.7), ta có ln f(t) — ln 1 ∫ b f (τ) dτ ≤ (b− a) f (t) − 1. b− a a ∫ b f (τ)dτ Lay tích phân theo t trên (a,b) bieu thức trên, ta có ∫ b 1 ∫ b (b−a) ∫ b f (t)dt a ln f (t)dt − (b − a)ln b a f (τ)dτ ≤ ∫ b —(b − a) = 0. Từ đây suy ra ve phải của (2.8). Áp dụng bat đȁng thức này đoi vói 1 f thay cho f vói chú ý rang ta nh n đưoc ve trái của (2.8). ln 1 = ln f(t) f(t) Bài toán 2.9. Vói 0 < a < b, chứng minh bat đȁng thức L i giai. Vói f(x 1 ) = x , LM(a, b) < IM(a,b) < AM(a, b). (2.10) ta có A a b 1 ∫ b dx ln b − ln a 1 f ( , ) = b−a a x = 1 ∫ b b− a = L(a, b) , Gf (a,b) = eb−a a (−ln x)x = e−ln IM(a,b) = 1 IM(a,b) Suy ra LM(a, b) < IM(a,b). Áp dụng bat đȁng thức Gf < Af vói f (x) = x ta có IM(a,b) < AM(a,b). Như v y ta có a f (τ)dτ a − a
  • 40. 35 Viết đề tài giá sinh viên – ZALO:0973.287.149-TEAMLUANVAN.COM 2 2 H qua 2.2. Vói các so dương a,b có các bat đȁng thức kép HM(a, b) ≤ GM(a, b) ≤ LM(a, b) ≤ AM(a, b) + GM(a, b) ≤ IM(a, b) ≤ AM(a, b). (2.11) ChÝng minh. Ta chỉ can chứng minh bat đȁng thức trung gian giữa LM và IM. Ta có LM(a,b) ≤ AM(a,b)+ GM(a,b) ≤ IM(a,b) ⇔ AM(a, b) + GM(a, b) ≤ 2IM(a, b) ≤ 2AM(a, b) ⇔ GM(a,b) ≤ AM(a, b). Trong thục tien, có rat nhieu bài toán không liên quan trục tiep đen các hàm mũ và hàm hyperbolic, nhưng chúng có những moi quan h m t thiet vói nhau, có the sử dụng các hàm hyperbolic như là phương pháp giải hi u quả. Bài toán 2.10. Cho a,b,c > 0 thỏa mãn đieu ki n a+ √ a2 −1 b+ √ b2 −1 c+ √ c2 −1 = 10. Tìm giá trị nhỏ nhat của P = 4a3 +4b3 +4b3 − 3a−3b−3c. L i giai. Theo giả thiet, ta có a,b,c ≥ 1 nên có the đ t a = coshx,b = coshy,c = coshz, x,y,z ≥ 0. Khi đó ta tìm đưoc x = ln a+ √ a2 −1 ;y = ln b+ √ b2 −1 ;z = ln c+ √ c2 −1 . Do đó ta có the viet giả thiet dưói dạng x + y + z = ln 10. Sử dụng tính chat 1.9, ta thu đưoc P = cosh3x + cosh3y + cosh3z. Ta lại thay (cosh(3x))′′ = 9.cosh(3x) > 0, ∀x nên cosh(3x) là hàm loi trên R.
  • 41. 36 Viết đề tài giá sinh viên – ZALO:0973.287.149-TEAMLUANVAN.COM 3 / ∀ GM LM Áp dụng bat đȁng thức Jensen, ta đưoc P = cosh3x + cosh3y + cosh3z ≥ 3cosh 3x + 3y + 3z hay P eln 10 + e−ln 10 303 ≥ 3cosh(ln 10) = 3. Dau đȁng thức xảy ra khi 2 = 200 . a = b = c = √ 3 10+ √ 3 10−1 2 . Bài toán 2.11. Chứng minh bat đȁng thức 1 < sinh x < cosh x, x = 0. x L i giai. Ta có A(ex ,e−x ) = cosh x, GM(ex , e−x ) = 1, LM(ex ,e−x ) = sinhx . x Vói x = / 0, ex /= e−x , áp dụng bat đȁng thức GM < LM < AM, ta có ngay đieu phải chứng minh. Bài toán 2.12. Chứng minh bat đȁng thức L i giai. Ta có cosh x + x > 2, x 0. sinhx AM(ex ,e−x ) = cosh x, GM(ex , e−x ) = 1, LM(ex , e−x ) = sinh x . x Vói x 0, ex e−x . Áp dụng bat đȁng thức AM − GM ≥ LM − GM ,
  • 42. 37 Viết đề tài giá sinh viên – ZALO:0973.287.149-TEAMLUANVAN.COM ≥ ∀ ∈ 2n ta thu đưoc đieu phải chứng minh. Bài toán 2.13. Chứng minh bat đȁng thức L i giai. Ta có sinh x x < coshx+3coshx/3 4 , ∀x 0. (2.12) LM(ex ,e−x ) = sinhx , M1 (ex , e−x ) = cosh x + 3 cosh x/3 . x /3 4 Sử dụng bat đȁng thức LM ≤ M1/3 ta đưoc bat đȁng thức (2.12). Bài toán 2.14. Cho a < b. Chứng minh rang ton tại c ∈ (a;b) sao cho 2 2 a− c < sinhc < b − c . L i giai. Đau tiên ta thay neu a < 0 < b thì sinhx đoi dau trên (a;b) vì sinh0 = 0. Tức là ton tại x1 < 0,x2 > 0 sao cho sinhx1 ≥ 0,sinh x2 ≤ 0. Khi đó, chỉ can chon c = 0, ta đưoc đieu can chứng minh. Bài toán 2.15. Cho a < b. Chứng minh rang ton tại c ∈ (a;b) sao cho 2 2 a− c < coshc < b− c . L i giai. Vì coshx < 0 không đoi dau trên (a;b). Ta giả sử phản chứng rang vói moi x ∈ (a;b), ta có coshx 2 , x (a;b). b − x Xét dãy xn = b− b− a , ∀n ∈ N. Ket hop vói định lí giá trị trung bình Lagrange, ta thay ton tại giá trị c1 ∈ (a;x1) sao cho sinhx1 − sinha = sinh a+ b − sinha = coshc1 b− a ≥ 2 . b −a ≥ 1. 2 2 b−c1 2
  • 43. 38 Viết đề tài giá sinh viên – ZALO:0973.287.149-TEAMLUANVAN.COM ≤ . − . t t Cứ tiep tục quá trình trên, ta xây dụng dãy (cn)n≥1 sao cho cn+1 ∈ (xn;xn+1) và sinhxn 1 − sinhxn = coshcn 1(xn 1 − xn) ≥ 2 b− a ≥ 1. + + + b − cn+1 2n+1 Do đó sinhxn − sinha ≥ n, ∀n ∈ N∗ . Đieu này cho thay sinhx là hàm không bị ch n trên (a;b) (mâu thȁn vói tính chat hàm so sinhx là hàm liên tục khả vi . Do đó giả sử là sai. V y ton tại giá trị c ∈ (a;b) sao cho coshc = sinh′ c < 2 . Chứng minh tương tụ, ta thay ton tại giá trị c ∈ (a;b) sao cho 2 a − c < coshc. b − c V y ta có đieu phải chứng minh. Bài toán 2.16. Giả sử |coshx −coshy| ≤ M|x − y|, ∀x,y ∈ (0,1). Chứng minh rang |coshx| ≤ M, ∀x,y ∈ (0,1). L i giai. Xét hàm so f (x) = coshx. Co định x ∈ (0;1) và xét t /= 0, sao cho x +t ∈ (0;1). Theo giả thiet, ta có | f′ (x)| ≤ . f (x +t) f (x) .+ . f (x+t) f (x) − f ′ (x). M + f(x+t)− f(x) f′ (x) . t Chuyen qua giói hạn khi t → 0, ta có bat đȁng thức | f ′(x)| ≤ M hay |coshx| ≤ M, ∀x,y ∈ (0,1). Bài toán 2.17. Cho hàm so f (x) = e−x2 . Chứng minh rang L i giai. Ta có max x∈R f′′ (x)| ≥ e1/2 4 . f ′(x) = −2xe−x2 |
  • 44. 39 Viết đề tài giá sinh viên – ZALO:0973.287.149-TEAMLUANVAN.COM | | ≤ ∀ ∈ R x∈R x∈R |f ′(x)| ≤ f 2+ 2 = √ 2e1/2 , ∀x ∈ R. Dau đȁng thức xảy ra khi x = 2+ 2 . x∈R x∈R và f′′(x) = e−x2 (4x2 −1). Nh n xét rang | f (x)| ≤ 1, ∀x ∈ R. Dau đȁng thức xảy ra khi x = 0. Tương tụ, f ′(x) f 1 , x . Dau đȁng thức xảy ra khi x 2 Áp dụng bat đȁng thức Landau, ta có 1 = 2 . max|f′(x)| ≤ 2 q max|f(x)|M2, trong đó M2 = max x∈R f′′ (x)|. Suy ra M2 ≥ e1/2 4 , đieu phải chứng minh. Bài toán 2.18. Cho hàm so f(x) = e−x2+2x . Chứng minh rang max| f ′′(x)| ≥ e . L i giai. Ta có x∈R 2 f ′(x) = 2(1− x)e−x2+2x và f ′′(x) = 2e−x2+2x (2x2 −4x+1). Nh n xét rang √|f(x)| ≤ 1, ∀x ∈ R. Dau đȁng thức xảy ra khi x = 1. Tư√ ơng tụ, 2 2 Áp dụng bat đȁng thức Landau, ta có max|f′(x)| ≤ 2 q max|f(x)|M2, trong đó M2 = max| f ′′(x)|. Suy ra M2 ≥ e , đieu phải chứng minh. x∈R 2 |
  • 45. 40 Viết đề tài giá sinh viên – ZALO:0973.287.149-TEAMLUANVAN.COM ≥ 3C 2 2 cosh − = 1 ⇔ A = B = C = 2.1.2. Bat đang th c trong tam giác trong lỚp hàm hyperbolic Bài toán 2.19. Chứng minh rang trong tam giác ABC ta luôn có sinh 3A+sinh 3B + sinh 3C ≥ 3 sinhπ. L i giai. Ta có the giải bài toán 1.1 theo hai cách như sau: Cách 1: Bien đoi trục tiep dụa vào tính chat coshx ≥ 1. ∀x ∈ R sinh 3A+sinh 3B + sinh 3C+sinh π = 2sinh 3A + 3B cosh 3A − 3B + 2sinh 3C + π cosh 3C − π ≥ 2sinh 2 3A + 3B 2 + 2sinh 2 2 2 3C+π 2 = 4sinh 3A + 3B + 3C + π cosh 3A+ 3B − 3C − π Suy ra ≥ 4sinh 4 4 3A + 3B + 3C + π 4 = 4sinhπ. sinh3A+sinh 3B + sinh 3C ≥ 3sinh π. Dau đȁng thức xảy ra khi và chỉ khi cosh 3A−3B = 1 cosh 3A+3B− 3C −π = 1 Cách 2: Xét hàm so y = sinhx. Do (sinhx)′′ = sinhx > 0, ∀x > 0 nên hàm so sinhx loi trên (0;+∞) nên sinh3A+sinh 3B + sinh3C 3sinh 3A + 3B + 3C 3 ⇔ sinh3A+sinh3B + sinh3C ≥ 3sinh π. π π 3 . 4
  • 46. 41 Viết đề tài giá sinh viên – ZALO:0973.287.149-TEAMLUANVAN.COM ≥ ≥ ≥ cosh2 x ≥ 2 2 2 2 3 Dau đȁng thức xảy ra khi và chỉ khi A = B = C = π . 3 Ta có the tong quát thành bài toán sau: Chứng minh rang trong tam giác ABC ta luôn có sinhA+sinh B + sinhC 3sinh kπ ,k > 0. 3 Bài toán 2.20. Chứng minh rang trong tam giác ABC ta luôn có coshkA + coshkB + coshkC 3coshk π ,k > 0. 3 L i giai. Xét hàm so y = coshx Do (coshx)′′ = coshx ≥ 1, ∀x ∈ R coshx loi trên R nên coshkA + cosh kA + cosh kA ≥ 3 cosh k A + B +C 3 π ⇔ coshkA + coshkA + coshkA ≥ 3coshk 3 . Dau đȁng thức xảy ra khi và chỉ khi A = B = C = π . 3 Bài toán 2.21. Chứng minh rang trong tam giác ABC ta luôn có tanhkA + tanhkA + tanhkA 3tanhk π ,k > 0. 3 L i giai. Xét hàm so y = tanhx. Do (tanhx)′′ = 2tanhx ≥ 0,∀x > 0 suy ra y = tanhx loi trên (0;+∞) nên tanhkA + tanhkA + tanhkA 3tanhk A + B+C , k > 0 3 π ⇔ tanhkA + tanhkA + tanhkA ≥ 3tanhk 3 . Dau đȁng thức xảy ra khi và chỉ khi A = B = C = π . 3 Bài toán 2.22. Chứng minh rang trong tam giác ABC ta luôn có sinh2 A + sinh2 B + sinh2C ≥ 3 cosh π − 1 .
  • 47. 42 Viết đề tài giá sinh viên – ZALO:0973.287.149-TEAMLUANVAN.COM ≥ ≥ 2 2 2 2 3 cosh2 A +cosh2 B +cosh2C ≥ 3 cosh π +1 . L i giai. Ta có công thức: sinh2 x = coshx−1 . 2 2 Suy ra sinh2 A + sinh2 B + sinh2C = coshA − 1 + coshB − 1 + coshC − 1 2 2 2 2 2 2 cosh A + coshB + coshC 3 = 2 − 2 . Áp dụng bat đȁng thức Jensen, ta đưoc coshA + coshB + coshC 3cosh π . 3 Suy ra sinh2 A + sinh2 B + sinh2C ≥ 3 cosh π − 1 . Bài toán 2.23. Chứng minh rang trong tam giác ABC ta luôn có 2 2 L i giai. Ta có công thức 2 2 3 cosh2 x = coshx + 1 2 2 nên cosh2 A + cosh2 B + cosh2C = coshA + 1 + cosh B + 1 + coshC + 1 2 2 2 2 2 2 cosh A + coshB + coshC 3 = 2 + 2 . Áp dụng bat đȁng thức Jensen, ta đưoc coshA + coshB + coshC 3cosh π . 3 Suy ra
  • 48. 43 Viết đề tài giá sinh viên – ZALO:0973.287.149-TEAMLUANVAN.COM 2 2 2 2 3 1 2 2 cosh2 A +cosh2 B +cosh2C ≥ 3 cosh π +1 . Bài toán 2.24. Chứng minh rang trong tam giác ABC ta luôn có sinhA+sinhB + sinhC ≥ sinh π − A + sinh π − B + sinh π −C . 2 2 2 L i giai. Ta có sinhA+sinhB = 2sinh A + B cosh A − B ≥ 2sinh π −C , 2 2 2 sinhB + sinhC = 2sinh B +C cosh B −C ≥ 2sinh π − A , 2 2 2 sinhC+sinhA = 2sinh C + A cosh C − A ≥ 2sinh π − B . 2 2 2 Do đó sinhA+sinhB + sinhC ≥ sinh π − A + sinh π − B + sinh π −C . 2 2 2 Dau đȁng thức xảy ra khi và chỉ khi tam giác ABC đeu. Bài toán 2.25. Chứng minh rang trong tam giác ABC ta luôn có sinh A + sinh B + sinh C ≥ tanh A + tanh B + tanh C . 2 2 2 2 2 2 L i giai. Xét hàm so y = sinh x − tanh x , ∀x ≥ 0. Do y′ 1 cosh x 1 1 nên y′′ 1 sinh x sinh x 2 0 x 0 = 2 2 − 2 cosh2 x , 2 = 4 2 + 2 cosh3 x ≥ 2 , ∀ ≥ . Suy ra y(x) x ≥ y(0)+y′(0).(x−0), ∀x ≥ 0. Do đó sinh 2 x —tanh 2 ≥ 0, ∀x ≥ 0. Thay x lan lưot bỏi A, B, C vào bat đȁng thức roi c ng theo ve, ta đưoc đieu phải
  • 49. 44 Viết đề tài giá sinh viên – ZALO:0973.287.149-TEAMLUANVAN.COM ≤ A B C s . Bài toán 2.26. Chứng minh rang trong tam giác ABC ta luôn có 3(sinhA + sinh B + sinhC) ≥ π sinhA + sinhB + sinhC . A B C L i giai. Xét hàm so y = x − tanhx. Ta có . . . ? Do đó hàm so y = x − tanhx là hàm so đong bien trên R nên y ≥ y(0), ∀x ≥ 0. Suy ra x− tanhx ≥ 0,∀x ≥ 0 nên x ≥ tanhx,∀x ≥ 0. Xét hàm so y= sinhx . Ta có Do đó hàm so y= x sinhx là hàm so đong bien (0;+∞) x Giả sử A ≥ B ≥ C ta có sinhA A ≥ sinhB B ≥ sinhC C . Theo bat đȁng thức Chebyshev, ta có (A + B +C) sinhA + sinhB + sinhC ≤ 3(sinhA + sinhB + sinhC). A B C Từ đó suy ra 3(sinhA + sinh B + sinhC) ≥ π sinhA + sinhB + sinhC . Dau “=” xảy ra khi và chỉ khi tam giác ABC là tam giác đeu. Bài toán 2.27. Chứng minh rang trong tam giác ABC ta luôn có sinhAsinh BsinhC sinh3 π . 3 L i giai. Xét hàm so y= ln (sinhx), ∀x > 0. Ta có chứng minh inh A + sinh B + sinh C ≥ tanh A + tanh B + tanh C 2 2 2 2 2 2
  • 50. 45 Viết đề tài giá sinh viên – ZALO:0973.287.149-TEAMLUANVAN.COM 3 3 ≤ ≥ 3 3 ≤ 3 3 Do đó hàm so y= ln (sinhx) là hàm so lõm trên (0;+∞) nên ta có bat đȁng thức ln (sinhA)+ln (sinhB)+ln (sinhC) ≤ 3ln sinh A+B+C . Suy ra ln (sinhAsinhBsinhC) ≤ ln sinh3 π . Vì hàm so y= ln x là hàm so đong bien trên (0;+∞) nên ta có đieu phải chứng minh sinhAsinh BsinhC sinh3 π . 3 Dau đȁng thức xảy ra khi và chỉ khi tam giác ABC đeu. Bài toán 2.28. Chứng minh rang trong tam giác ABC ta luôn có coshAcosh BcoshC cosh3 π . 3 L i giai. Xét hàm so y= ln (coshx), ∀x > 0. Ta có Do đó hàm so y= ln (coshx) là hàm so loi trên (0;+∞) nên ta có bat đȁng thức ln (coshA)+ln (coshB)+ln (coshC) ≤ 3ln cosh A+B+C . Suy ra ln (coshAcoshBcoshC) ≥ ln cosh3 π . Vì hàm so y= ln x là hàm so đong bien trên (0;+∞) nên ta có đieu phải chứng minh coshAcosh BcoshC cosh3 π . 3 Dau đȁng thức xảy ra khi và chỉ khi tam giác ABC đeu. Bài toán 2.29. Chứng minh rang trong tam giác ABC ta luôn có sinhA +sinhB+ sinhC − tanhA− tanhB− tanhC ≥ 3 sinh π − tanh π .
  • 51. 46 Viết đề tài giá sinh viên – ZALO:0973.287.149-TEAMLUANVAN.COM cosh2 x sinhA +sinhB+ sinhC − tanhA− tanhB− tanhC ≥ 3 sinh π − tanh π . L i giai. Xét hàm so y = sinhx − tanhx, ∀x > 0. Ta có y′ = coshx − 1 ,y′′ = sinhx + sinhx cosh3 x > 0, ∀x > 0. Do đó hàm so y = sinhx − tanhx là hàm so loi trên (0;+∞) nên ta có bat đȁng thức (sinhA − tanh A) + (sinh B − tanh B) + (sinhC − tanhC) 3 ≥ sinh A+B+C − tanh A+ B+C . 3 3 Suy ra 3 3 Dau đȁng thức xảy ra khi và chỉ khi tam giác ABC đeu. Bài toán 2.30. Chứng minh rang trong tam giác ABC ta luôn có sinh2 A +sinh 2 L i giai. Ta có công thức 2 B + sinh 2 2C > 2 A2 + B2 +C2 4 . sinh2 x = coshx− 1 2 2 nên bat đȁng thức can chứng minh tương đương vói coshA+ coshB + coshC − 3 ≥ A2 + B2 +C2 2 . Xét hàm so y = 2coshx−x2 , ∀x ≥ 0. Ta có y′ = 2sinhx−2x,y′′ = 2coshx−1 ≥ 0, ∀x ≥ 0. Suy ra y(x) > y(0)+y′(0).(x−0), ∀x > 0 nên 2coshx−x2 > 2, ∀x > 0. Lan lưot thay x bỏi A, B, C vào bat đȁng thức trên ta đưoc roi c ng từng ve các bat đȁng thức đó, ta đưoc 2coshA − A2 + 2coshB− B2 + 2coshC −C2 > 6.
  • 52. 47 Viết đề tài giá sinh viên – ZALO:0973.287.149-TEAMLUANVAN.COM Suy ra coshA + coshB +coshC − 3 > A2 + B2 +C2 2 . Bài toán 2.31. Chứng minh bat đȁng thức ln sinhx > 1 xcoshx −sinhx , x > 0 (2.13) x 2 sinhx L i giai. Đe chứng minh bat đȁng thức (2.13) có the sử dụng bat đȁng thức của các đại lưong trung bình. Ta có LM(ex ,e−x ) = sinhx , GM(ex ,e−x ) = 1, IM(ex ,e−x ) = excothx−1 . (2.14) x Lay logarit hai vé của (2.14), ta có bat đȁng thức (2.13). 2.2 Các dạng toán c c trị sinh bƠi hàm mũ và hyperbolic 2.2.1. Các dạng toán c c trị cơ ban Trong mục này trình bày m t so kien thức cơ bản tìm cục trị hàm so. Định nghĩa 2.3 (xem [1]). Giả sử hàm so f xác định trên t p hop D (D ⊂ R) và x0 ∈ R. a) x0 đưoc goi là m t điem cục đại của hàm so f neu ton tại m t khoảng (a; b) chứa điem x0 sao cho (a; b) ⊂ D và f (x) < f (x0) vói moi x ∈ (a; b) {x0}. Khi đó f(x0) đưoc goi là giá trị cục đại của hàm so f. b) x0 đưoc goi là m t điem cục tieu của hàm so f neu ton tại m t khoảng (a; b) chứa điem x0 sao cho(a;b) ⊂ D và f (x) > f (x0) vói moi x ∈ (a;b) {x0}. Khi đó f (x0) đưoc goi là giá trị cục tieu của hàm so f . Điem cục đại và điem cục tieu goi chung là điem cục trị. Giá trị cục đại và giá trị cục tieu đưoc goi chung là cục trị. Định lí 2.4 (Đieu ki n can đe hàm so đạt cục trị, xem [1]). Giả sử hàm so f đạt cục trị tại điem x0 . Khi đó, neu f có đạo hàm tại x0 thì f′(x0) = 0.
  • 53. 48 Viết đề tài giá sinh viên – ZALO:0973.287.149-TEAMLUANVAN.COM 3 ≤ 3 x Định lí 2.5 (Đieu ki n đủ đe hàm so đạt cục trị, xem [1]). Giả sử hàm so f liên tục trên khoảng (a;b)chứa điem x0 và có đạo hàm trên khoảng (a;x0) và (x0;b). Khi đó a) Neu f′(x0) < 0 vói moi x ∈ (a;x0) và f ′(x0) > 0 vói moi x ∈ (x0;b) thì hàm so đạt cục tieu tại điem x0. b) Neu f′(x0) > 0 vói moi x ∈ (a;x0) và f′(x0) < 0 vói moi x ∈ (x0;b) thì hàm so đạt cục đại tại điem x0. Định lí 2.6 (xem [1]). Giả sử hàm so f có đạo hàm cap m t trên khoảng (a;b) chứa điem x0, f′(x0) = 0 và f có đạo hàm cap hai khác 0 tại điem x0. a) Neu f′′(x0) < 0 thì hàm so f đạt cục đại tại điem x0. b) Neu f′′(x0) > 0 thì hàm so f đạt cục tieu tại điem x0. Bài toán 2.32. Tìm giá trị nhỏ nhat của M = coshx+2 − √ 3 coshx, x /= 0. L i giai. Ta có AM(ex ,e−x ) = cosh x, GM(ex ,e−x ) = 1. (2.15) Sử dụng bat đȁng thức √ 3 AG2 AM +2GM 3 và (2.15) de dàng suy ra bat đȁng thức √ 3 coshx ≤ coshx+2 , ∀x ∈ R. (2.16) Từ đó suy ra M ≥ 0 và minM = 0 khi x = 0. Bài toán 2.33. Tìm giá trị nhỏ nhat của L i giai. sinhx 2t M = tanhx t , x /= 0.t > 0. x +
  • 54. 49 Viết đề tài giá sinh viên – ZALO:0973.287.149-TEAMLUANVAN.COM Vói x /= 0, ta có LM(ex ;e−x ) = sinhx , GM(ex ;e−x ) = 1, AM(ex ;e−x ) = coshx, LM tanhx = . x Từ đây, ta có chứng minh bat đȁng thức AM x (2.17) sinhx 2t tanhx t > 2, x ∈ R.t > 0. Dau đȁng thức không xảy ra xảy ra nên không có giá trị nhỏ nhat. Bài toán 2.34. Tìm giá trị nhỏ nhat của bieu thức M = excothx−1 ,x ∈ R. L i giai. Ta chứng minh bat đȁng thức 1 ≤ excothx−1 , x ∈ R. Ta có IM(ex ; e−x ) = excothx−1 ,GM(ex ;e−x ) = 1, AM(ex ; e−x ) = coshx. (2.18) V y nên ta có đieu phải chứng minh. Do đó minM = 1 khi x = 0. Bài toán 2.35. Tìm giá trị nhỏ nhat của bieu thức M = coshx − excothx−1 , x ∈ R. L i giai. Ta chứng minh bat đȁng thức excothx−1 ≤ coshx, x ∈ R. IM(ex ; e−x ) = excothx−1 ,GM(ex ;e−x ) = 1, AM(ex ; e−x ) = cosh x. (2.19) V y nên ta có đieu phải chứng minh. Do đó minM = 0 khi x = 0. x x +
  • 55. 50 Viết đề tài giá sinh viên – ZALO:0973.287.149-TEAMLUANVAN.COM Bài toán 2.36. Tìm giá trị nhỏ nhat của M = coshx + 3coshx/3 − e(xcothx−1)/2 < sinhx , x ∈ R. 4 x L i giai. Ta chứng minh bat đȁng thức e(xcothx−1)/2 < sinhx < coshx + 3coshx/3 , x ∈ R. x 4 Ta có LM(ex ;e−x ) = sinhx , I(ex ;e−x ) = excothx−1 , (2.20) x GM(ex ;e−x ) = 1, M1/3 (ex ;e−x ) = coshx+3coshx/3 4 (2.21) Chú ý (2.19), áp dụng bat đȁng thức LM ≤ M1/3 đoi vói c p đoi so, ta có đieu phải chứng minh. V y nên minM = 0 khi x = 0. Bài toán 2.37. Tìm giá trị lón nhat của M = cosh2 x −2e2(xcothx−1), x ∈ R. L i giai. Ta chứng minh bat đȁng thức cosh2 x + 1 ≤ 2e2(xcothx−1), x ∈ R. Ta có AM(ex ,e−x ) = coshx, GM(ex ,e−x ) = 1, IM(ex ,e−x ) = excothx−1 . Áp dụng bat đȁng thức AM2 + GM2 ≤ 2IM2
  • 56. 51 Viết đề tài giá sinh viên – ZALO:0973.287.149-TEAMLUANVAN.COM p đoi vói c p đoi so, ta có ngay đieu phải chứng minh. Từ đó suy ra M = cosh2 x − 2e2(xcothx−1) ≤ −1, x ∈ R. Do đó minM = −1 khi x = 0. Bài toán 2.38. Tìm giá trị lón nhat của M = p(p + 1)sinh2x − 2sinh(p + 1)x + 2(p2 − 1)sinhx, p > −1, x ∈ R. L i giai. Vói p = 0 ho c x = 0, de thay rang bat đȁng thức trỏ thành đȁng thức. Giả sử p /= 0(p + 1 > 0) và x 0. Khi đó ta có 0 < e−x < ex và AM(e−x ,ex ) = coshx, GM(e−x ,ex ) = 1, Lp (e−x ,ex ) = 1 sinh(p + 1)x . Áp dụng bat đȁng thức p LMp − GMp (p + 1) sinh x ta có GMp−1 ≥ p(AM − GM), pcoshx ≤ 1 sinh(p + 1)x + p − 1. (2.22) Chú ý rang (p + 1) sinh x 2sinhxcoshx = sinh2x, sinhaxsinhbx > 0, ∀x /= 0, ab > 0 từ (2.22) suy ra p(p+1)sinh2x ≤ 2sinh(p+1)x +2(p2 −1)sinhx, ∀p > −1, ∀x ∈ R. (2.23) V y maxM = 0 khi x = 0. Bài toán 2.39. Xét phương trình x2 1 −ax−√ 2 = 0 có các nghi m x1 và x2. Tìm giá trị nhỏ nhat của bieu thức
  • 57. 52 Viết đề tài giá sinh viên – ZALO:0973.287.149-TEAMLUANVAN.COM | | + x2 +x0 + x0 + x2 2 1 1 2 f (a) = (x1 −x2) + x1 −x2 + x1 − x2 . 1 L i giai. Vì x1.x2 = −√ 2 < 0 nên có the coi x1 < 0,x2 > 0. Đ t x0 = −x1 > 0. Khi đó 2 1 1 2 Sử dụng bat đȁng thức 1 x0 1 + x2 4 ≥ x0 + x2 vói x0 > 0,x2 > 0, ta đưoc f (a) ≥ (x2 +x0)2 + 4 2 x0 + x2 + (x0 + x2) Suy ra = 2(x2 + x0)2 + 16 (x0 + x2)2 + 8 ≥ 8 √ 2 + 8. x0 = x2 min f(a) = 8 √ 2+8. 8 1 1 1 khi 0. (x 0 +x2)2 = (x0 + x2)2 ⇔ x0 = x2 = √ 4 2 ⇔ x1 = −√ 4 2 ;x2 = √ 4 2 ⇔ a = Bài toán 2.40. Cho hàm so f (x) = e−x2−x . Goi M2 là giá trị lón nhat của | f ′′(x)| trên trục thục, tức Chứng minh rang M2 > e1/2 4 . M2 = max f′′(x) . x∈R L i giai. Ta có và f ′(x) = (−2x + 1)e−x2+x f′′(x) = e−x2 (4x2 −1). f (a) = (x2 +x0) .
  • 58. 53 Viết đề tài giá sinh viên – ZALO:0973.287.149-TEAMLUANVAN.COM 1 | | ≤ ∀ ∈ R | | | | ≤ ∀ ∈ √ x∈R x∈R Tương tụ, ta có | f ′(x)| ≤ f 2 + 2 = √ 2e1/2 , ∀x ∈ R. Dau đȁng thức xảy ra max|f′(x)| ≤ 2 q max|f(x)|M2, Nh n xét rang | f (x)| ≤ 1, ∀x ∈ R và dau đȁng thức xảy ra khi x = 0. Tương tụ, ta có f′(x) f , x . Dau đȁng thức xảy ra khi x 2 Áp dụng bat đȁng thức Landau, ta có max|f′(x)| ≤ 2 q max|f(x)|M2, 1 = 2 . trong đó M2 = đưoc đpcm. max x∈R f′′ (x)|. V y nên giá trị minM2 = e1/2 4 không đạt đưoc, ta thu Bài toán 2.41. Cho hàm so f(x) = e−x2+2x . Goi M2 là giá trị lón nhat của |f′′(x)| trên trục thục, tức Chứng minh rang M2 > e . 2 M2 = max f′′(x) . x∈R L i giai. Ta có và f ′(x) = 2(1 − x)e−x2+2x f ′′(x) = 2e−x2+2x (2x2 −4x+1). Nh n xét rang f (x) 1, x R và dau đȁng thức xảy ra khi x = 1. khi x = 2+ √ 2 2 2 . Áp dụng bat đȁng thức Landau, ta có x∈R trong đó M2 = max| f ′′(x)|. x∈R x∈R e V y nên giá trị M2 = 2 không đạt đưoc, ta thu đưoc đpcm. Bài toán 2.42. Xét tính đơn đi u của các hàm so sau: |
  • 59. 54 Viết đề tài giá sinh viên – ZALO:0973.287.149-TEAMLUANVAN.COM —∞ —1 —∞ +∞ ct +∞ ln 2 x a) f (x) = ln x − x; b) f (x) = e2x − x. L i giai. a) T p xác định: D = (0;+∞). Ta có f′(x) = 1 −1 = 1−x ; f′(x) = 0 ⇔ x = 1 ∈ D. x x Bảng bien thiên: x 0 1 +∞ f ′(x) + 0 − f (x) −∞ −1 −∞ V y hàm so đong bien trên khoảng (0;1) và nghịch bien trên khoảng (1;+∞). b) T p xác định: D = R. Ta có f ′(x) = 2e2x − 1, f ′(x) = 0 ⇔ e2x = 1 ⇔ x = 1 ln 1 = −ln √ 2. 2 2 2 Bảng bien thiên: x −∞ −ln √ 2 +∞ f ′(x) − 0 + f (x) ct V y hàm so nghịch bien trên khoảng (−∞;−ln √ 2) và đong bien trên khoảng (−ln √ 2;+∞). Bài toán 2.43. Tìm cục đại, cục tieu của hàm so f(x) = x . ln x L i giai. Đieu ki n: x > 0, x /= 1. Ta có f ′(x) = ln x − 1 ≥ 0 ⇔ ln x ≥ 1, ⇔ x ≥ e. Bảng bien thiên:
  • 60. 55 Viết đề tài giá sinh viên – ZALO:0973.287.149-TEAMLUANVAN.COM +∞ e +∞ —∞ loge e 0 → → xln 10 2 = e x 1 e +∞ f ′(x) − 0 + f (x) e V y hàm so chỉ có duy nhat m t cục tieu xct = e; yct = e. Chú ý. lim x 1+ lim x 1− lim x→0+ f (x) = +∞ vì tử so → 1, mȁu so → 0+, f (x) = −∞ vì tử so → 1, mȁu so → 0−,f (x) = 0 vì tử so → 0 , mȁu so → −∞. Từ bảng bien thiên có the hỏi thêm: 1) Phương trình x lnxx = m Có hai nghi m khi nào ?; Đáp so : m > e 2) Phương trình ln x = m Vô nghi m khi nào ?; Đáp so : 0 ≤ m < e Bài toán 2.44. Tìm cục đại, cục tieu của hàm so f(x) = logx trên R+ . L i giai. Ta có f′ (x) = x 1 − logx x loge− logx x2 x 0 nên x ≤ e. Bảng bien thiên: x 0 e +∞ f ′(x) + 0 − f (x) −∞ loge e 0 V y hàm so có duy nhat m t cục đại xcd = e; ycd = loge . ≥
  • 61. 56 Viết đề tài giá sinh viên – ZALO:0973.287.149-TEAMLUANVAN.COM +∞ 1 +∞ 0 1 3e 0 x2 x x2 Nh n xét 2.1. Có the hỏi thêm các bài toán xác định max, min của hàm so và khảo sát bat phương trình m > logx . x Bài toán 2.45. Tìm cục đại, cục tieu của hàm so f (x) = ex .ln x, x ∈ R+ . L i giai. Ta có f ′(x) = ex . 1 + ln x.ex = ex .( 1 + ln x) và ex > 0, ∀x ∈ R+ . 1 x x Xét h(x) = x + ln x; x > 0, khi đó h′(x) = − 1 + 1 = x −1 = 0, suy ra x = 1. Bảng bien thiên: x 0 1 +∞ h′(x) − 0 + h(x) 1 Từ bảng bien thiên suy ra h(x) ≥ 1. V y nên f (x) > 0 ∀x > 0, suy ra hàm so đã cho không có cục trị. Bài toán 2.46. Tìm cục trị của hàm so f(x) = x.e−3x trên R. L i giai. Ta có f′(x) = e−3x +x.e−3x .(−3) = e−3x .(1−3x) = 0 nên x Bảng bien thiên: x −∞ 1 e +∞ f ′(x) + 0 − f (x) 0 1 3e 0 V y hàm so có duy nhat m t điem cục tieu xct = 1 ; yct = 1 . 1 = 3 . 3 3e
  • 62. 57 Viết đề tài giá sinh viên – ZALO:0973.287.149-TEAMLUANVAN.COM 0 1 e2 0 1 0 ( ) = − — ≤ — − ≤ Bài toán 2.48. Tìm cục trị của hàm so f(x) = |x|.e−|x−1| trên R. Chú ý. Đe tính lim x→∓∞ f (x) = 0, ta sử dụng quy tac H’Lôpital. Bài toán 2.47. Tìm cục trị của hàm so f x ex +e−x 2 trên R. L i giai. Ta có f′ (x) = ex e−x 2 = 0 nên ex = e−x , suy ra x = 0. Cách 1: f′′ hàm so. (x) = ex +e−x 2 > 0, ∀x ∈ R nên x = 0 là hoành đ điem cục tieu của Cách 2: L p bảng bien thiên. KL: Hàm so có duy nhat m t điem cục tieu xct = 0; yct = 1. Chú ý có the sử dụng quy tac 1, quy tac 2 đeu giải đưoc. L i giai. Ta có f(x) = Suy ra x.ex−1 khi : x 0 x. ex−1 khi : 0 < x < 1 x. e1−x khi : x ≥ 1 f ′ (x) = 1. ex−1 x.ex−1 khi : x 0 ex−1 +x.ex−1 khi : 0 < x < 1 e1−x − x.e1−x khi : x ≥ 1 ex−1 .(−x− 1) khi : x ≤ 0 = Ta có bảng bien thiên: ex−1 .(x+1) khi : 0 < x < 1 ex−1 .(1− x) khi : x ≥ 1 x −∞ −1 0 1 +∞ f ′(x) + 0 − ǁ + ǁ − f (x) 0 1 e2 0 1 0
  • 63. 58 Viết đề tài giá sinh viên – ZALO:0973.287.149-TEAMLUANVAN.COM e2 V y hàm so có hai điem cục đại: xcd = −1 nên ycd = 1 , xcd = 1 nên ycd = 1 và hàm so có m t điem cục tieu: xct = 0 tức yct = 0. Nh n xét 2.2. Từ bảng bien thiên ta có the đ t các câu hỏi liên quan mang n i dung ứng dụng của các điem cục trị như sau. 1) Tìm a đe phương trình |x|.e−|x−1| = a có 4 nghi m phân bi t. 2) Tìm các đưòng ti m c n của hàm so tương ứng.
  • 64. 59 Viết đề tài giá sinh viên – ZALO:0973.287.149-TEAMLUANVAN.COM ⇔ ± − ∈ √ Chương 3 M t so dạng toán liên quan 3.1 Các phương trình đại so giai bang phương pháp hàm hy- perbolic Xét các phương trình cơ bản sau: sinhx = a ⇔ x = ln (a+ √ a2 +1), a ∈ R, coshx = a x = ln (a a2 1), a [1; +∞) , 1 1+ a tanhx = a ⇔ x = ln , a ∈ (−1;1), 2 1 − a cothx = a ⇔ x = 1 ln a+1 , a ∈ (−∞;−1) ∪(1;+∞). 2 a − 1 Nh n xét 3.1 (Phương trình b c 3). a. Xét phương trình dạng 4x3 − 3x = q. (3.1) Trưòng hop 1. Neu q < −1, thì đ t x = cost,t ∈ [0;π] , ta thu đưoc phương trình cos3t = q ⇔ t = ± 1 arccos q + k2π . 3 3 Từ đó ta tìm đưoc 3 nghi m t1,t2,t3 ∈ [0;π] và x ∈ {cost1,cost2,cost3}. Trưòng hop 2. Neu q > 1, sử dụng đạo hàm đe chứng minh phương trình có
  • 65. 60 Viết đề tài giá sinh viên – ZALO:0973.287.149-TEAMLUANVAN.COM 3 3 3 3 q √ q − √ nghi m duy nhat. Đ t q = cosh3t thì phương trình trỏ thành 4x3 − 3x = cosh3t ⇔ 4x3 −3x = 4cosh3 t − 3cosht. Suy ra phương trình có nghi m x = cosht. Ta có cosh3t = q ⇔ t = 1 ln (q± √ q2 −1). Suy ra x = cosh 1 ln (q ± √ q2 − 1) x = cosh 1 ln (q + √ q2 − 1) ⇔ x = cosh 1 ln (q− √ q2 −1) q 3 √ 2 q 3 √ −1 ⇔ x = x = q 3 q+ q q− √ q2 —1 + —1 + q 3 q+ q − 1 q− √ q2 −1 ⇔ x = 3 x = q 3 q+ q2 1+ 3 q− √ q2 −1+ q 3 q− q2 − 1 q+ √ q2 − 1 V y phương trình (3.1) có nghiêm duy nhat x = q 3 q+ √ q2 −1+ q 3 q− √ q2 −1. Trưòng hop 3. Neu q < −1 thì 4(−x)3 −3(−x) = −q. Đ t y = (−x), ta thu đưoc phương trình 4y3 − 3y = −q. Đây chính là trưòng hop đã xét ỏ trên. b. Xét phương trình dạng 4x3 + 3x = q. (3.2) 2 −1
  • 66. 61 Viết đề tài giá sinh viên – ZALO:0973.287.149-TEAMLUANVAN.COM sinh3t = q ⇔ t = 1 ln (q± √ q2 +1) ⇔ x = sinh 1 ln (q+ √ q2 + 1) . a a a Ta chứng minh phương trình (3.2) có nghi m duy nhat. Th t v y, đ t x = sinht, ta đưoc phương trình 3 3 Từ đó, ta đưoc nghi m x = q 3 q+ √ q2 +1+ q 3 q− √ q2 +1. c. Nh n xét rang phương trình x3 + px = q luôn quy ve đưoc dạng (3.1) ho c (3.2) bang cách đ t x = my;m2 = ±4p. d. Xét phương trình ax3 + bx2 + cx + d = 0, a 0. Chia cả hai ve cho a, ta đưoc x3 + b x2 + c x + d = 0, a /= 0. Phương trình này luôn quy đưoc ve dạng (3.1) bang phép đ t y = x + b . 3a Bài toán 3.1. Giải phương trình x3 − 3x = 10. L i giai. Đ t x my ta đưoc m3 y3 3my 2015 chon m3 4 m 2 = − = 3m = 3 ⇒ = Thay vào ta đưoc 8y3 −6y = 10 ⇔ 4y3 −3y = 5 Áp dụng công thức nghi m của phương trình (3.1) ta đưoc suy ra y = q 3 x = 2 q 3 5+ √ 24+ q 3 5+ √ 24+ q 3 5 − √ 24 5− √ 24 . Bài toán 3.2. Giải phương trình x3 − 12x = −32.
  • 67. 62 Viết đề tài giá sinh viên – ZALO:0973.287.149-TEAMLUANVAN.COM √ 3 √ √ L i giai. Đ t x my ta đưoc m3 y3 12my 32 chon m3 4 m 4 = − = − 12m = 3 ⇒ = Thay vào ta đưoc 64y3 − 48y = −32 ⇔ 4y3 − 3y = −2 ⇔ 4(−y)3 − 3(−y) = 2. Đ t z = −y, thì 4z3 − 3z = 2. Áp dụng công thức nghi m của phương trình (3.1), ta đưoc suy ra z = q 3 2+ √ 3+ q 3 2 − √ 3, x = −4 q 3 2+ √ 3+ q 3 2− √ 3 . Bài toán 3.3. Giải phương trình x3 + 5x = 1. L i giai. Đ t x my ta đưoc m3 y3 5my 1 Chon m sao cho m3 4 thì m = 20 . 3 + = . 5m = 3 , = Thay vào phương trình đã cho, ta thu đưoc 20 r 20 y3 +5 r 20 y = 1 3 3 3 3 3 3 ⇔4y +3y = 5 √ 20 . Áp dụng công thức nghi m của phương trình (3.2) ta đưoc Suy ra y = s 3 3 √ 3+ √ 527 5 √ 20 + 3 √ 3 − √ 527 . 5 20 x = √ 20 s 3 3 √ 3+ √ 527 5 √ 20 + 3 √ 3− √ 527 . 5 20 Bài toán 3.4. Giải phương trình x3 − 3x2 + 4x+3 = 0. r s 3 s 3
  • 68. 63 Viết đề tài giá sinh viên – ZALO:0973.287.149-TEAMLUANVAN.COM s √ 2 2 2 2 L i giai. Ta sử dụng các bien đoi như sau x3 −3x2 +4x+3 = 0 ⇔ x3 −3x2 +3x−1+x+4 = 0 ⇔ (x−1)3 +(x−1) = −5. Đăt y = x − 1 thì phương trình đã cho trỏ thành y3 + y = −5. 3 3 m3 4 2 Đ t y = mz, ta đưoc m z + mz = −5. Chon m sao cho m = 3 hay m = √ 3 . Thay vào phương trình đã cho, ta thu đưoc 8 3 √ 3 z 2 + √ 3 z = −5 ⇔ 4z +3z = −15 √ 3 . 2 Áp dụng công thức nghi m của phương trình (3.2), ta đưoc hay z = s 3 −15 √ 3 + √ 676 + 3 2 −15 √ 3 − √ 676 2 2 x = √ 3 s 3 3 √ 3+ √ 527 5 √ 20 + 3 √ 3 − √ 527 5 20 + 1. Nh n xét 3.2. Ta nêu cách xây dụng các phương trình và các h quả liên quan: sinhu = sinhv ⇔ 2cosh u+v sinh u− v = 0 2 2 ⇔ sinh u−v ⇔ u = v, sinhu = −sinhv ⇔ 2sinh u+v cosh u −v = 0 2 2 ⇔ sinh u+v ⇔ u = −v, coshu = coshv ⇔ 2sinh u+v sinh u− v = 0 3 3 s 3
  • 69. 64 Viết đề tài giá sinh viên – ZALO:0973.287.149-TEAMLUANVAN.COM ⇔ — ⇔ √ √ 2 3 3 sinh3t = √ 2 ⇔ t = 3 ln √ 2 + ⇔ sinh u+v ⇔ u = −v 2 tanhu = tanhv sinh(u − v) cosh u cosh v tanhu = tanhv sinh(u + v) cosh u cosh v u = v = 0 ⇔ sinh(u − v) ⇔ u = v, = 0 ⇔ sinh(u + v) ⇔ u = −v, a2 −b2 = 1 ⇒ ∃u : coshu = |a|;sinhu = b. Bài toán 3.5. Giải phương trình 1+ √ 1 +x2 = x 5− 2 √ 1+ x2 . L i giai. Đ t x = sinh2t phương trình trỏ thành q 1 + √ 1 + sinh2 2t = x 5 − 2 √ 1 + sinh2 2t ⇔ q 1 + √ cosh2 2t = sinh2t 1 + 2 √ cosh2 2t ⇔ √ 1 + cosh2t = sinh2t (1 + 2cosh2t) ⇔ √ 2cosht = 2sinht cosht 1 + 2(1 + 2sin2 t) ⇔ √ 2 = 2sinht 3 + 4sin2 t 1 1 1+ √ 3 ! ⇔ x = sinh 2 ln 3 1+ √ 3 !! 1 1 √ 3 !2 1+ √ 3 !−2 ⇔ x = 2 √ 2 − √ 2 .
  • 70. 65 Viết đề tài giá sinh viên – ZALO:0973.287.149-TEAMLUANVAN.COM √ 2 3 3 sinh3t = √ 2 ⇔ t = 3 ln √ 2 + Bài toán 3.6. Giải phương trình √ 1+ √ 1+x2 = x 1+2 √ 1+x2 . L i giai. Đ t x = sinh2t phương trình trỏ thành q 1 + √ 1 + sinh2 2t = x 1 + 2 √ 1 + sinh2 2t q 1+ √ cosh2 2t = sinh2t 1+2 √ cosh2 2t ⇔ √ 1 + cosh2t = sinh2t (1 + 2cosh2t) ⇔ √ 2cosht = 2sinht cosht 1 + 2(1 + 2sin2 t) ⇔ √ 2 = 2sinht 3 + 4sin2 t 1 1 1+ √ 3 ! ⇔ x = sinh 2 ln 3 1+ √ 3 !! 1 1 √ 3 !2 1+ √ 3 !−2 ⇔ x = 2 √ 2 − √ 2 . Bài toán 3.7. Giải phương trình 16x3 + 7x = 3 √ 1 + x2. L i giai. Đ t x = sinht phương trình trỏ thành 16sinh3 t + 7sinht = 3 √ 1 + sinh2 t 3 ⇔ 16sinh t + 7sinht = 3cosht ⇔ 16sinh3 t + 12sinht = 5sinht + 3cosht
  • 71. 66 Viết đề tài giá sinh viên – ZALO:0973.287.149-TEAMLUANVAN.COM √ − − − √ − − ⇔ = ( + ) ⇔ 3 5 3 ⇔ 4sinh t + 3sinht = 4 sinht + 4 cosht. (∗) Do 5 2 3 2 = 1 ⇒ ∃u : coshu = 5 4 sinhu = 3 4 nên u = ln 2. (∗) ⇔ sinh3t = coshusinht + sinhucosht u ⇔ sinh3t = sinh(t + u) ⇔ t = 2 = ln 2 2 . Thay vào ta đưoc x = sinh ln 2 1 = 2 √ 2 . Bài toán 3.8. Giải phương trình 32x4 − 32x2 − 5x + 4 = 3 √ x2 − 1. L i giai. Đieu ki n x ≥ 1 x ≤ −1 Neu x ≥ 1 thì đ t x = cosht phương trình trỏ thành 32cosh4 t 32cosh2 t 5cosht + 4 = 3 cosh2 t 1 ⇔ 32cosh4 t − 32cosh2 t −5cosht + 4 = 3 sinh2 t ⇔ 4cosh4t = 5cosht +3|sinht|. Nh n xét 3.3. Nh n xét rang neu phương trình có nghi m t thì nó cũng có nghi m −t nên ta chỉ can xét t ≥ 0. Phương trình trỏ thành 4cosh4t = 5cosht + 3sint ⇔ cosh4t = 5 cosht + 3 sint 4 4 cosh4t cosh t ln 2 4t = t ln 2 4t = t +ln 2 ⇔ t = −ln 2 5 t = ln 2 3 ⇔ t = ln 2 3 . 2 4 4 −
  • 72. 67 Viết đề tài giá sinh viên – ZALO:0973.287.149-TEAMLUANVAN.COM 3 √ − − √ Thay vào phương trình ban đau, ta đưoc x = cosh ln 2 = √ 3 4+1 2 √ 3 2 . Neu x ≤ −1, đ t x = −cosht phương trình trỏ thành 32cosh4 t 32cosh2 t + 5cosht + 4 = 3 cosh2 t 1 ⇔ 32cosh4 t − 32cosh2 t + 5cosht + 4 = 3 sinh2 t ⇔ 4cosh4t = −5cosht + 3|sinht|. Nh n xét rang neu phương trình có t là nghi m thì −t cũng là nghi m, nên ta chỉ can xét t ≥ 0. Khi đó, phương trình trỏ thành 4cosh4t = −5cosht +3sint ⇔ cosh4t = − 5 cosht − 3 sint 4 4 ⇔ cosh4t = −cosh(t − ln 2) ⇔ 0/. 3.2 Khao sát m t so lỚp phương trình ch a hàm mũ và hàm hyperbolic Bài toán 3.9. Giải phương trình 9x (3x + 2x ) = 2x (8x + 7x ) + 5x (5x − 2x ). L i giai. Phương trình này chứa các hàm so mũ x, cơ so khác nhau. Ta sẽ dùng định lí Lagrange sau khi đã đưa đưoc bieu thức trên ve dạng f(a) = f(b). Ta có 9x (3x + 2x ) = 2x (8x + 7x ) + 5x (5x − 2x ) ⇔ 10x − 16x − 25x = 14x − 18x − 27x ⇔ 10x + 12x −16x −25x = 12x + 14x −18x −27x . (∗) Đen đây, ta xét hàm so f (t) = tx + (t + 2)x − (t + 6)x − (t + 15)x , t > 0, trong đó
  • 73. 68 Viết đề tài giá sinh viên – ZALO:0973.287.149-TEAMLUANVAN.COM − 0 0 x là nghi m của phương trình đã cho. Do đó (∗) ⇔ f(10) = f (12). Hàm f (t) liên tục trên đoạn [10, 12] nên theo định lí Lagrange, ton tại c ∈ (10, 12) sao cho f′(c) = f(10)− f(12) = 0. Do đó, neu x là nghi m của phương trình đã cho 10 12 thì nó phải thỏa mãn đieu ki n x[cx−1 +(c+2)x−1 −(c+6)x−1 −(c+15)x−1 ] = 0 ⇔x = 0∨cx−1 +(c+2)x−1 = (c+6)x−1 +(c+15)x−1 . Đȁng thức thứ hai cho ta x = 1 vì • Neu x < 1 thì x−1 < 0 nên cx−1 +(c+2)x−1 > (c+6)x−1 +(c+15)x−1 . • Neu x > 1 thì x−1 > 0 nên cx−1 +(c+2)x−1 < (c+6)x−1 +(c+15)x−1 . Thử trục tiep, ta thay hai giá trị x = 0, x = 1 thỏa mãn bài ra nên phương trình đã cho có hai nghi m là x = 0, x = 1. Bài toán 3.10. Giải phương trình ex +(x3 −x)ln (x2 +1) = e √ 3 x . L i giai. Ta de thay phương trình này có ba nghi m là x = 0, x = ±1. Tiep theo, ta sẽ chứng minh trong trưòng hop x /= 0, x /= ±1 thì phương trình đã cho vô nghi m . Th t v y, giả sử có so x0 /= 0, x0 Khi đó, rõ ràng là x0 /= √ 3 x0 và ±1 cũng là nghi m của phương trình đã cho. ex0 +(x3 −x0)ln (x2 +1) = e √ 3 x0. (3.3) Vì hàm so f(t) = et liên tục nên ton tại m t giá trị c nam giữa hai so x0, √ 3 x0 sao cho ex0 −e √ 3 x0 ec = x √ x . 0 − 3 0
  • 74. 69 Viết đề tài giá sinh viên – ZALO:0973.287.149-TEAMLUANVAN.COM 0 0 Nhưng từ (3.3), ta lại có ex0 − e √ 3 x0 = x0 − √ 3 x0 − x2 −x0 √ 3 x0 + q 3 x2 ln (x0 2 +1) < 0, tức là ec < 0, vô lí. Do đó, ta có đpcm. Phương trình đã cho chỉ có ba nghi m là x = 0, x = ±1. Bài toán 3.11. Cho a,b là các so thục vói a < b. Xét hàm f(x) = sinhx và g(x) = coshx. Chứng minh rang các phương trình sinha − sinhb = (a − b)coshx và cosha − coshb = (a − b)sinhx đeu có ít nhat m t nghi m thu c (a;b). L i giai. Áp dụng định lí giá trị trung bình Lagrange, ta thay neu f(x) khả vi trong (a,b) và liên tục trong [a.b] thì ton tại x1 ∈ (a;b) sao cho f(a)− f (b) = (a−b) f′(x1). M t khác, vì (coshx)′ = sinhx và (sinhx)′ = coshx nên ta có the áp dụng định lí giá trị trung bình Lagrange, thì ton tại x1,x2 ∈ (a;b) sao cho sinha − sinhb = (a − b)coshx1 và cosha − coshb = (a − b)sinhx2, ta thu đưoc đieu can chứng minh.
  • 75. 70 Viết đề tài giá sinh viên – ZALO:0973.287.149-TEAMLUANVAN.COM − 2 b − a b a x0 2 b a x0 2 Bài toán 3.12. Cho 2 so thục 0 < a < b. Chứng minh rang phương trình xcoshx sinhx = asinhb − bsinha b − a có ít nhat m t nghi m thu c (a;b). L i giai. Xét các hàm so g(x) = sinhx và h(x) = 1 trong (a,b). Ta có g′(x) = xcoshx − sinhx , h′ (x) = x x x −1 . x2 Theo định lý Cauchy thì ton tại x0 ∈ (a;b) sao cho [h(b) − h(a)]g′(x0) = [g(b) − g(a)]h′(x0), hay 1 − 1 x0 coshx0 −sinhx0 = sinhb − sinha −1 . Do đó (a − b)(x0 coshx0 − sinhx0) = − a sinhb − bsinha . bax0 2 abx0 2 Suy ra x0 coshx0 − sinhx0 = asinhb − bsinha . V y phương trình xcoshx − sinhx = asinhb −bsinha có ít nhat m t nghi m thu c (a;b). Bài toán 3.13. Cho a > nghi m thục duy nhat. b − a 0 chứng minh rang phương trình aex = 1 + x + x 2 có m t L i giai. Trưóc het, xét hàm so g(x) = e−x f(x). Ta có g′(x) = e−x [ f ′(x) − f (x)] > 0. Do v y g là hàm tăng nên g(x) > g(x0) = 0,∀ 2 x > x0. V y f (x) > 0,∀x > x0. Áp dụng vói hàm so f (x) = aex −1−x − x . Khi đó lim f (x) = −∞ và lim f (x) = 2 x→+∞ x→+∞ +∞. Do v y f có ít nhat m t nghi m thục. Ta chứng minh f có nghi m duy nhat. 2
  • 76. 71 Viết đề tài giá sinh viên – ZALO:0973.287.149-TEAMLUANVAN.COM 1 và axbx — − — − Th t v y, ∀x ∈ R ta có f ′(x) = aex − 1− x > f (x). Theo nh n xét trên thì phương trình có m t nghi m duy nhat. Bài toán 3.14. Giải phương trình 4|x| + 2|x| = 4x + 2. L i giai. Theo bat đȁng thức Bernoulli tα + α −1 > αt ∀t > 1,α > 1 tα + α −1 < αt ∀t > 1,0 < α < 1 suy ra 2x ≥ x+1, 4x ≥ 3x +1 khi x ≥ 2x ≤ x+1 4x ≤ 3x+1 khi x ∈ [0;1) Khi x < 0 thì ve trái của phương trình nhỏ hơn ve phải của phương trình. Đȁng thức xảy ra khi và chỉ khi x = 0 và x = 1, từ đó suy ra phương trình có nghi m x = 0, x = 1. Bài toán 3.15. Giải phương trình 41+x +41−x = 2x +2−x +31+x +31−x . L i giai. Sử dụng bat đȁng thức a > b > 1 thì ax + a−x ≥ bx + b−x , ∀x ∈ R. ⇔ (ax −bx )+ a−x − b−x ≥ 0 ⇔ (ax − bx ) 1 − 1 ≥ 0. Neu x > 0 thì (ax bx ) > 0;1 1 (ab) Neu x < 0 thì (ax bx ) < 0;1 1 (ab) x > 0. x < 0. Neu x = 0 thì ta có đȁng thức
  • 77. 72 Viết đề tài giá sinh viên – ZALO:0973.287.149-TEAMLUANVAN.COM 1 Áp dụng vào bài toán đã cho, ta có 4x + 4−x ≥ 2x + 2−x . 4x + 4−x ≥ 3x + 3−x . Do v y 4[4x + 4−x ] ≥ 2x + 2−x + 3(3x + 3−x ). Dau = xảy ra khi và chỉ khi x = 0. V y phương trình có nghi m duy nhat x = 0. Bài toán 3.16 (Tuyen t p Olympic 30 tháng 4, lan XII - 2006). Giải phương trình 3x3+x+2 + x3 − 3x + 1 .32x−x3 = 34x+1. L i giai. Ta có 3x3+x+2 + x3 − 3x + 1 .32x−x3 = 34x+1 ⇔ 3x3+x+2 .3x3−2x + x3 − 3x + 1 .32x−x3 .3x3−2x = 34x+1 .3x3−2x ⇔ 32x3−x+2 + x3 − 3x+ 1 = 3x3+2x+1 ⇔ 32x3−x+2 + 2x3 − x + 2 − x3 +2x + 1 = 3x3+2x+1 ⇔ 32x3−x+2 + 2x3 − x+ 2 = 3x3+2x+1 + x3 + 2x + 1 . Đ t f (t) = 3t +t,u = 2x3 − x + 2,v = x3 + 2x + 1. Phương trình trỏ thành f (u) = f (v), f ′(t) = 3.ln 3+1 > 0, ∀t ∈ R nên f (t) đong bien trên R. V y nên f (u) = f (v) ⇔ u = v. V y nên 2x3 −x+2 = x3 +2x+1 ⇔ x3 −3x+1 = 0. Đ t g(x) = x3 −3x+1. Ta có g(x) liên tục trên R và g(1) = −1 < 0,g(2) = 3 > 0. Đ t x = 2cosα; α ∈ (0;π) thì phương trình x3 −3x+1 = 0 trỏ thành: 8cos3 α − 6cosα + 1 = 0 ⇔ 2.cos3α = −1 2π ⇔ cos3α = − 2 ⇔ α = ± 9 + k2π 3 .(k ∈ Z)
  • 78. 73 Viết đề tài giá sinh viên – ZALO:0973.287.149-TEAMLUANVAN.COM 2 V y phương trình đã cho có ba nghi m x1 = 2cos 2π ;x2 = 2cos 4π ;x3 = 2cos 8π . 9 9 9 Bài toán 3.17. Tìm nghi m nguyên dương a và b thỏa mãn phương trình ab = ba . L i giai. Ta thay (a,b) = (n,n),n ∈ N∗ là nghi m. ln x Xét trưòng hop a b. Giả sử a < b. Xét hàm so f(x) = x . Rõ ràng là ab = ba khi và chỉ khi f(a) = f(b). Ta có f ′(x) = 1 − ln x , do đó f x) tăng trên (0,e) và giảm trên (e,∞). x Như v y, đe f(a) = f(b) thì 0 < a < e,b > e. Ta có a ∈ {1,2},b ∈ {3,4, ......}. Vói a = 1, bài toán không có lòi giải. Vói a = 2, ta có b = 4. Rõ ràng là 24 = 42 . Ket lu n: Phương trình đã cho có nghi m (a,b) = (n,n), n ∈ N∗ , (a,b) = (2,4) và (a,b) = (4,2).
  • 79. 74 Viết đề tài giá sinh viên – ZALO:0973.287.149-TEAMLUANVAN.COM Ket lu n Lu n văn “M t so dạng toán cục trị trong lóp hàm mũ và hàm hyperbolic” đã trình bày những van đe sau: - Lu n văn trình bày chi tiet m t so đȁng thức và bat đȁng thức cơ bản liên quan đen hàm mũ và hàm hyperbolic. - Tiep theo trình bày chi tiet các dạng toán cục trị trong lóp hàm mũ và hàm hyperbolic và nêu các phương pháp khảo sát chúng. - Cuoi cùng, lu n văn trình bày các dạng toán liên quan đen phương trình, bat phương trình và m t so bài toán chon loc từ các đe thi HSG quoc gia, Olympic khu vục và quoc te.
  • 80. 75 Viết đề tài giá sinh viên – ZALO:0973.287.149-TEAMLUANVAN.COM Tài li u tham khao A Tieng Vi t [1] Nguyen Văn M u 2006, Bat đȁng thŕc, định lí và áp dnng, NXB Giáo dục. [2] Nguyen Văn M u (Chủ biên) 2010, So phŕc và áp dnng, NXB Giáo dục. [3] Nguyen Văn M u, Phạm Thị Bạch Ngoc, 2003, M t so bài toán chon loc ve lượng giác, NXB Giáo dục. [4] Nguyen Văn M u, Lê Ngoc Lăng, Phạm the Long, Nguyen Minh Tuan (2006), Các đe thi olympic Toán sinh viên toàn quoc, NXB Giáo dục. [5] Tạ Duy Phưong, Hoàng Minh Quân (2017), Phương trình b c ba với các h thŕc hình hoc và lượng giác trong tam giác, NXBGD Vi t Nam. [6] Tạp chí TH&TT (2007), Các bài thi Olympic Toán trung hoc phő thông Vi t Nam (1990-2006), NXB Giáo dục. [7] Trương Đức Thịnh (2015), Đȁng thŕc và bat đȁng thŕc trong lớp hàm hyper- bolic và áp dnng, Lu n văn Thạc sĩ, ĐH Thái Nguyên. B Tieng Anh [8] Paulo Ney de Sauza, Jorge- Nume Silva (1998), Berkeley Problems in Mathe- matics, Springer. [9] T-L.T. Radulescu, V.D. Radulescu, T.Andreescu (2009). Problems in Real Analysis: Advanced Calculus on the real axis. Springer.